0% found this document useful (0 votes)
521 views348 pages

Seam 6 Main Modular 2020

The document discusses the trapezoidal rule for approximating the definite integral of a function between bounds. It defines the trapezoidal rule, explaining that the area under the curve is divided into trapezoids rather than rectangles. The rule takes the average of the left and right sums. It provides the formula for calculating the approximation using n subintervals of equal width. Examples are given to illustrate how the area of each trapezoid is calculated and how they sum to the total approximation.

Uploaded by

Mark Anthony Lim
Copyright
© © All Rights Reserved
We take content rights seriously. If you suspect this is your content, claim it here.
Available Formats
Download as DOCX, PDF, TXT or read online on Scribd
0% found this document useful (0 votes)
521 views348 pages

Seam 6 Main Modular 2020

The document discusses the trapezoidal rule for approximating the definite integral of a function between bounds. It defines the trapezoidal rule, explaining that the area under the curve is divided into trapezoids rather than rectangles. The rule takes the average of the left and right sums. It provides the formula for calculating the approximation using n subintervals of equal width. Examples are given to illustrate how the area of each trapezoid is calculated and how they sum to the total approximation.

Uploaded by

Mark Anthony Lim
Copyright
© © All Rights Reserved
We take content rights seriously. If you suspect this is your content, claim it here.
Available Formats
Download as DOCX, PDF, TXT or read online on Scribd
You are on page 1/ 348

Issue No.

1 Page 1 of 348
ST. THERESE- MTC COLLEGES SEAMANSHIP 6
Iloilo, Philippines (Advance Trim, Stability and Stress)
STUDENT LEARNING MODULE
Revision No. 1 Effectivity date: Reviewed by: Approved by:

10 August 2020 QMR President

Lesson 1: Calculation of Areas and Volumes

Learning Module 1.1: Approximate Calculation of Areas and


Volumes

Competence, Course Outcomes and Learning Outcomes

Competence:

Control trim, stability and stress

Course Outcome:

1. Explain the importance of maintaining stability during loading, unloading


and in-transit in various conditions.
2. Calculate the effect on trim and stability of a ship in the event of damage
to and consequent flooding of a compartment and countermeasures to be
taken.

Learning Outcomes:

At the end of the lesson, the student should be able to:

1. Discuss the trapezoidal rule for the area under a curve in terms of the number of
ordinates, the interval and the ordinate value
2. Use the trapezoidal rule to calculate the area under curve defined by given ordinates

Overview

Ship stability is a difficult concept to define. The idea of a stable character is readily
grasped, although it is harder to put into words. The same applies to the notice of a stable
ship, as stability is an intangible and complex entity.

Nevertheless, it plays an extremely important role in navigation. If a ship is unstable, there


is immediate danger to ship, crew, passengers and the environment. Therefore, stability
regulations have been established for all types of ships. Every crew member of every type of
ship is well versed in this domain.
Issue No. 1 Page 2 of 348
ST. THERESE- MTC COLLEGES SEAMANSHIP 6
Iloilo, Philippines (Advance Trim, Stability and Stress)
STUDENT LEARNING MODULE
Revision No. 1 Effectivity date: Reviewed by: Approved by:

10 August 2020 QMR President

The strength of this module is that stability is made visible, accessible and understandable
for every student’s though many figures and drawing contained within.

All the necessary topics are addressed and are easy to understand with the help of clear
illustration. The terminology and abbreviations conform to international usage as much as
possible.

Discussion

1.1Trapezoid Rule Definition

y = f(x)

0 a x1 x2 x3 x4 …………………x1……xn-1 b x
Figure 1.1

Trapezoidal Rule is a rule that evaluates the area under the curve by dividing the total area
into smaller trapezoids rather than using rectangles. Trapezoids Rule integration works by
approximating the region under the graph of a function as a trapezoid and it calculates the
area. It takes the average of the left and the right sum. The Trapezoidal Rule does not give
accurate value as Simpson’s Rule when the underlying function is smooth. It is because
Simpson’s Rule uses the quadratic approximation instead of linear approximation. Both
Simpson’s Rule and Trapezoidal Rule give the approximation value but Simpson’s Rule
results in even more accurate approximation value of the integrals.

Let f(x) be a continuous function on the interval [a,b]. Now divide the intervals [a,b] into
equal subintervals with each of width,

b−a
∆ x= , Such thatx0 <x1<x2<x3<……..<xn = b
n
Issue No. 1 Page 3 of 348
ST. THERESE- MTC COLLEGES SEAMANSHIP 6
Iloilo, Philippines (Advance Trim, Stability and Stress)
STUDENT LEARNING MODULE
Revision No. 1 Effectivity date: Reviewed by: Approved by:

10 August 2020 QMR President


b

Then the Trapezoidal Rule formula for area approximating the definite integral ∫ f (x)dx is
a

given by:

b
∆h
∫ f (x)dx ≈ T n =
2
[f(x0) + 2f(x1) + 2f(x2)+2f(x3) + …+ 2f(xn-1) + f(xn)]
a

Many applications of calculation involve definite integrals. If we can find an anti-derivative


for the integrand, then we can evaluate the integral fairly easily. When we cannot, we turn
to numerical methods. The numerical method we will discuss here is called the Trapezoid
Rule. Although we often can carry out the calculations by hand, the method is most
effective with the use of a computer or programmable calculator. But at the moment let’s
concern ourselves with these details. We will describe the method first, and then consider
ways to implement it.
ƒ

Area 1 Area 2 Area 3

y0 y1 y2 y3
h h h
a= x 0 x1 x2 x3 = b
Figure 1.2

The general idea is to use trapezoids instead of rectangles to approximate the area under
the graph of a function. A trapezoids looks like a rectangle except that it has a slanted line
for a top. Working on the interval [ a,b], we subdivide it into n subintervals of equal width
h= (b – a)/n. this gives rise to the partition a = x0 ≤ x1 ≤ x2 ≤ ….≤ xn = b, where for
each j, xj = a + jh, 0 ≤ j ≤ n. Moreover, we let yj = ƒ(xj), 0 ≤ j ≤ n. That is, the vertical
edges go from the x-axis to the graph of ƒ. Consult the sketch above where we have
shown a finite number of subintervals.

If we are going to use trapezoids instead of rectangles as our basic area elements, then we
have to have a formula for the area of a trapezoid.

yR – yL

yR
Issue No. 1 Page 4 of 348
ST. THERESE- MTC COLLEGES SEAMANSHIP 6
Iloilo, Philippines (Advance Trim, Stability and Stress)
STUDENT LEARNING MODULE
Revision No. 1 Effectivity date: Reviewed by: Approved by:

10 August 2020 QMR President

yL

h
Figure 1.3

With reference to the sketch above, the area of a trapezoid consists of the area of the
rectangle plus the area of the triangle, or hyL + (h/2)(yR – yL) = h(yL + yR)2. So the area
is h times the average of the lengths of the two vertical edges.

Now, we return to the original problem of finding the definite integral of a function ƒ
defined on the interval [a,b]. We define the Trapezoidal Rules as follows.
b

Definition: The n – subinterval trapezoid approximation to∫ ƒ (x) dx is given by:


a

Where:∫ f = definite integral of a function ƒ defined on the interval [ a,b].


a

Σ= sum total
h
Tn = (y0 + 2y1 + 2y2 +2y3 + …+ 2yn-1 + yn)
2

( )
n-1

=
h
2
y0 + yn + 2
Σy
J =1
j

To see where the formula comes from, let’s carry out the process of adding the area of the
trapezoids. Refer to the original sketch (Figure 1.1), and use the formula we derived for the
area of a trapezoid. Note that when we add the areas of the trapezoids starting on the left,
the area of the first, second, and third are:
h
Area 1 = (y0 + y1)
2
h
Area 2 = (y1 + y2)
2
h
Area 3 = (y2 + y3)
2
So, y0 and y3, the first and last, each appear once; and all the other yj’s appear exactly
twice. We can see from this example that there will be a similar pattern no matter the
number of trapezoids: The first and the last vertical edge appears once and all other vertical
edges appear two times when we sum the areas of the trapezoids. This is exactly what the
Trapezoid Rule entails in the formula.
Issue No. 1 Page 5 of 348
ST. THERESE- MTC COLLEGES SEAMANSHIP 6
Iloilo, Philippines (Advance Trim, Stability and Stress)
STUDENT LEARNING MODULE
Revision No. 1 Effectivity date: Reviewed by: Approved by:

10 August 2020 QMR President


1.2 Trapezoid Rule Examples

Process me

Example 1:

Approximate the area under the curve y= f(x) between x=0 and x=8 using the trapezoidal
Rule with n= 4 subintervals. A function f(x) is given in the table of values.

X 0 2 4 6 8
f(x) 3 7 11 9 3

y = f(x)
9
11
7
3

0 2 4 6 8 x
Figure 1.4

Required: Approximate the area under the curve.


Given: n=4
Solution:

The Trapezoidal Rule formula for n=4 subintervals is given as:

∆x
T4 = 2 [f(x0) + 2f(x1) + 2f(x2)+2f(x3) + f(x4)]

Here the subinterval width∆ x = 2.

Now, substitute the values from the table, to find the approximate value of the area under
the curve.
Issue No. 1 Page 6 of 348
ST. THERESE- MTC COLLEGES SEAMANSHIP 6
Iloilo, Philippines (Advance Trim, Stability and Stress)
STUDENT LEARNING MODULE
Revision No. 1 Effectivity date: Reviewed by: Approved by:

10 August 2020 QMR President


2
A ≈ T4 = [3 + 2(7) + 2(11)+2(9) + 3]
2

2
A ≈ T4 = (3 + 14 + 22+ 18 + 3]
2

A ≈ T4 = 60 m2

Example 2:
Approximate the area under the curve y= f(x) between x=-4 and x=2 using the Trapezoidal
Rule with n= 6 subintervals. A function f(x) is given in the table of values.

X -4 -3 -2 -1 0 1 2
f(x) 0 4 5 3 10 11 2

11
10
9
8
7
y = f(x)
6
5
4
3
2
1

-4 -3 -2 -1 0 1 2 x

Figure 1.5

Required: Approximate the area under the curve.


Given: n=6

Formula:
Issue No. 1 Page 7 of 348
ST. THERESE- MTC COLLEGES SEAMANSHIP 6
Iloilo, Philippines (Advance Trim, Stability and Stress)
STUDENT LEARNING MODULE
Revision No. 1 Effectivity date: Reviewed by: Approved by:

10 August 2020 QMR President


The Trapezoidal Rule formula for n=6 subintervals is given as:

∆x
T6= 2 [f(x0) + 2f(x1) + 2f(x2)+2f(x3) + 2f(x4) + 2F(X5) + f(x6)]

Here the subinterval width∆ x = 1.

Solution:

Now, substitute the values from the table, to find the approximate value of the area under
the curve.

1
A ≈ T6 = [0 + 2(4) + 2(5)+2(3) + 2(10) + 2(11) +2]
2
1
A ≈ T6 = (8 + 10 + 6+ 20 + 22 + 2)
2
1
A ≈ T6 = (68)
2
68
A ≈ T6 =
2
A ≈ T6 = 34 m2

Therefore, the approximate value of area under the curve using Trapezoids Rule is 34 m2.

Do this

Activity 1 – Trapezoid Rule

Calculate the approximate area under the curve y= f(x) between x=0 and x=10 using the
Trapezoidal Rule with n=5 subintervals. The subinterval width ∆ x = 2.

X 0 2 4 6 8 10
f(x) 4 6 6 4 4 5
Issue No. 1 Page 8 of 348
ST. THERESE- MTC COLLEGES SEAMANSHIP 6
Iloilo, Philippines (Advance Trim, Stability and Stress)
STUDENT LEARNING MODULE
Revision No. 1 Effectivity date: Reviewed by: Approved by:

10 August 2020 QMR President

a. Answer

Assessment

Assessment 1 – Trapezoidal Rule

A function ƒ(x) is given by the tables of values. Calculate the approximate the area under
the curve y= f(x) between x=0 and x=8 using the Trapezoidal Rule with n=4 subintervals.

X 0 2 4 6 8
f(x) 2 6 10 8 2
The subinterval width∆ x = 2.

Write your solutions and answer in the space provided.


Issue No. 1 Page 9 of 348
ST. THERESE- MTC COLLEGES SEAMANSHIP 6
Iloilo, Philippines (Advance Trim, Stability and Stress)
STUDENT LEARNING MODULE
Revision No. 1 Effectivity date: Reviewed by: Approved by:

10 August 2020 QMR President

Lesson 2: Simpson’s Rule

Learning Module 2.1: Simpson’s Rule

Competence, Course Outcomes and Learning Outcomes

Competence:

Control trim, stability and stress

Course Outcome:

1. Explain the importance of maintaining stability during loading, unloading


and in-transit in various conditions.
2. Calculate the effect on trim and stability of a ship in the event of damage
to and consequent flooding of a compartment and countermeasures to be
taken.

Learning Outcomes:

At the end of the lesson, the student should be able to:

1. Interpret knowledge of the theories of the rule in the construction and stability of the
ship:
– Simpson’s First Rule;
– Simpson’s Second Rule;
– Simpson’s Third Rule
2. Calculate the area of the water-plane and stability of the ship using the three
Simpson’s Rule

Overview
In this lesson we will introduce Simpson’s Rules and demonstrate how you can apply them
to calculate water-plane areas. We will also introduce area calculations for vessel’s with
appendages and how to use combine Simpson’s Rules when calculating water-planes areas.
Issue No. 1 Page 10 of 348
ST. THERESE- MTC COLLEGES SEAMANSHIP 6
Iloilo, Philippines (Advance Trim, Stability and Stress)
STUDENT LEARNING MODULE
Revision No. 1 Effectivity date: Reviewed by: Approved by:

10 August 2020 QMR President

Discussion

Areas and Volumes

Simpson’s Rule may be used to find the areas and volumes of irregular figures. The rules are
based on the assumption that the boundaries of such figures are curves which follow a
definite mathematical law. When applied to ships they give a good approximation of areas
and volumes. The accuracy of the answers obtained will depend upon the spacing of the
ordinates and upon how near the curve follows the law.

Area of plane Figures

The areas of certain common plane figures are often used in stability calculations.

Square – Where a is the length of each side: –


Area = a2
Rectangle – Where a, b and c are the lengths of the sides: –
Area = a x b
Triangle – Where a, b and c are the lengths of the respective sides; h the perpendicular
height; and Ø the angle between a and b: –

Area = bh
2
a Area = ab x sin Ø
c
h 2

Ø b Area = √s(s –a)(s-b)(s-c)


Figure 2.1
Where s =½(a + b + c)

Trapezium or Trapezoid – is a four-sided figure, having two of its sides parallel.

Where a and b are the lengths of the parallel sides, and h the perpendicular distance
between them:

Area = h_ (a + b) b
2 h
a

Figure 2.2

Simpson’s Rules for Calculating Areas Under the Curves


Issue No. 1 Page 11 of 348
ST. THERESE- MTC COLLEGES SEAMANSHIP 6
Iloilo, Philippines (Advance Trim, Stability and Stress)
STUDENT LEARNING MODULE
Revision No. 1 Effectivity date: Reviewed by: Approved by:

10 August 2020 QMR President

Another technique for approximating the value of a definite integral is called Simpson’s Rule.
Whereas the main advantage of Trapezoidal rule is its rather easy conceptualization and
derivation, Simpson’s rule approximation usually achieve a given level of accuracy faster.
Moreover, the derivation of Simpson’s rule is only marginally more difficult. Both rules are
examples of what we refer to as numerical methods.

In the Trapezoid rule method, we start with rectangular area elements and replace their
horizontal-lime tops with slanted lines. The area-elements used to approximate, say, the
area under the graph of a function and above a closed interval then become trapezoids.
Simpson’s method replaces the slanted-line tops with parabolas.

Though two points determine the equation of a line, three are required for a parabola. We
also need to develop a formula for the area of a parabolic-top area element if the sum of
such areas is to become the Simpson approximation.

Finding the area of a water plane is one of the important topics that you will learn in the
ship stability course. You will be using Simpson’s Rules for finding areas under a curve for
this purpose. These are three different variations of this rule. We will review them one by
one.

Simpson’s Rules use ordinates to calculate the water-plane area. The rules also require that
one side of the area we are trying to calculate must be a straight line.

First of all let us recall that an ordinate is the y-coordinate of a point which defines the
vertical distance from a horizontal axis.

When calculating water plane areas, an


ordinate is the vertical distance
between two points on the water plane, as
shown in figure 2.3.
Ordinate
The midline will divide the ordinates in
two equal parts. Hence the ordinates on
either side of the midline are referred to as
Figure 2.3
half ordinates or semi-ordinates as
shown in Figure 2.4.

Figure 2.4
Issue No. 1 Page 12 of 348
ST. THERESE- MTC COLLEGES SEAMANSHIP 6
Iloilo, Philippines (Advance Trim, Stability and Stress)
STUDENT LEARNING MODULE
Revision No. 1 Effectivity date: Reviewed by: Approved by:

10 August 2020 QMR President

The shape of the water plane is symmetrical about the midline, therefore it will be sufficient
to calculate the area of one half the shape and then double the answer to find the total
area. The midline satisfies the straight line boundary requirement of Simpson’s Rules.

The following steps are common for all three versions of Simpson’s Rules:

1. Divide the water plane onto two halves using the midline.
2. Choose on half to work with.
3. Sub-divide the chosen half into area strips or sections of equal width, “h”.
4. Based on the number of area sections and hence the number of half ordinates,
choose which version of Simpson’s rules to apply.
5. In some cases you may have combine two rules to calculate the area.

Simpson’s Rules provide a simple means of calculating areas under curves without having to
resort to integration techniques, which can be somewhat confusing for the non-
mathematician.

Simpson’s Rules were designed for finding the area under two types of curve. The first
and the Five Eight rule are used when the curve is a parabola. The second rule is used when
the curve is a cubic curve. These curves are similar to the shapes of the edge of water-
planes and other ship-sections and we can use them to find areas and volumes of the ship
shapes with sufficient accuracy for practical purposes. All rules are equally accurate but the
first is usually used for preference.

The preliminary steps in calculating the area of a water-plane or section are as follows. A
number of equidistant points are taken along the center line and perpendiculars are dropped
from these points to meet the curved sides. The lengths of these perpendiculars are
measured and also the distance between them. The perpendiculars are called “Ordinates”
and the distance between them, the “Common Interval”. The latter is usually denoted in
formulae as “h”.

Figure 2.5 represents a water-plane. In this case, the center line (AB) is divided into six
equal parts, each having a length of h (the common interval). The ordinates are HH, JJ, KK,
etc. A and B are also ordinates, although in this case they have no length.
J K L
H M

A h h h h h h
B
C D E F G

H1 M1
J1 K1 L1
Figure 2.5
Issue No. 1 Page 13 of 348
ST. THERESE- MTC COLLEGES SEAMANSHIP 6
Iloilo, Philippines (Advance Trim, Stability and Stress)
STUDENT LEARNING MODULE
Revision No. 1 Effectivity date: Reviewed by: Approved by:

10 August 2020 QMR President

It will be noticed that half of the figures has been drawn in plain lines and half in dotted
lines. The perpendicular distances shown in the plain lines (CH, DJ, EK, etc.) for the half-
water-plane are usually called “Half Ordinates”’ in order to distinguish them.

When a ship’s plans are drawn, they usually show only the half-water-plane. It is easier, in
practice, to measure the half-ordinates from the plans, to put them through the Rules and
then to double the half-ordinates from the plans, to put them through the Rules and then to
double the half-area so found to give the whole area.

The half ordinates, put through the rules, give the area of the half-water-plane; the
ordinates will give the area of the whole water-plane, when put through the same rules.

Simpson’s Rules provide a simple means of calculating areas under curves without having to
resort to integration techniques, which can be somewhat confusing for the non-
mathematician.

There are three Rules that are used to calculate areas under the curves of statical stability,
these being necessary to verify that a ship’s proposed loaded condition complies with the
intact stability criteria laid down by IMO.

1. Simpson’s First Rule

This rule is also called the 3-ordinate rule because you need at least three ordinates to be
able to apply this rule. When you have an even number of strips and hence an odd
number of half ordinates, you can use Simpson’s First Rule.

In its simplest form, this rule states: The area between any three consecutive ordinates is
equal to the sum of the end ordinates, plus four times the middle ordinate, all multiplied by
one-third of the common interval.

t u v w x y
h h h h h h z

1 4 2 4 2 4 1
Figure 2.6

Consider in Figure2.6 the area contained between the half-ordinates t and v. If the common
h
interval is h, this area equals: (t + 4u + v)
3
The total area of the half water-plane can be obtained by finding, in the same way, the area
between v and x, and x and z, and taking the sum of the three.
Issue No. 1 Page 14 of 348
ST. THERESE- MTC COLLEGES SEAMANSHIP 6
Iloilo, Philippines (Advance Trim, Stability and Stress)
STUDENT LEARNING MODULE
Revision No. 1 Effectivity date: Reviewed by: Approved by:

10 August 2020 QMR President


h
Area between t and v = (t + 4u + v)
3
h
Area between v and x = (v + 4w + x)
3
h
Area between x and z = (x + 4y + z)
3
h h h
Total area = (t + 4u + v) + (v + 4w + x) + (x + 4y + z)
3 3 3

The numbers by which the successive half ordinates, or ordinates, are multiplied (in this
case 1, 4, 2, 4, 2 ,4, -----1) are called “Simpson’s Multipliers”.

From the above, we can see two things about this rule:-

a) it can be used when, and only when, an odd number (e.g. 3, 5, 7, 9 etc.) of
ordinates are taken.
b) The area is found by multiplying successive ordinates, including the ends, by the
multipliers 1, 4, 2, 4, 2, 4,-----1, adding the results together, and then multiplying by
one-third of the common interval.

Note that “h” is also called the common interval or CI, and represents the width of each of
the area sections or strips.

Consider the shape shown:

A B C
Issue No. 1 Page 15 of 348
ST. THERESE- MTC COLLEGES SEAMANSHIP 6
Iloilo, Philippines (Advance Trim, Stability and Stress)
STUDENT LEARNING MODULE
Revision No. 1 Effectivity date: Reviewed by: Approved by:

10 August 2020 QMR President

h h

1 4 1
Figure 2.7

Let the curve in Figure 2.7 be a parabola of the second order. Let A, B and Care known as
ordinates, equally spaced at “h” units apart, having lengths expressed in meters.

The spacing between each of the ordinates must be the same, with h being the distance
between the ordinates, known as the Common Interval, expressed in meters.

To calculate the area of the shape: AREA = 1/3 x h x (1A + 4B + 1C) where the
numbers 1, 4 and 1 are known as Simpson’s Multipliers.

Let the first ordinate = a the second ordinate = b, the third ordinate = c, etc.

The area can be found by:


1
Area = x h x (1a + 4b + 1c) in the case of 3 ordinates
3

h h
or Area = (1a + 4b + 1c) or Area = (a + 4b + c)
3 3

1
Area = x h x (1a + 4b + 2c + 4d + 1e) in the case of 5 ordinates
3

h h
or Area = (1a + 4b + 2c + 4d + 1e) or Area = (a + 4b + 2c + 4d + e)
3 3
1
Area = x h x (1a + 4b + 2c + 4d + 2e + 4f + 1g) in the case of 7 ordinates
3

h
or Area = (1a + 4b + 2c + 4d + 2e + 4f + 1g) in the case of 7 ordinates
3

h
or Area = (a + 4b + 2c + 4d + 2e + 4f + g) in the case of 7 ordinates
3

and so on Simpson’s First Rule being that the multipliers follow the configuration of 1,4,1 or
1,4,2,4,1 or 1,4,2,4,2,4,1, etc. according to the number of ordinates. The greater the
number of ordinates, more accurate is the answer. You can think of the multipliers as set of
“141” e.g.
1 4 1 for 3 ordinates
+ 1 4 1
Issue No. 1 Page 16 of 348
ST. THERESE- MTC COLLEGES SEAMANSHIP 6
Iloilo, Philippines (Advance Trim, Stability and Stress)
STUDENT LEARNING MODULE
Revision No. 1 Effectivity date: Reviewed by: Approved by:

10 August 2020 QMR President

1 4 2 4 1 for 5 ordinates
+ 1 4 1

1 4 2 4 2 4 1 for 7 ordinates
Thus:
With 3 ordinates the Simpson’s multipliers are: 141
With 5 ordinates the Simpson’s multipliers are: 14241
With 7 ordinates the Simpson’s multipliers are: 1424241
With 9 ordinates the Simpson’s multipliers are: 142424241 and so on……..

In the formula, “h” is the common interval”, i.e. the distance between each pair of
ordinates.

Note: The common interval “h” MUST remain constant throughout, or the Rule will not
work.

Process me
Let’s try an example to better understand how to use the formula:

Example 1

A section of steel plate to be used in the construction of a ship’s deck has dimensions as
shown. Calculate the area of the plate.

4.3 m
5.4 m
1.9 m

h=6.3 h=6.3
m m
Figure 2.8

Two approaches to the solution are shown.

First of all, we need to check if we can apply Simpson’s First Rule to calculate the area:

Number of ordinates = 3 and since 3 is odd number, we can use Simpson’s First Rule to
find the area of this water-plane.
Issue No. 1 Page 17 of 348
ST. THERESE- MTC COLLEGES SEAMANSHIP 6
Iloilo, Philippines (Advance Trim, Stability and Stress)
STUDENT LEARNING MODULE
Revision No. 1 Effectivity date: Reviewed by: Approved by:

10 August 2020 QMR President


Common interval “h” = 6.3 meters

1. Given: Common interval (h) = 6.3 meters


1st ordinate value (a) = 5.4 meters
2ndordinate value (b) = 4.3 meters
3rd ordinate value (c) = 1.9 meters

2. Formula:
h
Area of the plate = (1a + 4b + 1c)
3

3. Solution (1):
6.3
Area of the plate = [(1x5.4) + (4 x 4.3) + (1 x 1.9)]
3

6.3
Area of the plate = (5.4 + 17.2 + 1.9)
3
6.3
Area of the plate = (24.5)
3
154.35
Area of the plate =
3

Area of the plate = 51.45 m2

4. Answer: Area of plate = 51.54 m2

The calculation is very tedious in the format shown. A tabular approach makes the
calculation much easier as follows.

Solution (2)
h
Area of the plate = (1a + 4b + 1c)
3
Using a table, first calculate what is in the brackets

No. Ordinates
(t) SM(Simpson’s Product
multiplier)
a 5.4 1 5.4
b 4.3 4 17.2
c 1.9 1 1.9
(Sum) Σ1 24.5
Issue No. 1 Page 18 of 348
ST. THERESE- MTC COLLEGES SEAMANSHIP 6
Iloilo, Philippines (Advance Trim, Stability and Stress)
STUDENT LEARNING MODULE
Revision No. 1 Effectivity date: Reviewed by: Approved by:

10 August 2020 QMR President

Σ1is used because it is a total using Simpson’s First Rule.


h
Area of plate = (Σ )
3 1
6.3
Area of plate = (24.5)
3
6.3 x 24.5
Area of plate =
3
154.35
Area of plate = = 51.45 m2
3

Answer = 51.45 m2

Extension of the First Rule

Consider the shape of a ship’s half water-plane area shown. The area has five ordinates.
One way of calculating the whole area would be to divide it up into two, use Simpson’s first
rule to calculate both areas 1 and 2 separately and then sum them up to work out the total.

AREA 1 AREA 2

h
h h h
Figure 2.9
A more convenient way is to consider the Simpson’s multipliers for both of the areas
concerned. Simply sum the multipliers for the ordinate that forms the boundary of both
shapes and treat the whole as a single shape. This gives multipliers of 14241.

Example 2

Calculate the area of a water-plane which has the following half-ordinates, spaced 12
meters apart: 1.7; 5.9; 7.0; 5.2; 1.3 meters.

We need to check if we can apply Simpson’s First Rule to calculate the area:

Number of ordinates = 5 and since 5 is odd number, we can use Simpson’s First Rule to
find the area of this water-plane.

Common interval “h” = 12 meters


Issue No. 1 Page 19 of 348
ST. THERESE- MTC COLLEGES SEAMANSHIP 6
Iloilo, Philippines (Advance Trim, Stability and Stress)
STUDENT LEARNING MODULE
Revision No. 1 Effectivity date: Reviewed by: Approved by:

10 August 2020 QMR President


The following figure will help us visually represent the information:

a=1.7m b= 5.9m c= 7.0m d= 5.2m


h h h e= 1.3m
h

12 m 12 m 12 m 12 m
Figure 2.10

1. Given: Common interval (h) = 12 meters


1st ordinate value (a) = 1.7 meters
2ndordinate value (b) = 5.9 meters
3rd ordinate value (c) = 7.0 meters
4th ordinate value (d) = 5.2 meters
5th ordinate value (e) = 1.3 meters
2. Formula:

h
Half-area of WP = (a + 4b + 2c + 4d + e)
3
3. Solution:
12
Half-area of WP = [1.7 + (4x5.9) + (2x 7.0) + (4 x 5.2) + 1.3]
3
12
Half-area of WP = [1.7 + 23.6 + 14 + 20.8 + 1.3]
3
12
Half-area of WP = (61.4)
3
Note that in the calculation we then
12 multiply the formula by 2 in order
Area of WP =2 x (61.4)
3 to obtain the entire water-plane
area, since the area calculated using
2 x 12 x 61.4
Area of WP = Simpson’s First Rule is for one half of
3 the ship’s water-plane area.
1473.6
Area of WP =
3

Area of WP= 491.2 m2

4. Answer: Area of WP= 491.2 m2


Another way of solving the problem:

b= 5.9m c= 7.0m d= 5.2


a=1.7m
h h h h e= 1.3m
Issue No. 1 Page 20 of 348
ST. THERESE- MTC COLLEGES SEAMANSHIP 6
Iloilo, Philippines (Advance Trim, Stability and Stress)
STUDENT LEARNING MODULE
Revision No. 1 Effectivity date: Reviewed by: Approved by:

10 August 2020 QMR President

12 m 12 m 12 m 12 m

1 4 1
+ 1 4 1
1 4 2 4 1 Simpson’s multipliers
for 5 ordinates

No. Half-
Ordinates Multipliers Product
a 1.7 1 1.7
b 5.9 4 23.6
c 7.0 2 14.0
d 5.2 4 20.8
e 1.3 1 1.3
(Sum) Σ1 61.4

1. Formula:
h
Half-area of WP = (Σ1)
3

2. Solution:
12
Half-area of WP = (61.4)
3
Note that in the calculation we then
12 multiply the formula by 2 in order
Area of WP = 2 x (61.4)
3 to obtain the entire water-plane
area, since the area calculated using
2 x 12 x 61.4 Simpson’s First Rule is for one half of
Area of WP =
3 the ship’s water-plane area.
1473.6
Area of WP =
3
3. Answer: Area of WP= 491.2 m2

Example 3

A ship 120 meters long at the water-plane has equidistantly spaced half-ordinates
commencing from forward as follows: 0, 3.7, 5.9, 7.6, 7.5, 4.6, 0.1 meters respectively.
Find the area of the water-plane.

Again, we need to check if we can apply Simpson’s First Rule to calculate the area:
Issue No. 1 Page 21 of 348
ST. THERESE- MTC COLLEGES SEAMANSHIP 6
Iloilo, Philippines (Advance Trim, Stability and Stress)
STUDENT LEARNING MODULE
Revision No. 1 Effectivity date: Reviewed by: Approved by:

10 August 2020 QMR President


Number of ordinates = 7 and since 7 is odd number, we can use Simpson’s First Rule to
find the area of this water-plane.
length of ship 120 m
Let us compute the Common interval “h”: h= = = 20 meters
6 6
The following figure will help us visually represent the information:

b=3.7 m c=5.9 m d=7.6m e=7.5 m f=4.6 m

a=0 m g=0.1 m
20 m 20 m 20 m 20 m 20 m 20 m
120 meters
Figure 2.11

1. Given: Common interval (h) = 20 meters


1st ordinate value (a) = 0 meters

2ndordinate value (b) = 3.7 meters


3rd ordinate value (c) = 5.9 meters
4th ordinate value (d) = 7.6 meters
5th ordinate value (e) = 7.5 meters
6th ordinate value (f) = 4.6 meters
7th ordinate value (g) = 0.1 meters

2. Formula:
h
Half-area of WP = [a + 4b + 2c + 4d + 2e + 4f + g]
3
3. Solution:
20
Half-area of WP = [0 + (4 x 3.7)+(2 x 5.9)+(4 x 7.6)+(2 x7.5)+( 4 x 4.6)+ 0.1]
3
20
Half-area of WP = [0 + 14.8 + 11.8 + 30.4 + 15.0+ 18.4 + 0.1]
3
20
Half-area of WP = (90.5)
3
20 Note that in the calculation we then
Area of WP = 2 x (90.5)
3 multiply the formula by 2 in order
to obtain the entire water-plane
2 x 20 x 90.5 area, since the area calculated using
Area of WP =
3 Simpson’s First Rule is for one half of
the ship’s water-plane area.
3620
Area of WP =
3
Area of WP = 1206.7 m2
Issue No. 1 Page 22 of 348
ST. THERESE- MTC COLLEGES SEAMANSHIP 6
Iloilo, Philippines (Advance Trim, Stability and Stress)
STUDENT LEARNING MODULE
Revision No. 1 Effectivity date: Reviewed by: Approved by:

10 August 2020 QMR President

4. Answer: Area of WP = 1206.7 m2

Another way of solving the problem:

b=3.7 m c=5.9 m d=7.6m e=7.5 m f=4.6 m

a=0 m g=0.1 m
20 m 20 m 20 m 20 m 20 m 20 m
120 meters
1 4 1
1 4 1
+1 4 1
1 4 2 4 2 4 1
Simpson’s multipliers for 7 ordinates

1. Given: Common interval (h) = 20 meters

1st ordinate value (a) = 0 meters


2ndordinate value (b) = 3.7 meters
3rd ordinate value (c) = 5.9 meters
4th ordinate value (d) = 7.6 meters
5th ordinate value (e) = 7.5 meters
6th ordinate value (f) = 4.6 meters
7th ordinate value (g) = 0.1 meters

No. Half-
Ordinates Multipliers Product
a 0 1 0
2. b 3.7 4 14.8 Formula:
c 5.9 2 11.8 Half-area of WP =
d 7.6 4 30.4 h (Σ1)
e 7.5 2 15.0 3
f 4.6 4 18.4
3. g 0.1 1 0.1 Solution:
(Total) Σ1 90.5 Half-area of WP =
20
(90.5)
3
20
Area of WP = 2 x (90.5)
3
2 x 20 x 90.5
Area of WP =
3
Issue No. 1 Page 23 of 348
ST. THERESE- MTC COLLEGES SEAMANSHIP 6
Iloilo, Philippines (Advance Trim, Stability and Stress)
STUDENT LEARNING MODULE
Revision No. 1 Effectivity date: Reviewed by: Approved by:

10 August 2020 QMR President


3620
Area of WP =
3

4. Answer: Area of WP = 1206.7 m2

Example 4:

A ship 70 meters long at the water-plane has equidistantly spaced half-ordinates


commencing from forward as follows: 0, 5.2, 6.4, 7.0, 6.0, 4.9, 0.3 meters respectively.
Find the area of the water-plane.

b=5.2 m c=6.4 m d=7.0m e=6.0 m f=4.9 m

a=0 m g=0.3 m
11.7 11.7 11.7 11.7 11.7 11.7
70 meters

Figure 2.12
Let us find the value of the common interval “h” which can be calculated by dividing the
length of the water-plane (70m) by the number of area sections (6).

Ship length 70
H=
6
= 6
= 11.7 m

1. Given:
1st ordinate value (a) = 0 meters
2ndordinate value (b) = 5.2 meters
3rd ordinate value (c) = 6.4 meters
4th ordinate value (d) = 7.0 meters
5th ordinate value (e) = 6.0 meters
6th ordinate value (f) = 4.9 meters
7th ordinate value (g) = 0.3 meters

2. Required: Area of the water-plane


3. Formula:
h
Half-area of WP = [a + 4b + 2c + 4d + 2e + 4f + g]
3
4. Solution:
11.7
Half-area of WP = [0+(4 x 5.2)+(2 x 6.4)+(4 x 7.0)+(2 x 6.0)+( 4 x 4.9)+
3
0.3]

11.7
Half-area of WP = [0 + 20.8 + 12.8 + 28 + 12+ 19.6 + 0.3]
3
Issue No. 1 Page 24 of 348
ST. THERESE- MTC COLLEGES SEAMANSHIP 6
Iloilo, Philippines (Advance Trim, Stability and Stress)
STUDENT LEARNING MODULE
Revision No. 1 Effectivity date: Reviewed by: Approved by:

10 August 2020 QMR President

11.7
Half-area of WP = (93.5)
3
11.7 Note that in the calculation we then
Area of WP = 2 x (93.5)
3 multiply the formula by 2 in order
to obtain the entire water-plane
2 x 11.7 x 93.5 area, since the area calculated using
Area of WP =
3 Simpson’s First Rule is for one half of
the ship’s water-plane area.
2187.9
Area of WP =
3
Area of WP = 729.3 m2

5. Answer: Area of WP = 1206.7 m2

You can check answer using the tabular method.

Example 5

Find the area of a collision bulkhead 12 meters high. The half-breadths at equal intervals
from top one 7.0m, 4.8m, 2.95m, 2.0m, 1.65m, 1.3m and 0 meters respectively.
7.0 m

4.8 m

2.95 m

12
h= =2 12 m 2.0 m
6
1.65 m

1.3 m

0
Figure 2.13

1. Formula:
h
Area = x Σ1 x 2
Half- 3
2. Solution: ordinate S. Multipliers Product
7.0 1 7
4.8 4 19.2
2.95 2 5.9
2.0 4 8
1.65 2 3.3
1.3 4 5.2
0 1 0
(Sum) Σ1 48.6
Issue No. 1 Page 25 of 348
ST. THERESE- MTC COLLEGES SEAMANSHIP 6
Iloilo, Philippines (Advance Trim, Stability and Stress)
STUDENT LEARNING MODULE
Revision No. 1 Effectivity date: Reviewed by: Approved by:

10 August 2020 QMR President

2
Area = x 48.6 x 2
3

2 x 48.6 x 2 194.4
Area = =
3 3

Area = 64.8 m2

Do this

Activity 1 – Simpson’s First Rule

Find the area of the water-plane of a barge with a length of 124 meters and equidistantly
spaced half-ordinates commencing from forward as follows: 3, 10, 16, 19.5, 21,19, 15.5, 10,
6 meters respectively.

Write your solutions and answer in the space provided.

Activity 2 – Simpson’s First Rule

Find the area of the water-plane of a barge with a length of 80 meters long and
equidistantly spaced half-ordinates commencing from forward as follows: 0, 3.6, 5.0, 5.3,
4.8 meters respectively.

Write your solutions and answer in the space provided.


Issue No. 1 Page 26 of 348
ST. THERESE- MTC COLLEGES SEAMANSHIP 6
Iloilo, Philippines (Advance Trim, Stability and Stress)
STUDENT LEARNING MODULE
Revision No. 1 Effectivity date: Reviewed by: Approved by:

10 August 2020 QMR President

Assessment

Lesson 2: Assessment 1 – Simpson’s First Rule

Calculate the area of a water-plane which is 72 meters long, using Simpson’s First Rule with
the following half-ordinates commencing from forward: 0.2, 2.2, 5.0, 5.8, 6.0, 5.9, 4.9, 2.0
and 0.2 respectively.

Write your solutions and answer in the space provided.


Issue No. 1 Page 27 of 348
ST. THERESE- MTC COLLEGES SEAMANSHIP 6
Iloilo, Philippines (Advance Trim, Stability and Stress)
STUDENT LEARNING MODULE
Revision No. 1 Effectivity date: Reviewed by: Approved by:

10 August 2020 QMR President

2. Simpson’s Second Rule


The area between any four consecutive ordinates is equal to the sum of the end ordinates,
plus three times each of the middle ordinates, all multiplied by three-eights of the common
interval.

This rule assumes that the equation of the curve is of the third order, i.e. of a curve whose
equation, referred to the coordinate axes. Consider the shape shown:

Figure 2.14

AREA = 3/8 x h x (1A + 3B + 3C + 1D)


Issue No. 1 Page 28 of 348
ST. THERESE- MTC COLLEGES SEAMANSHIP 6
Iloilo, Philippines (Advance Trim, Stability and Stress)
STUDENT LEARNING MODULE
Revision No. 1 Effectivity date: Reviewed by: Approved by:

10 August 2020 QMR President

Simpson’s multipliers are 1, 3, 3 and 1.

Process me
Example 1

A plate section has dimensions as shown. Calculate the area.


a. Illustration

1.8m 3.4m 4.4m 4.9m

4.6m 4.6m 4.6m


Figure 2.15

b. Given in the problem:


Common interval (h) = 4.6 meters
1st ordinate value (a) = 1.8 meters

2ndordinate value (b) = 3.4 meters


3rd ordinate value (c) = 4.4 meters
4th ordinate value (d) = 4.9 meters

c. Formula:
Area = 3/8 x h x (1a+ 3b + 3c + 1c)

d. Solution:
3
Area of plate= x 4.6 x [(1 x 1.8) + (3 x 3.4) + (3 x 4.4) + (1 x 4.9)]
8
3
Area of plate = x 4.6 (1.8 + 10.2 + 13.2 + 4.9)
8
3
Area of plate = x 4.6 (30.1)
8

3 x 4.6 x 30.1
Area of plate =
8
3 x 30.1
Area of plate =
8
Issue No. 1 Page 29 of 348
ST. THERESE- MTC COLLEGES SEAMANSHIP 6
Iloilo, Philippines (Advance Trim, Stability and Stress)
STUDENT LEARNING MODULE
Revision No. 1 Effectivity date: Reviewed by: Approved by:

10 August 2020 QMR President

415.38
Area of plate =
8

Area of plate = 51.92 m2


e. Answer: Area of plate = 51.92 m2

Solution (2):

Simpson’s multiplier:1 3 3 1

Using a table, calculate what is in the brackets.

No. Ordinates
Multipliers Product
A 1.8 1 1.8
B 3.4 3 10.2
C 4.4 3 13.2
D 4.9 1 4.9
(Sum) Σ2 30.1
Σ 2is used because it is a total
using Simpson’s Second Rule

3
Solution: Area = x h x Σ2
8
3
Area = x 4.6 x 30.1
8
3
Area = (138.46)
8
Issue No. 1 Page 30 of 348
ST. THERESE- MTC COLLEGES SEAMANSHIP 6
Iloilo, Philippines (Advance Trim, Stability and Stress)
STUDENT LEARNING MODULE
Revision No. 1 Effectivity date: Reviewed by: Approved by:

10 August 2020 QMR President


3 x 138.46 415.38
Area = = = 51.92 m2
8 8

Extension of the Second Rule

Consider in figure 2.16, the area between the half-ordinates t and w. if the common interval
(CI) is h, this area is equal to:
3
Area between t and w = = h (t + 3u + 3v + w)
8

t u v w x y
h h h h h h z
Figure 2.16

The area between w and z can be found in the same way and added to the above, to give
the total area of the half-waterplane.

3
Area between t and w = h (t + 3u + 3v + w)
8

3
Area between w and z = h (w + 3x + 3y + z)
8

3
Total area = h (t + 3u + 3v + 2w + 3x + 3y + z)
8

The conclusions we can draw for the rule are: -

a) The Rule can be used when, and only when, the number of ordinates is four, or four
plus some multiple of three (e.g. 4,7,10,13,16, etc.).
b) The area is found by multiplying successive ordinates, including the ends, by the
multipliers 1, 3, 3, 2, 3, 3, 2, -----1, adding the results together, and then multiplying
by three-eights of the common interval.

Note: Simpson’s second rule can be used whenever there is a number of ordinates such
that:
Number of ordinates−1
= a whole number
3
 With 4 ordinates the Simpson’s multipliers are: 1331
4−1
where: =1
3
 With 7 ordinates the Simpson’s multipliers are: 1332331
Issue No. 1 Page 31 of 348
ST. THERESE- MTC COLLEGES SEAMANSHIP 6
Iloilo, Philippines (Advance Trim, Stability and Stress)
STUDENT LEARNING MODULE
Revision No. 1 Effectivity date: Reviewed by: Approved by:

10 August 2020 QMR President


7−1
where: =2
3
 With 10 ordinates the Simpson’s multipliers are: 1332332331
10−1
where: =3
3
and so on……..
There may be instances when either rule may be used, such as with the previous example
having 7 ordinates.

Simpson’s first and second rules will be applied to the calculation of areas under the curve
of statical stability.

Process me
Let’s try an example to better understand how to use the formula:

Consider the shape of a ship’s half water-plane area shown. The area has seven ordinates.
The approach is the same as above.
The multipliers in this case are 1332331. Consider the example.

AREA 1 AREA 2

h h h h h h
Figure 2.17
1 3 3 1
1 3 3 1
SM 1 3 3 2 3 3 1

Example 2

A small boat has a half water-plane area with equally spaced half-ordinates as follows:
0.20m, 1.20m, 1.70m, 1.82m, 1.75m, 1.65m and 1.21m respectively. The half-ordinates are
equally spaced at 1.40m apart. Calculate the water-plane area.

Illustration:
Issue No. 1 Page 32 of 348
ST. THERESE- MTC COLLEGES SEAMANSHIP 6
Iloilo, Philippines (Advance Trim, Stability and Stress)
STUDENT LEARNING MODULE
Revision No. 1 Effectivity date: Reviewed by: Approved by:

10 August 2020 QMR President

1.82m
1.70m 1.75m
1.20m
1.65m
1.21m
0.20m

Figure 2.18
Given in the problem:
Common interval (h) = 1.40 meters
1st ordinate value (a) = 0.20 meters
2nd ordinate value (b) = 1.20 meters
3rd ordinate value (c) = 1.70 meters
4th ordinate value (d) = 1.82 meters
5th ordinate value (e) = 1.75 meters
6th ordinate value (f) = 1.65 meters
7th ordinate value (g) = 1.21 meters

3
Formula to use: Half WPA= x h (1a+ 3b + 3c + 2d + 3e + 3f + 1g)
8

Solution:
3
Half WPA = x 1.40[(1x0.20)+(3x1.20)+(3x1.70)+(2x1.82)+(3x1.75)+(3x1.65)+(1x1.21)]
8

3
Half WPA = x 1.40[0.20+3.60+5.10+3.64+5.25+4.95+1.21)
8
3 Note that in the calculation we then
Half WPA = x 1.40(23.95)
8 multiply the formula by 2 in order
to obtain the entire water-plane
3 x 1.40 x 23.95 100.59 area, since the area calculated using
Half WPA = = = 12.57375
8 8 Simpson’s Second Rule is for one half
Total WPA = 2 x 12.57375 of the ship’s water-plane area.

Total WPA =25.15 m2

No. Half- Solution (2):


Ordinates Multipliers Product
a 0.20 1 0.20 Using a table, calculate what
is b 1.20 3 3.60 in the brackets.
c 1.70 3 5.10
d 1.82 2 3.64
e 1.75 3 5.25
f 1.65 3 4.95
g 1.21 1 1.21
(Total) Σ2 23.95
Issue No. 1 Page 33 of 348
ST. THERESE- MTC COLLEGES SEAMANSHIP 6
Iloilo, Philippines (Advance Trim, Stability and Stress)
STUDENT LEARNING MODULE
Revision No. 1 Effectivity date: Reviewed by: Approved by:

10 August 2020 QMR President

Σ2is used because it is a total using Simpson’s Second Rule.


1. Formula:
3
Half-area of WP = x h (Σ2)
8

2. Solution:
3
Half-area of WP = x 1.40(23.95)
8
3
Area of WP = 2 x (33.53)
8
2 x 3 x 33.53
Area of WP =
8
201.18
Area of WP =
8

3. Answer: Area of WP = 25.15 m2

Do This

Activity 3 – Simpson’s Second Rule

Calculate the area of a water-plane, using Simpson’s Second Rule. The common interval is
15 meters and the ordinates are: 0, 9.0, 13.3, 14.7, 12.8, 7.5 and 0.6.

Write your solutions and answer in the space provided.


Issue No. 1 Page 34 of 348
ST. THERESE- MTC COLLEGES SEAMANSHIP 6
Iloilo, Philippines (Advance Trim, Stability and Stress)
STUDENT LEARNING MODULE
Revision No. 1 Effectivity date: Reviewed by: Approved by:

10 August 2020 QMR President

Activity 4 – Simpson’s Second Rule

A ship 100 meters long at the water-plane has equidistantly spaced as follows:
0, 3.3, 5.0, 7.5, 7.8, 7.6, 7.1, 5.9, 3.7, and 0.4 respectively. Calculate the area of the water-
plane.

Write your solutions and answer in the space provided.


Issue No. 1 Page 35 of 348
ST. THERESE- MTC COLLEGES SEAMANSHIP 6
Iloilo, Philippines (Advance Trim, Stability and Stress)
STUDENT LEARNING MODULE
Revision No. 1 Effectivity date: Reviewed by: Approved by:

10 August 2020 QMR President

Assessment

Assessment 2 – Simpson’s Second Rule

The lengths of half-ordinates of a 90 m long water-plane commencing from forward are as


follows:
0, 2.0, 3.6, 5.0, 5.5, 6.0, 4.8, 3.4, 2.0 and 0.5 respectively.

Calculated the area of water-plane.


Write your solutions and answer in the space provided.
Issue No. 1 Page 36 of 348
ST. THERESE- MTC COLLEGES SEAMANSHIP 6
Iloilo, Philippines (Advance Trim, Stability and Stress)
STUDENT LEARNING MODULE
Revision No. 1 Effectivity date: Reviewed by: Approved by:

10 August 2020 QMR President

3. The Five/Eight Rule (Simpson’s Third Rule)


This rule may be used to find the area between two consecutive ordinates when there three
consecutive ordinates are known (see Figure 2.19). We must know the length of one other
equally ordinate, next to the area which we wish to measure.

The rule is that the area is equal to five times one end ordinate, plus eight times the other,
minus the known external ordinate (in that order), all multiplied by one-twelfth (1/12)of the
common interval.

Suppose that we wish to find the area between a and b in Figure 2.19 c is the external
ordinate and in the common interval.
h 1
Thus: Area 1 = (5a + 8b – c) or x h x Σ3
12 12
Similarly, the area between b and c would be equal to:
h 1
Area 2 = (5c + 8b – a) or x h x Σ3
12 12
Σ3is used because it is a total, using Simpson’s Third Rule.
Issue No. 1 Page 37 of 348
ST. THERESE- MTC COLLEGES SEAMANSHIP 6
Iloilo, Philippines (Advance Trim, Stability and Stress)
STUDENT LEARNING MODULE
Revision No. 1 Effectivity date: Reviewed by: Approved by:

10 August 2020 QMR President

Figure 2.19

Process me
Example 1:

Three consecutive ordinates in a ship’s water-plane, spaced 6 meters apart, are 14, 15, and
15.5 meters respectively. Calculate the area between the last two ordinates.

Shaded
Area

Figure 2.20

Required: Area between the last two ordinates.

Given: a = 14 m b = 15 m c = 15.5 m h=6m


h
Shaded area = (5c + 8b – a)
12
6
= [(5x15.5) + (8x15) – 14]
12
6
= (77.5 + 120 – 14)
12
6
= (197.5 – 14)
12
6
= (183.5)
12
6 x 183.5
=
12
Issue No. 1 Page 38 of 348
ST. THERESE- MTC COLLEGES SEAMANSHIP 6
Iloilo, Philippines (Advance Trim, Stability and Stress)
STUDENT LEARNING MODULE
Revision No. 1 Effectivity date: Reviewed by: Approved by:

10 August 2020 QMR President


1101
=
12
Shaded area = 91.75 m2

Solution (2):

Using a table, calculate the shaded area.


No. Half-
Ordinates Multipliers Product
c 15.5 5 77.5
b 15 8 120.00
a 14 1 - 14.00
(Sum) Σ3 183.50
Σ3 is used because it is a total using Simpson’s Third Rule.
1. Formula:
1
Shaded-area = x h x Σ3
12
1
Shaded-area = x 6 x 183.5
12

1101
=
12

Shaded area = 91.75 m2

Example 2:

Three consecutive ordinates in a ship’s water-plane, spaced 5 meters apart, are 4.5, 5.5,
and

5m 5m

5.5m 6.5m
4.5m AREA?

Figure 2.21

Required: Area between the first two ordinates.


Issue No. 1 Page 39 of 348
ST. THERESE- MTC COLLEGES SEAMANSHIP 6
Iloilo, Philippines (Advance Trim, Stability and Stress)
STUDENT LEARNING MODULE
Revision No. 1 Effectivity date: Reviewed by: Approved by:

10 August 2020 QMR President


Given: a = 4.5 m b = 5.5 m c = 6.5 m h = 5m
h
Area = (5a + 8b – c)
12
5
= [(5 x 4.5) + (8 x 5.5) – 6.5]
12
5
= (22.5 + 44 – 6.5)
12
5
= (66.5 – 6.5)
12
5
= (60)
12
5 x 60
=
12
300
=
12
Area = 25 m2
Solution (2):

Using a table, calculate the area between the first two ordinates.

No. Half-
Ordinates Multipliers Product
a 4.5 5 22.5
b 5.5 8 44.0
c 6.5 1 - 6.5
(Sum) Σ3 60

Σ3 is used because it is a total using Simpson’s Third Rule.

2. Formula:
1
Area = x h x Σ3
12
1
= x 5 x 60
12

300
=
12
Area = 25 m2

Example 3:
Issue No. 1 Page 40 of 348
ST. THERESE- MTC COLLEGES SEAMANSHIP 6
Iloilo, Philippines (Advance Trim, Stability and Stress)
STUDENT LEARNING MODULE
Revision No. 1 Effectivity date: Reviewed by: Approved by:

10 August 2020 QMR President


Three consecutive half-ordinates in a ship’s water-plane, spaced 12 meters apart, are 6.0,
3.5

12m 12m

6.0m 3.5m

? 0.2m

Figure 2.22

Given: h= 12m

Required: area between the last two ordinates from forward.


h 1
Formulae: Half WP = (5c + 8b – a) or x h x Σ3
12 12
12
Half WP = [5(0.2) + 8(3.5) – 6.0]
12
12
Half WP = (1+28 – 6)
12

12
Area = (23) x 2
12
Area = 46 m2

Solution (2):

Using a table, calculate the area between the first two ordinates.

No. Half-
Ordinates Multipliers Product
c 0.2 5 1.00
b 3.5 8 28.00
a 6.0 1 - 6.00
(Sum) Σ3 23.00

Σ3 is used because it is a total using Simpson’s Third Rule.


Formula:
Issue No. 1 Page 41 of 348
ST. THERESE- MTC COLLEGES SEAMANSHIP 6
Iloilo, Philippines (Advance Trim, Stability and Stress)
STUDENT LEARNING MODULE
Revision No. 1 Effectivity date: Reviewed by: Approved by:

10 August 2020 QMR President


1
Half WP = x h x Σ3
12

1
Half WP = x 12 x 23
12

12
Area = (23) x 2
12
Area = 46 m2

Do This

Activity 5 – Simpson’s Third Rule

Find the area of a steel plate of the following shape:


Issue No. 1 Page 42 of 348
ST. THERESE- MTC COLLEGES SEAMANSHIP 6
Iloilo, Philippines (Advance Trim, Stability and Stress)
STUDENT LEARNING MODULE
Revision No. 1 Effectivity date: Reviewed by: Approved by:

10 August 2020 QMR President


The area of section Y is known to be 25 m2.

Write your solutions and answer in the space provided.

Assessment

Assessment 3 – Simpson’s Third Rule

Three consecutive ordinates in a ship’s water-plane area are 6.3 m, 3.35 m and 0.75 m
respectively. The common interval is 6.0 meters. Calculate the area contained between the
last two ordinates.

Write your solutions and answer in the space provided.


Issue No. 1 Page 43 of 348
ST. THERESE- MTC COLLEGES SEAMANSHIP 6
Iloilo, Philippines (Advance Trim, Stability and Stress)
STUDENT LEARNING MODULE
Revision No. 1 Effectivity date: Reviewed by: Approved by:

10 August 2020 QMR President

4. Subdivided common intervals/Half-intervals


Near the ends of a ship, where the curvature is great, we sometimes bring in intermediate
ordinates, halfway between the main ordinates, in order to obtain greater accuracy.

Consider Figure 2.23 which shows a water-plane with an intermediate ordinate midway
1
between a and b. the intervals ax and bx are each h.
2

a x b c d e f y g

1/2h 1/2h 1/2h 1/2h

Figure 2.23

1/2h
Area between a and b = (a + 4x =b)
3
Issue No. 1 Page 44 of 348
ST. THERESE- MTC COLLEGES SEAMANSHIP 6
Iloilo, Philippines (Advance Trim, Stability and Stress)
STUDENT LEARNING MODULE
Revision No. 1 Effectivity date: Reviewed by: Approved by:

10 August 2020 QMR President


h
=3 ( 1 1
2 a +2x + 2 b )
Area between b and f = 3
h
( b + 4c + 2d + 4e + f )
The area could be calculated separately and the results added together afterwards. Usually,
however, we combine the calculations by adding together the formula as shown below.
There are advantages to be gained by using this combined method of calculation when
finding centers of gravity of shapes.

Total Area= 3
h
( 12 1
a + 2x + 1 2 b+ 4c + 2d +4e +f )
If there are intermediate ordinates at the other end of the water-plane, they are treated in
the same way. For instance, it we had the ordinate y midway between f and g:

Total Area= 3
h
( 12 1
a + 2x + 1 2 b+ 4c + 2d +4e +
1
1
2 f + 2y +
1
2g )

Process me
Example 1:

A water-plane has ordinates, 20 meters apart, of 1.6, 13.6, 26.5, 25.1, 10.6, and 2.1
meters.

9.4 m
13.6m 26.5m 25.1m 10.6m 2.1m

1.6m h = 20 m

Figure 2.24
Issue No. 1 Page 45 of 348
ST. THERESE- MTC COLLEGES SEAMANSHIP 6
Iloilo, Philippines (Advance Trim, Stability and Stress)
STUDENT LEARNING MODULE
Revision No. 1 Effectivity date: Reviewed by: Approved by:

10 August 2020 QMR President

Given: Common Interval or h = 20 meters


Ordinates: a = 1.6 meters intermediate ordinate = 9.4 meters
b = 13.6 meters
c = 26.5 meters
d = 25.1 meters
e = 10.6 meters
f = 2.1 meters
Formula:

Total Area = 3
h
( 12 1
a + 2x + 1 2 b+ 4c + 2d +4e +f )
Total Area = 3
20
( 12 ( 1
1.6) + 2(9.4) + 1 2 (13.6)+ 4(26.5) + 2(25.1) +4(10.6) + 2.1 )
Total Area = 3
20
( 0.8 + 18.8 + 20.4+ 106 + 50.2 + 42.4 + 2.1 )
20
Total Area = (240.7)
3

20 x 240.7 4814
Total Area = =
3 3

Total Area = 1605 m2

Solution (2):
h
1. Formula: Total area = x Σ1
3
2. Solution:

Using a table, calculate the area between the first two ordinates.
No. Ordinates
Multipliers Product
a 1.6 ½ 0.8
x 9.4 2 18.8
b 13.6 1½ 20.4
c 26.5 4 106
d 25.1 2 50.2
e 10.6 4 42.4
f 2.1 1 2.1
(Sum) Σ1 240.7
Issue No. 1 Page 46 of 348
ST. THERESE- MTC COLLEGES SEAMANSHIP 6
Iloilo, Philippines (Advance Trim, Stability and Stress)
STUDENT LEARNING MODULE
Revision No. 1 Effectivity date: Reviewed by: Approved by:

10 August 2020 QMR President

h
Total area = x Σ1
3
20 x 240.7 4814
Total Area = = =1605 m2
3 3
Total Area = 1605 m2

Example 2: (continuation to example 1)


If there is also an intermediate ordinates at the other end of the water-plane, ordinate
midway

9.4 m 2.1m
13.6m 26.5m 25.1m 10.6m
1.6m 9.4 m 1.6m

Figure 2.25

Given: Common Interval or h = 20 meters


Ordinates: a = 1.6 meters
b = 13.6 meters
c = 26.5 meters
d = 25.1 meters
e = 10.6 meters
f = 2.1 meters

g = 1.6 meters
Intermediate ordinate midway between the first two = 9.4 meters
Intermediate ordinate midway between the f and g = 9.4 meters
Formula:

Total Area = 3
h
( 12 1
a + 2x + 1 2 b+ 4c + 2d +4e +
1
1
2 f + 2y +
1
2g )
3 [2 (
20 1 1 1
Total Area = 1.6) + 2(9.4) + 1 2 (13.6) + 4(26.5) + 2(25.1) +4(10.6) +1 2 (2.1) +
Issue No. 1 Page 47 of 348
ST. THERESE- MTC COLLEGES SEAMANSHIP 6
Iloilo, Philippines (Advance Trim, Stability and Stress)
STUDENT LEARNING MODULE
Revision No. 1 Effectivity date: Reviewed by: Approved by:

10 August 2020 QMR President


1
2(9.4) + 2 (1.6) ]
Total Area = 3
20
( 0.8 + 18.8 + 20.4+ 106 + 50.2 + 42.4 +3.15 + 18.8 + 0.8 )
3( )
20
Total Area = 261.35

20 x 261.35 5227
Total Area =
3
= 3
Total Area = 1742.3 m2

Solution (2):
1. Formula:
h
Total area = x Σ1
3
2. Solution:
Using a table, calculate the area between the first two ordinates.

No. Ordinates
Multipliers Product
a 1.6 ½ 0.8
x 9.4 2 18.8
b 13.6 1½ 20.4
c 26.5 4 106
d 25.1 2 50.2
e 10.6 4 42.4
f 2.1 1½ 3.15
y 9.4 2 18.8
g 1.6 ½ 0.3
(Sum) Σ1 261.35

h
Total area = x Σ1
3

20 x 261.35 5227
Total Area = = = 1742.3 m2
3 3
Issue No. 1 Page 48 of 348
ST. THERESE- MTC COLLEGES SEAMANSHIP 6
Iloilo, Philippines (Advance Trim, Stability and Stress)
STUDENT LEARNING MODULE
Revision No. 1 Effectivity date: Reviewed by: Approved by:

10 August 2020 QMR President

Do This

Activity 6 – Subdivided Common Intervals Problem


The length of a ship’s water-plane is 100 meters. The lengths of the half-ordinates
commencing from forward are as follows.0, 3.6, 6.0, 7.3, 7.7, 7.6, 4.8 and 0.6 meters
respectively.

Midway between the last two half-ordinates is one whose length is 2.8 meters. Calculate the
area of the water-plane.

Write your solutions and answer in the space provided.

5. Volumes of Ship Shapes and Similar Figures


Issue No. 1 Page 49 of 348
ST. THERESE- MTC COLLEGES SEAMANSHIP 6
Iloilo, Philippines (Advance Trim, Stability and Stress)
STUDENT LEARNING MODULE
Revision No. 1 Effectivity date: Reviewed by: Approved by:

10 August 2020 QMR President


The ship is divided up into a number of equally spaced sections, the area of each of which is
found by Simpson’s Rules. The volume is found by putting these through the rules in the
same way as ordinary ordinates. The sections may be eithervertical or horizontal, as
convenient.

(a) (b)

Figure 2.26

Figure 2.26a shows how the calculation may be done by using the vertical sections B, C, D,
E and F. The area of each section is found by the Rules in the ordinary way and the volume
can then be calculated by Simpson’s First Rule, thus:
h
Volume = (A+ 4B + 2C + 4D + 2E + 4F + G)
3

CI
or Volume = x Σ1
3

Figure 2.26b shows how the same volume could be found by using horizontal sections.
Where A is the deck area, G the area at the keel and B, C, D, E and F, the areas of
intermediate sections.

h
Volume = (A+ 4B + 2C + 4D + 2E + 4F + G)
3

CI
or Volume = x Σ1
3

Σ1 is used because it is a total using Simpson’s First Rule.


Thus the volume of displacement of a ship to any particular draft can be found first by
calculating the area of water-planes or transverse areas at equidistant intervals and then
using these areas as ordinates to find the volume by Simpson’s Rules.
Issue No. 1 Page 50 of 348
ST. THERESE- MTC COLLEGES SEAMANSHIP 6
Iloilo, Philippines (Advance Trim, Stability and Stress)
STUDENT LEARNING MODULE
Revision No. 1 Effectivity date: Reviewed by: Approved by:

10 August 2020 QMR President

Process me
Example 1:

The area of a ship’s water-planes are as follows:


Draft (m) 0 1 2 3 4
Area of WP (sq. m) 650 660 662 661 660

Calculate the ship’s underwater volume.

S. Multipliers

W L
A 660
1
661
B 4
662
Draft 4m C 2
660 CI or h = 1 m
D
4
650
E 1
Figure 2.27
Formula:
h
Underwater Volume = (1A+ 4B + 2C + 4D + 1E)
3
1
Underwater Volume = [660+ 4(661) + 2(662) + 4(660) + 650]
3
1
Underwater Volume = (660+ 2644 + 1324 + 2640 + 650)
3
1 7918
Underwater Volume = (7918) = = 2639.3 m3
3 3
Underwater Volume = 2639.3 m3

Solution (2):

1. Formula:
CI
Volume = x Σ1
3
Issue No. 1 Page 51 of 348
ST. THERESE- MTC COLLEGES SEAMANSHIP 6
Iloilo, Philippines (Advance Trim, Stability and Stress)
STUDENT LEARNING MODULE
Revision No. 1 Effectivity date: Reviewed by: Approved by:

10 August 2020 QMR President

2. Solution:
Using a table, calculate the area between the first two ordinates.
Draft Area of WP
(m2) S. Multipliers Volume
function
0 650 1 650
1 660 4 2640
2 662 2 1324
3 661 4 2644
4 660 1 660
(Sum) Σ1 7918
Σ1is used because it is a total using Simpson’s First Rule.
h
Underwater Volume = x Σ1
3
1
Underwater Volume = (7918)
3
7918
Underwater Volume = = 2639.3 m3
3
Underwater Volume = 2639.3 m3

Example2:

The ship is floating upright on an even keel at 6.0 m draft F and A. The areas of the ship’s
water-planes are as follows:
Draft (m) 0 1 2 3 4 5 6

Area of WP (sq. m) 5000 5600 6020 6025 6025 6025 6025

Calculate the ship’s underwater volume.


Formula:
h
Underwater Volume = (1A+ 4B + 2C + 4D + 2E + 4F + 1G)
3
S. Multipliers

W A 6025 1 L
B 6025 4
C 6025 2
D 6025 CI or h = 1 m 4
Draft 6m
E 6020 2
F 5600 4
G 5000 1

Figure 2.28
Issue No. 1 Page 52 of 348
ST. THERESE- MTC COLLEGES SEAMANSHIP 6
Iloilo, Philippines (Advance Trim, Stability and Stress)
STUDENT LEARNING MODULE
Revision No. 1 Effectivity date: Reviewed by: Approved by:

10 August 2020 QMR President

h
Underwater Volume = (1A+ 4B + 2C + 4D + 2E + 4F + 1G)
3
1
Underwater Volume = [6025+4(6025) + 2(6025)+ 4(6025)+ 2(6020)+ 4(5600)+ 5000]
3
1
Underwater Volume = (6025+ 24100 + 12050 + 24100 + 12040 +22400 + 5000)
3
1
Underwater Volume = (105,715)
3
105715
Underwater Volume=
3
Underwater Volume= 35,238.3 m3

Solution (2):

1. Formula:
CI
Volume = x Σ1
3
2. Solution:

Using a table, calculate the area between the first two ordinates.
Water- Area of WP
plane (m2) S. Multipliers Volume
function
A 6025 1 6025
B 6025 4 24100
C 6025 2 12050
D 6025 4 24100
E 6020 2 12040
F 5600 4 22400
G 5000 1 5000
(Sum) Σ1 105,715
Σ1is used because it is a total using Simpson’s First Rule.
h
Underwater Volume = x Σ1
3
1
Underwater Volume = (105,715)
3

105715
Underwater Volume = = 35,238.3 m3
3

Underwater Volume = 35,238.3 m3


Issue No. 1 Page 53 of 348
ST. THERESE- MTC COLLEGES SEAMANSHIP 6
Iloilo, Philippines (Advance Trim, Stability and Stress)
STUDENT LEARNING MODULE
Revision No. 1 Effectivity date: Reviewed by: Approved by:

10 August 2020 QMR President

Do This

Activity 7 – Volume Calculation


A ship is floating upright in S.W. on an even keel at 7 meters draft F and A. The Calculate
the underwater volume. The areas of the ship’s water-planes are as follows:
Draft (m) 1 2 3 4 5 6 7
Area of WP (sq. m) 60.1 60.4 60.6 60.6 60.6 60.6 60.6

The volume between the outer bottom and 1 meters draft is 3,060 m 3.
Calculate the ship’s total underwater volume.
Issue No. 1 Page 54 of 348
ST. THERESE- MTC COLLEGES SEAMANSHIP 6
Iloilo, Philippines (Advance Trim, Stability and Stress)
STUDENT LEARNING MODULE
Revision No. 1 Effectivity date: Reviewed by: Approved by:

10 August 2020 QMR President

Assessment

Assessment 4 – Volume Calculation

The following data gives the areas of the ship’s water-planes at various drafts:
Draft (m) 6 7 8
Area of WP (sq. m) 700 765 800

Calculate the volume of displacement.

6. Appendages and Intermediate Ordinates


Issue No. 1 Page 55 of 348
ST. THERESE- MTC COLLEGES SEAMANSHIP 6
Iloilo, Philippines (Advance Trim, Stability and Stress)
STUDENT LEARNING MODULE
Revision No. 1 Effectivity date: Reviewed by: Approved by:

10 August 2020 QMR President


It has been mentioned previously that areas and volumes calculated by the use of Simpson’s
Rules depend for their accuracy on the curvature of the sides following a definite
mathematical law. It is very seldom that ship’s sides do follow one such curve. Consider the
ship’s water-plane are shown in Figure 2.29. The sides from the stem to the quarter form
one curve but from this point to the stern is part of an entirely different curve. To obtain an
answer which is as accurate as possible, the area from the stem to the quarter may be
calculated by the use of Simpson’s Rules and then the remainder of the area may be found
by a second calculation. Often, the main area of the water-plane is calculated separately
from the bow or stern ends of the ship, which is referred to as appendages.

Figure 2.29
Similarly, in Figure 2.30 the side of the ship forms a reasonable curve from the waterline
down to the turn of the bilge, but below this point the curve is one of a different form.

In this case the volume of displacement between the waterline (WL) and the water-plane XY
could be found by use of Simpson’s Rules and then the volume of the appendage found by
means of a second calculation.

W L

X Y

Appendage

Figure 2.30
Issue No. 1 Page 56 of 348
ST. THERESE- MTC COLLEGES SEAMANSHIP 6
Iloilo, Philippines (Advance Trim, Stability and Stress)
STUDENT LEARNING MODULE
Revision No. 1 Effectivity date: Reviewed by: Approved by:

10 August 2020 QMR President

Process me
Example 1:

A ship’s breadth at 9 meters intervals commencing from forward are as follows:


0, 7.6, 8.7, 9.2, 9.5, 9.4 and 8.5 meters respectively.
Abaft the last ordinate is an appendage of 50 sq. m. Calculate the total area of the water-
plane. A2 = 50m2

g=8.5
a=0 b=7.6 c=8.7 d=9.2 e=9.5 f=9.4

AREA 2
AREA 1 Appendage

Figure 2.31
Given in the problem:
Appendage area (Area 2) =50 m2
Common interval (h) = 9 meters
1st ordinate value (a) = 0
2ndordinate value (b) = 7.6 meters
3rd ordinate value (c) = 8.7 meters
4th ordinate value (d) = 9.2 meters
5th ordinate value (e) = 9.5 meters
6th ordinate value (f) = 9.4 meters
7th ordinate value (g) = 8.5 meters

Required: Total Area of the water-plane

Formula to use:
h
Area of WP = (a + 4b + 2c + 4d + 2e + 4f + g)
3
Solution:
9
Area of WP = (0+4(7.6) +2(8.7) +4(9.2) +2(9.5) +4(9.4) +8.5)
3
9
Area of WP = (0+ 30.4 +17.4 +36.8 +19 + 37.6 + 8.5)
3
9
Area of WP = (149.7)
3
9 x 149.7
Area of WP =
3
Issue No. 1 Page 57 of 348
ST. THERESE- MTC COLLEGES SEAMANSHIP 6
Iloilo, Philippines (Advance Trim, Stability and Stress)
STUDENT LEARNING MODULE
Revision No. 1 Effectivity date: Reviewed by: Approved by:

10 August 2020 QMR President


1347.3
Area of WP =
3
Area of WP = 449.1 m2 - Area 1
Appendage = + 50.0 m2 - Area 2
Area of WP= 499.1 m2

Note: You can double check the answer using the tabular method.

Example 2:

A ship’s water-plane area has been divided into common intervals of length 9 meters with
the following half-ordinates commencing from forward are as follows:

0, 1.0, 2.0, 3.0, 2.0 meters respectively.

Aft of the last ordinate is an appendage of 25 sq. m. Find the total area of the water-plane.
A2 = 25m2

e=2.0
a=0 b=1.0 c=2.0 d=3.0

AREA 2
AREA 1 Appendage
Figure 2.32

Given in the problem:


Appendage area (Area 2) =25 m2
Common interval (h) = 9 meters
1st ordinate value (a) = 0
2ndordinate value (b) = 1.0 meters
3rd ordinate value (c) = 2.0 meters
4th ordinate value (d) = 3.0 meters
5th ordinate value (e) = 2.0 meters

Required: Total Area of the water-plane

Formula to use:
h
Half Area of WP = (a + 4b + 2c + 4d + 1e)
3

Solution:
Issue No. 1 Page 58 of 348
ST. THERESE- MTC COLLEGES SEAMANSHIP 6
Iloilo, Philippines (Advance Trim, Stability and Stress)
STUDENT LEARNING MODULE
Revision No. 1 Effectivity date: Reviewed by: Approved by:

10 August 2020 QMR President


9
Half-Area of WP = (0+4(1.0)+2(2.0)+4(3.0)+2.0)
3
9
Half-Area of WP = (0+ 4 +4 +12 + 2)
3

9
Half-Area of WP = (22)
3
9 x 22
Half-Area of WP =
3
198 x 2 396
Area of WP = =
3 3
Area of WP = 132 m2
Appendage = 25 m2
Area of WP= 157 m2

7. Area and volumes having an Awkward Number of Ordinates

Occasionally the number of ordinates used is such that an area or volume concerned cannot
be found directly by use of either First or Second Rule. In such cases the area or volume
should be divided into two parts, the area part being calculated separately and the total
area found by adding the areas of the two parts together.

The multipliers in this case are 1332331. Consider the example.

AREA 1 AREA 2

b c d e f
a
Figure 2.33
Show how the area of a water plane may be found when using six semi-ordinates. Neither
the First nor the Second Rule can be applied directly to the whole area but the water plane
can be divided into two parts as shown in Figure 2.33. Area No. 1 can be calculated using
the First and area No. 2 by the Second Rule. The areas of the two parts may then be added
together to find the total area.

AREA 1 AREA 2
Issue No. 1 Page 59 of 348
ST. THERESE- MTC COLLEGES SEAMANSHIP 6
Iloilo, Philippines (Advance Trim, Stability and Stress)
STUDENT LEARNING MODULE
Revision No. 1 Effectivity date: Reviewed by: Approved by:

10 August 2020 QMR President

a b c d e f g h

Figure 2.34
Show how the area may be found when using eight semi-ordinates.

Divide the area up as shown in Figure 2.34. Find area no. 1 using the Second Rule and area
no. 2 using the First Rule.

An alternative method is again to find the area between the half-ordinates a and g by the
first rule and the area between the half-ordinate g and h by the “Five-eight” Rule.

Process me
Example 1: Combination of First and Second Rules to calculate Area

Find the area of a transverse bulkhead 10 meters high. The half-breadths at equal intervals
from top one 10m, 9.3m, 8.3m, 7.1m, 5.7m, and 3.8 meters respectively.
10 m
h 9.3 m
Rule 1
h
8.3 m

h
10 m 7.1 m
h Rule 2
5.7 m
h
3.8 m
Figure 2.35

10
h= =2 for Area 1 = Rule 1
5

1. Formula:
h
Area = x Σ1 x 2
3

2. Solution:
Ordinate
S. Multipliers Product
10.0 1 10.0
9.3 4 37.2
8.3 1 8.3
(Sum) Σ1 55.5
Issue No. 1 Page 60 of 348
ST. THERESE- MTC COLLEGES SEAMANSHIP 6
Iloilo, Philippines (Advance Trim, Stability and Stress)
STUDENT LEARNING MODULE
Revision No. 1 Effectivity date: Reviewed by: Approved by:

10 August 2020 QMR President

h 2
a. Area 1 = x Σ1 x 2= x 55.5 x 2
3 3
111
Area 1 = x2
3
Area 1 = 37 x 2 = 74 m2

For Area 2 = Rule 2

Ordinate
S. Multipliers Product
8.3 1 8.3
7.1 3 21.3
5.7 3 17.1
3.8 1 3.8
(Sum) Σ2 50.5

10
h= =2
5
3h 3x 2
b. Area 2 = x Σ1 x 2= x 50.5 x 2
8 8
606
Area2 =
8
Area 2 = 75.75 m2

Total Area = Area 1 + Area 2


= 74 m2 + 75.75 m2

Total Area = 149.75 m2


Example 2
Find the area of a tank top 21 meters long when the equidistant breadths are as follow:
19.2 m, 18 m, 17.1 m, 16.2 m, 14.4 m, 12 m, 9.3 m, 6 m.
Issue No. 1 Page 61 of 348
ST. THERESE- MTC COLLEGES SEAMANSHIP 6
Iloilo, Philippines (Advance Trim, Stability and Stress)
STUDENT LEARNING MODULE
Revision No. 1 Effectivity date: Reviewed by: Approved by:

10 August 2020 QMR President

Figure 2.36

1. Using Rule 2:
Ordinate
S. Multipliers Product
19.2 1 19.2
18 3 54.0
17.1 3 51.3
16.2 1 16.2
(Sum) Σ1 140.7
21m
h= =3
7
3h 3x 3
Area1 = x Σ1 Area1= x 140.7
8 8
1266.3
Area 1 =
8
Area1 = 158.29 m2

2. Using Rule 1: Ordinate


S. Multipliers Product
16.2 1 16.2
14.4 4 57.6
12 2 24
9.3 4 37.2
6 1 6
(Sum) Σ2 141.0
h 3 423
Area2 = x Σ2 = x 141 = = 141
3 3 3
Total Area = Area 1 + Area 2
= 158.29 m2 + 141 m2
Total Area = 299.29 m2

Do This

Activity 8 – Combination of First and Second Rules to calculate Area

The lengths of the half-ordinates of a 60 m long water-plane commencing from forward are
as follows:
Issue No. 1 Page 62 of 348
ST. THERESE- MTC COLLEGES SEAMANSHIP 6
Iloilo, Philippines (Advance Trim, Stability and Stress)
STUDENT LEARNING MODULE
Revision No. 1 Effectivity date: Reviewed by: Approved by:

10 August 2020 QMR President


0, 4.2m, 5.8m, 6.0m, 3.5m, and 0.2m respectively.
Find the area of the water-plane.
Write your solutions and answer in the space provided.

Write your solutions and answer in the space provided.


Issue No. 1 Page 63 of 348
ST. THERESE- MTC COLLEGES SEAMANSHIP 6
Iloilo, Philippines (Advance Trim, Stability and Stress)
STUDENT LEARNING MODULE
Revision No. 1 Effectivity date: Reviewed by: Approved by:

10 August 2020 QMR President

Assessment

Assessment 5 – Combination of First and Second Rules to calculate Area

Find the area of part of a water-plane which has the following ordinates, spaced 6.0 meters
apart:
2.4 m, 3.7 m, 5.3 m, 6.6 m, 7.6 m, and 8.2 meters.
Write your solutions and answer in the space provided.
Issue No. 1 Page 64 of 348
ST. THERESE- MTC COLLEGES SEAMANSHIP 6
Iloilo, Philippines (Advance Trim, Stability and Stress)
STUDENT LEARNING MODULE
Revision No. 1 Effectivity date: Reviewed by: Approved by:

10 August 2020 QMR President

Lesson 3: Effects of Density

Learning Module 3.1: Effects of Density

Competence, Course Outcomes and Learning Outcomes

Competence:

Control trim, stability and stress

Course Outcome:

1. Explain the importance of maintaining stability during loading, unloading


Issue No. 1 Page 65 of 348
ST. THERESE- MTC COLLEGES SEAMANSHIP 6
Iloilo, Philippines (Advance Trim, Stability and Stress)
STUDENT LEARNING MODULE
Revision No. 1 Effectivity date: Reviewed by: Approved by:

10 August 2020 QMR President


and in-transit in various conditions.
2. Calculate the effect on trim and stability of a ship in the event of damage to
and consequent flooding of a compartment and countermeasures to be
taken.

Learning Outcomes:

At the end of the lesson, the student should be able to:

1. Discuss the significance of density in preserving the trim and stability relevant to:
– Displacement
– TPC
– Fresh Water Allowance (FWA)
2. Calculate the density in preserving the trim and stability relevant to:
– Displacement
– TPC
– Fresh Water Allowance (FWA)

Discussion

3.1 Displacement

Displacement of a ship (or any floating object) is defined as the number of tones of water it
displaces. It is usual to consider a ship displacing salt water of density 1.025t/m 3, however,
freshwater values of displacement (1.000t/m 3) are often quoted in ship’s hydrostatic data.

The volume of displacement is the underwater volume of a ship afloat i.e. the volume below
the waterline.

To calculate the displacement (W) of a ship the following needs to be known:


The volume of displacement (V)
The density of the water in which it floats ( ρ )
Issue No. 1 Page 66 of 348
ST. THERESE- MTC COLLEGES SEAMANSHIP 6
Iloilo, Philippines (Advance Trim, Stability and Stress)
STUDENT LEARNING MODULE
Revision No. 1 Effectivity date: Reviewed by: Approved by:

10 August 2020 QMR President


Since: Mass = Volume x Density

the mass, or displacement, of a ship is calculated by:


Displacement = Volume of Displacement x Water Density
i.e. W=Vxρ

Relative Density (RD)

The Relative Density (RD) of a substance is quoted instead of Density. This is simply a ration
of the density of the substance in question to that of Fresh Water.

The density of fresh water is 1.000t/m3.

Density of water in which ship typically floats

A ship is presumed to always float in water that lies in the following density range:

Fresh water (FW): 1.000t/m3 (RD 1.000) to

Salt water (SW): 1.025t/m3 (RD 1.025)

Water that lies between these two extremes is termed Dock Water (DW).

If the question states that a ship is floating in salt water (SW) then it can be always
assumed that the water density is 1.025t/m 3.

Similarly, if in fresh water (FW) then the density of 1.000t/m 3.

3.1.1 Effect of Change of Density when Displacement is Constant

When a ship moves from water of one density to another, without changing her mass, the
draft will change. This will happen because the ship must displace the same mass of water
in each case. Since the density of water has changed, the volume of water displaced must
also change. This can be seen from the formula:

Mass = Volume x Density

If the density of the water increases, then the volume of water displaced must decrease to
keep the mass of water displaced constant, and vice versa.

The Effect on Boxed Shaped Vessels

New mass of water displaced – Old mass of water displaced

Therefore: New volume x New Density = Old volume x Old density


Issue No. 1 Page 67 of 348
ST. THERESE- MTC COLLEGES SEAMANSHIP 6
Iloilo, Philippines (Advance Trim, Stability and Stress)
STUDENT LEARNING MODULE
Revision No. 1 Effectivity date: Reviewed by: Approved by:

10 August 2020 QMR President


New volume Old density
Old volume
= New density

But Volume = L X B X Draft

L x B x New draft Old density


Therefore:
L x B x Old draft
= New density

New draft Old density


or =
Old draft New density

Process me
Example 1:

A box shaped vessel floats at a mean draft of 2.50 meters, in dock water of density 1020
kg/m3. Find the mean draft for the same mass displacement in salt water 1025 kg/m 3.

New draft Old density


=
Old draft New density

Old density
New draft = x Old draft
New denity

1020
= x 2.50 meters
1025

New draft = 2.49 m

Example 2:

A box shaped vessel floats at a mean draft of 6.5 meters, in dock water of density 1018
kg/m3. Find the mean draft for the same mass displacement in salt water 1021 kg/m 3.

New draft Old density


=
Old draft New density

Old density
New draft = New denity x Old draft

1018
=
1021
x 6.5

New draft= 6.48 m


Issue No. 1 Page 68 of 348
ST. THERESE- MTC COLLEGES SEAMANSHIP 6
Iloilo, Philippines (Advance Trim, Stability and Stress)
STUDENT LEARNING MODULE
Revision No. 1 Effectivity date: Reviewed by: Approved by:

10 August 2020 QMR President


3.1.2 The Effect on Ship-Shaped Vessels
It has already been shown that when the density of the water in which a vessel floats is
changed the draft will change, but the mass of water in kg or tonnes displaced will be
unchanged, i.e.
New Displacement = Old Displacement
or New volume x New density = Old volume x Old density
therefore
New volume Old density
Old volume
= New density

3.1.3 Effect of Density on Displacement when the Draft is Constant

Should the density of the water in which a ship floats be changed without the ship altering
her draft, then the mass of water displaced must have changed. The change in the mass of
water displaced may have been brought about by bunkers and stores being loaded or
consumed during a sea passage, or by cargo being loaded or discharged. In all cases:

New volume of water displayed = Old volume of water displaced


or New volume x New density = Old volume x Old density
therefore:
New displacement Old displacement
New density
= Old density
New displacement New density
or =
Old displacement Old density

Process me
Example 1

A ship displaces 7500 tonnes while floating in fresh water. Calculate the displacement of the
ship when floating at the same draft in water of density 1015 kg/m 3, i.e. 1.015 t/m3.

New displacement New density


Old displacement
= Old density
New density
New displacement = Old displacement x
Old density
1015
New displacement = 7500 x
1000
New displacement = 7,612.5 tonnes
Issue No. 1 Page 69 of 348
ST. THERESE- MTC COLLEGES SEAMANSHIP 6
Iloilo, Philippines (Advance Trim, Stability and Stress)
STUDENT LEARNING MODULE
Revision No. 1 Effectivity date: Reviewed by: Approved by:

10 August 2020 QMR President


Example 2

A ship displaces 7500 tonnes while floating in fresh water. Calculate the displacement of the
ship when floating at the same draft in water of density 1015 kg/m 3, i.e. 1.015 t/m3.

New displacement New density


=
Old displacement Old density
New density
New displacement = Old displacement x
Old density
1015
New displacement = 7500 x
1000
New displacement = 7,612.5 tonnes

Example 3

A ship of 6550 tonnes displacement is floating in salt water. The ship has to proceed to a
berth where the density of the water is 1009 kg/ m3. Calculate how much cargo must be
discharged if she is to remain at the salt water draft.

a. Calculate the New Displacement


New density
New displacement = Old displacement x
Old density
1009
New displacement = 6550 x
1025

New displacement = 6447.75 tonnes

b. Calculate the Cargo to discharge


Old displacement = 6550.0 tonnes
New displacement = - 6447.8 tonnes
Cargo to discharge = 102.2 tonnes

Example 4

A ship of 125 m x 15 m x 12 m has a block coefficient 0.800 and is floating at the load
Summer draft of 7.3 meters in fresh water. Calculate how much more cargo can be loaded
to remain at the same draft in salt water.

a. Calculate the Old Displacement


Issue No. 1 Page 70 of 348
ST. THERESE- MTC COLLEGES SEAMANSHIP 6
Iloilo, Philippines (Advance Trim, Stability and Stress)
STUDENT LEARNING MODULE
Revision No. 1 Effectivity date: Reviewed by: Approved by:

10 August 2020 QMR President


Old displacement = L x B x Draft x C b x Density
= 125 m x 15 m x 7.3 m x 0.8 x 1.000 tonnes
Old displacement = 10,950 tonnes

b. Calculate the New Displacement


New density
New displacement = Old displacement x
Old density
1025
New displacement = 10,950 x
1000
New displacement = 11,223.75 tonnes

c. Calculate the Cargo to discharge


Old displacement = 10,950.00tonnes
New displacement = ~11,223.75 tonnes

Cargo to discharge = 273.75 tonnes

Do this

Activity 1 – Effect of Change of Density when Displacement is Constant

A box shaped vessel floats at a mean draft of 3.50 meters, in dock water of density 1022
kg/m3. Find the mean draft for the same mass displacement in salt water 1025 kg/m 3.

Write your solutions and answer in the space provided.


Issue No. 1 Page 71 of 348
ST. THERESE- MTC COLLEGES SEAMANSHIP 6
Iloilo, Philippines (Advance Trim, Stability and Stress)
STUDENT LEARNING MODULE
Revision No. 1 Effectivity date: Reviewed by: Approved by:

10 August 2020 QMR President

Activity 2 – Effect of Density on Displacement when the Draft is Constant

A ship of 7000 tonnes displacement is floating in salt water. The ship has to proceed to a
berth where the density of the water is 1011 kg/ m3. Calculate how much cargo must be
discharged if she is to remain at the salt water draft.

Write your solutions and answer in the space provided.


Issue No. 1 Page 72 of 348
ST. THERESE- MTC COLLEGES SEAMANSHIP 6
Iloilo, Philippines (Advance Trim, Stability and Stress)
STUDENT LEARNING MODULE
Revision No. 1 Effectivity date: Reviewed by: Approved by:

10 August 2020 QMR President

3.2 TPC (Tonnes Per Centimeter Immersion)

A ship must always displace her own weight of water. If a weight is added to her, it will
cause her sink until she displaces an extra layer of water of equal weight.

“TPC” is defined as the number of tons loaded or discharged in order to change ship’s draft
by one (1) centimeter in salt water.

Consider the ship shown floating in salt water (RD 1.025) with a water-plane area (WPA) at
the waterline.

W L
Issue No. 1 Page 73 of 348
ST. THERESE- MTC COLLEGES SEAMANSHIP 6
Iloilo, Philippines (Advance Trim, Stability and Stress)
STUDENT LEARNING MODULE
Revision No. 1 Effectivity date: Reviewed by: Approved by:

10 August 2020 QMR President

Water-plane Area (WPA) (m2)

Figure 3.1

A weight of 40 tonnes is loaded on deck so that the mean draught increases by 1 cm.

1 cm

Figure 3.2

Since the ship’s displacement is equal to the mass of water displaced (Law of Flotation) it
follows that the mass of the additional ‘slice’ of displaced water is equal to the added weight
of 40 tonnes.

In this instance, 40 tonnes represents the value of the Tonnes per Centimeter Immersion
(TPC) for the ship at the initial draught before the weight was loaded.

3.2.1 TPC Formula

Consider the previous situation, since: Mass = Volume x Density

then: Mass of additional slice of water = Volume of the additional slice of water x Density
if the WPA is assured to not significantly change between the two waterlines,
then: Volume of the slice = WPA (m2) x 1 cm

1 cm
Water-plane Area (m2)

Figure 3.3
Issue No. 1 Page 74 of 348
ST. THERESE- MTC COLLEGES SEAMANSHIP 6
Iloilo, Philippines (Advance Trim, Stability and Stress)
STUDENT LEARNING MODULE
Revision No. 1 Effectivity date: Reviewed by: Approved by:

10 August 2020 QMR President


2
We cannot multiply m by centimeters, therefore:
1
Volume of slice = WPA (m2) x (m)
100
1
therefore: Added displacement (t) = WPA (m2) x (m) X density (t/m3);
100
therefore, the formula for TPC is:

WPA WPA WPA


TPC = Xρ for salt water: TPCSW = X ρ SW =
100 100 97.56

where: WPA -Water-plane area in square meter


ρ - density of water (1.000t/m 3 for Fresh water and 1,025t/m3 for Salt water)

The area of the waterplane of a box-shaped vessel is the same for all drafts if the trim is
constant and so the TPC will also be the same for all drafts and therefore the TPC will
reduce at lower drafts. The TPCs are calculated for a range of drafts extending beyond the
light and loaded drafts and these are then tabulated or plotted on a graph. From the table
or graph the TPC at intermediate drafts may be found.

Process me
Example 1

A ship floats at a water-plane of area 1,520 m 2. What is her TPC in water of density
1.020t/m3.

WPA
Formula: TPC = Xρ
100
2
1520 m
Solution: TPC = X 1.020 t/m3
100

TPC = 15.5 tonnes

Example 2

Calculate the TPC for a ship with a water-plane area of 1600 m 2 when it is floating in:
a. Fresh water
b. Dock water of R.D. 1.006
c. Salt water

Formula:
Issue No. 1 Page 75 of 348
ST. THERESE- MTC COLLEGES SEAMANSHIP 6
Iloilo, Philippines (Advance Trim, Stability and Stress)
STUDENT LEARNING MODULE
Revision No. 1 Effectivity date: Reviewed by: Approved by:

10 August 2020 QMR President


WPA
TPC = Xρ
100
Solution:
WPA 1600
a. TPC = Xρ= X 1.000 = 16.00 t
100 100

WPA 1600
b. TPC = X ρ= X 1.006 = 16.096 t
100 100

WPA 1600
c. TPC = X ρ= X 1.025 = 16.40 t
100 100

3.2.2 Factors affecting TPC


WPA
Consideration of the TPC formula: TPC = Xρ
100
shows that:
 TPC increases with WPA and for a normal ship-shape the WPA will increase with
draught.
 TPC increases with density

Two values of TPC are often quoted in ship’s hydrostatic data, TPC SW and TPCFW. However,
hydrostatic data for the example vessel is given for salt water only.

Consider the extract hydrostatic particulars (Table 1).

If a ship were floating at a draught of 5.00 m in salt water (RD 1.025) the displacement of
the ship would be 15,120 tonnes. To sink the ship by exactly 1 cm, 31.96 tonnes would
have to be loaded.

Consider the situation if the ship were to float at the same draught of 5.00 m but in fresh
water (RD 1.000).

Would the displacement and TPC values be the same as they were in salt water?

Consider the following diagrams showing the ship floating at the same draught but in
different water densities.

Salt water RD 1.025t/m3


Issue No. 1 Page 76 of 348
ST. THERESE- MTC COLLEGES SEAMANSHIP 6
Iloilo, Philippines (Advance Trim, Stability and Stress)
STUDENT LEARNING MODULE
Revision No. 1 Effectivity date: Reviewed by: Approved by:

10 August 2020 QMR President

Fresh water RD 1.000 t/m3

Figure 3.4

Displacement = Volume of Displacement x Water Density


For both situations the volume of displacement is the same.

It follows that the displacement of the ship when at a draught of 5.00 m in salt water must
be greater than the displacement of the ship when at the same draught in fresh water
(since salt water is denser than the fresh water).

Consider now the TPC value for both situations.

Salt water RD 1.025 t/m3


Figure 3.5a

Fresh water RD 1.000 t/m3

Figure 3.5b

By similar reasoning a 1 cm slice of salt water will have a greater mass than a 1 cm slice of
fresh water. Therefore, for the same draught of 5.00 m the TPC in salt water will be greater
than the TPC in fresh water.
Issue No. 1 Page 77 of 348
ST. THERESE- MTC COLLEGES SEAMANSHIP 6
Iloilo, Philippines (Advance Trim, Stability and Stress)
STUDENT LEARNING MODULE
Revision No. 1 Effectivity date: Reviewed by: Approved by:

10 August 2020 QMR President


Determination of the equivalent fresh water values of displacement and TPC for fresh water
is very simple.

The extract hydrostatic data in table 1 gives the following saltwater values for a draught of
4.90 m:

Displacement : 14,800 tonnes


TPC : 31.93 tonnes

Displacement in fresh
water (RD 1.000)
1.000
= 14,800 x =
1.025
14,439 t

TPC in fresh water (RD


1.000)

Table 1: Extract Hydrostatic Table

Process me
Example 1

Using the extract hydrostatic particulars (table 1) determine the displacement and TPC
values for the ship when floating at a draught of 6.30 m in:
a. salt water (RD 1.025)
b. fresh water (RD 1.000)
c. dock water (RD 1.012)
Issue No. 1 Page 78 of 348
ST. THERESE- MTC COLLEGES SEAMANSHIP 6
Iloilo, Philippines (Advance Trim, Stability and Stress)
STUDENT LEARNING MODULE
Revision No. 1 Effectivity date: Reviewed by: Approved by:

10 August 2020 QMR President

Table 1: Extract Hydrostatic Particular Table


Solution:

a. The hydrostatic data (Table 1) gives the following salt water values for a draught of
6.40 m.
Displacement = 19,640 tonnes TPC = 32.51 tonnes

1.000
b. Displacement in fresh water (RD 1.000) = 19640 x = 19,160.97 tonnes
1.025
1.000
TPC in fresh water (RD 1.000) = 32.51 x = 31.71 tonnes
1.025
1.012
c. Displacement in dock water (RD 1.012) = 19640 x = 19,390.91 tonnes
1.025
1.012
TPC in dock water (RD 1.012) = 32.51 x = 32.10 tonnes
1.025

Example 2

Using the extract hydrostatic particulars (table 1) determine the displacement and TPC
values for the ship when floating at a draught of 4.80 m in:
a. salt water (RD 1.020)
b. fresh water (RD 1.000)
c. dock water (RD 1.010)

Solution:

a. The hydrostatic data (Table 1) gives the following salt water values for a draught of
4.80 m.
Displacement = 14,480 tonnes TPC =31.90 tonnes
Issue No. 1 Page 79 of 348
ST. THERESE- MTC COLLEGES SEAMANSHIP 6
Iloilo, Philippines (Advance Trim, Stability and Stress)
STUDENT LEARNING MODULE
Revision No. 1 Effectivity date: Reviewed by: Approved by:

10 August 2020 QMR President


1.000
b. Displacement in fresh water (RD 1.000) = 14,480x = 14,196.07 tonnes
1.020
1.000
TPC in fresh water (RD 1.000) = 31.90x = 31.27 tonnes
1.020
1.010
c. Displacement in dock water (RD 1.010) = 14,480x = 14,338.04 tonnes
1.020
1.010
TPC in dock water (RD 1.010) = 31.90x = 31.59 tonnes
1.020

With ship shapes this formula should not be simplified further as it was in the case of a box
shape because the underwater volume is not rectangular. To find the change in draft of a
ship shape due to change of density a quantity known as the “Fresh Water Allowance” must
be known.

Do this

Activity 3 – Displacement and TPC Problems


Using the extract hydrostatic particulars (table 1) determine the displacement and TPC
values for the ship when floating at a draught of 5.60 meters in:
a. salt water (RD 1.025)
b. fresh water (RD 1.000)
c. dock water (RD 1.009)

Table 1: Extract Hydrostatic Particular Table


Write your solutions and answer in the space provided.
Issue No. 1 Page 80 of 348
ST. THERESE- MTC COLLEGES SEAMANSHIP 6
Iloilo, Philippines (Advance Trim, Stability and Stress)
STUDENT LEARNING MODULE
Revision No. 1 Effectivity date: Reviewed by: Approved by:

10 August 2020 QMR President

3.2.3 Loading/Discharge Problems

If given a TPC value for a particular draught, then the change in draught that will occur as a
result of loading or discharging weights, bodily sinkage or rise, may be calculated using:

w
Sinkage/Rise (cms) =
TPC

where w represents the total weight that is loaded or discharged. Having calculated the
sinkage/rise of the ship, this is then applied to the initial draught.

Use of the formula above may be also used to determine the weight to load or discharge to
achieve a required draught where:

w = Sinkage/Rise x TPC

Process me
Example 1

MV Prestige has an initial mean draft of 4.60 m in salt water and is required to complete
loading with a draught of 6.90m. Using the extract hydrostatic Particulars (Table 1A)
calculate the amount of cargo that must be loaded.
Issue No. 1 Page 81 of 348
ST. THERESE- MTC COLLEGES SEAMANSHIP 6
Iloilo, Philippines (Advance Trim, Stability and Stress)
STUDENT LEARNING MODULE
Revision No. 1 Effectivity date: Reviewed by: Approved by:

10 August 2020 QMR President


Given: Initial mean draft = 4.60 m
Loading draught = 6.90 m

Required: Amount of cargo that must be loaded


Procedure 1:

1. Read off the Displacement salt water values for both the initial and require
final draught;
2. Subtract the smaller from the larger;
3. Result equals the amount to load.

Solution:

Initial draught of 4.60 m Displacement SW = 13,840 t


Require draught 6.90 m Displacement SW = 21,270 t
Cargo to load = 7,430 t

Procedure 2:

1. Read off the TPC salt water (TPC sw) values for both the initial and require
final draught;
2. Calculate the mean TPCSW value;
3. Calculate the required change in draught;
4. Use the formula:
w
Sinkage/Rise (cms) =
TPC

Solution:

Initial draught of 4.60 m TPCSW = 31.84 t


Require draught 6.90 m TPCSW = 32.76 t

Mean TPCsw = 31.84 t + 32.76 t (+ sign due to the case is sinkage)


2
Mean TPCSW = 64.6 t= 32.3 t
2
Sinkage (cms) = Required draught – Initial draught
= 6.90 m – 4.60 m
= 2.30 m convert to cms
Sinkage (cms) = 230 cms

Calculate cargo to load (w):


Issue No. 1 Page 82 of 348
ST. THERESE- MTC COLLEGES SEAMANSHIP 6
Iloilo, Philippines (Advance Trim, Stability and Stress)
STUDENT LEARNING MODULE
Revision No. 1 Effectivity date: Reviewed by: Approved by:

10 August 2020 QMR President


w
Sinkage/Rise (cms) =
TPC SW

w = Sinkage x Mean TPCSW

w = 230 cms x 32.3 t

w = 7,429 tonnes – cargo to load

Note that the answer differ slightly because of two reasons:

1. In using the mean value of TPC it is assumed that the TPC value changes linearly
between the ranges of draughts concerned. This is not so, as the underwater form of
a ship does not (usually) change uniformly with draught.
2. Displacement values taken from the hydrostatic data in using procedure 1 will be
rounded to the nearest whole tonne.
Issue No. 1 Page 83 of 348
ST. THERESE- MTC COLLEGES SEAMANSHIP 6
Iloilo, Philippines (Advance Trim, Stability and Stress)
STUDENT LEARNING MODULE
Revision No. 1 Effectivity date: Reviewed by: Approved by:

10 August 2020 QMR President

Table 1A: Extract Hydrostatic Particular Table


Issue No. 1 Page 84 of 348
ST. THERESE- MTC COLLEGES SEAMANSHIP 6
Iloilo, Philippines (Advance Trim, Stability and Stress)
STUDENT LEARNING MODULE
Revision No. 1 Effectivity date: Reviewed by: Approved by:

10 August 2020 QMR President

Example 2 Table 1B: Extract Hydrostatic Particular Table

MV Prestige has an initial mean draft of 6.80 m in salt water and loads 11,900 t of cargo.
Using the extract hydrostatic Particulars (Table 1A & 1B) calculate the final displacement
and mean draught.

Given: Initial mean draft = 6.80 m


Cargo loaded = 11,900 tonnes

Required: Final displacement and mean draught.

Solution:

Method 1

Initial draught 6.80 m Displacement SW = 20,940 t


Cargo loaded = 11,900 t

Final Displacement = 32,840 t

Enter data with final displacement gives a final mean draught of 10.30m.

Method 2

Initial draught 6.80 m ---------Displacement SW = 20,940 t---------- TPCSW = 32.71 t


Cargo loaded = 11,900 t

Final Displacement = 32,840 t ------- TPCSW = 35.24 t


Issue No. 1 Page 85 of 348
ST. THERESE- MTC COLLEGES SEAMANSHIP 6
Iloilo, Philippines (Advance Trim, Stability and Stress)
STUDENT LEARNING MODULE
Revision No. 1 Effectivity date: Reviewed by: Approved by:

10 August 2020 QMR President


Mean TPC sw = 32.71 t + 35.24 t

2
Mean TPCSW = 67.95 t = 33.975 t
2

w 11,900t
Sinkage/Rise (cms) = = = 350.3 cms = 3.50 m
TPC 33.975t /cms

Initial draught 6.80 m


Sinkage + 3.50 m

Final Draught =10.30 m

Example 3

MV Prestige arrives in port with a mean draught of 6.30 m in dock water RD 1.017. How
much cargo may be loaded to ensure that the maximum draught on completion is 6.70 m in
the dock water? Using the extract hydrostatic Particulars (Table 1A & 1B)

Given: Initial draught = 6.30 m


R.D. Dock water= 1.017 t/m3
Final draught on completion= 6.70 m

Required: Cargo to load

Solution:

Method 1

a. Initial draught 6.30 m------------------Displacement SW = 19,310 t


RD DW
Displacement DW = Displacement SW x
RD SW

1.017
Displacement DW = 19,310 t x = 19,159 t
1.025

b. Final draught 6.70 m -------------------Displacement SW = 20,610 t


RD DW
Displacement DW = Displacement SW x
RD SW
Issue No. 1 Page 86 of 348
ST. THERESE- MTC COLLEGES SEAMANSHIP 6
Iloilo, Philippines (Advance Trim, Stability and Stress)
STUDENT LEARNING MODULE
Revision No. 1 Effectivity date: Reviewed by: Approved by:

10 August 2020 QMR President


1.017
Displacement DW = 20,610 t x = 20,449 t
1.025

c. Initial Displacement DW = 19,159 t


Final Displacement DW = 20,449 t

Cargo to load = 1,290 t

Method 2

a. Initial draught 6.30 m ---------------------------TPCSW = 32.46 t


RD DW
TPC DW = TPC SW x
RD SW

1.017
TPC DW = 32.46 t x = 32.21 t
1.025

b. Final draught 6.70 m ---------------------------TPCSW = 32.66 t


RD DW
TPCDW = TPCSW x
RD SW

1.017
TPC DW = 32.66 t x = 32.41 t
1.025

c. Mean TPC sw = 32.21 t + 32.41 t


2
Mean TPC SW = 64.62 t = 32.31 t
2

d. Calculate Sinkage:

Sinkage (cms) = Required draught – Initial draught


= 6.70 m – 6.30 m
= 0.40 m convert to cms
Sinkage (cms) = 40 cms

e. Calculate cargo to load


w
Sinkage(cms) =
Mean TPC DW

w = Sinkage x Mean TPCDW


Issue No. 1 Page 87 of 348
ST. THERESE- MTC COLLEGES SEAMANSHIP 6
Iloilo, Philippines (Advance Trim, Stability and Stress)
STUDENT LEARNING MODULE
Revision No. 1 Effectivity date: Reviewed by: Approved by:

10 August 2020 QMR President


w = 40 cms x 32.31 t

w = 1,292.4 tonnes - cargo to load

Do this

Activity 4 – Loading Problems – Using Extract Hydrostatic Particulars


MV Alvar has an initial mean draft of 6.50 m in salt water and loads 10,430 t of cargo. Using
the extract hydrostatic Particulars (Table 1A & 1B) calculate the final displacement and
mean draught.

Write your solution s and answer in the space provided.


Issue No. 1 Page 88 of 348
ST. THERESE- MTC COLLEGES SEAMANSHIP 6
Iloilo, Philippines (Advance Trim, Stability and Stress)
STUDENT LEARNING MODULE
Revision No. 1 Effectivity date: Reviewed by: Approved by:

10 August 2020 QMR President

3.3 Fresh Water Allowance (FWA)

Is defined as the number of millimeters that increases or decreases in ship’s mean draft
when the ship sails from salt water to fresh water and vice versa while floating at the
loaded draft.

The main purpose of FWA is to deduce the distance by which the appropriate load line
maybe submerged in dock water, so that on entering sea water, the ship will regain her
correct freeboard.
Displacement(¿ tonnes)
Formula: FWA (mm) =
4 X TPC

1000 kg/m3
or 1.025 t/m3
FWA

1025 kg/m3
or 1.025 t/m3
Issue No. 1 Page 89 of 348
ST. THERESE- MTC COLLEGES SEAMANSHIP 6
Iloilo, Philippines (Advance Trim, Stability and Stress)
STUDENT LEARNING MODULE
Revision No. 1 Effectivity date: Reviewed by: Approved by:

10 August 2020 QMR President

Figure 3.6 Load line Marks

3.3.1 Derivation of the Fresh Water Allowance (FWA) Formula

The displacement of a ship may be calculated by:


Displacement SHIP = L x B x d x CB x ρ

SW FW 1.000t/m3
1.025t/m3
Figure 3.7 Summer draft floating

Consider the ship in Figure 3.7 showing loaded to the Summer draught floating in salt water
and fresh water.

The displacement will be different but the volumes of displacement will be the same. If a
ship is floating at a draught in FW, to find the equivalent SW displacement for the same
draught:
Displacement SW = Displacement FW x 1.025

∴ for the ship shown:

Displacement FW x 1.025 = Displacement FW + (WPA x FWA x 1.000t/m3)

where FWA is expressed in meters.

Displacement FW x 1.025 = Displacement FW + (WPA x FWA)

1.025 Displacement FW = Displacement FW + (WPA x FWA)

1.025 Displacement FW – Displacement FW = (WPA x FWA)

0.025 Displacement FW = (WPA x FWA) ---------------------------------- (1)

Consider now the formula for TPC, where the TPC SW is the weight required to sink the ship
while at the summer displacement by 1 cm.

WPA
TPCSW = 100 x 1.025

Rearranging the formula:


Issue No. 1 Page 90 of 348
ST. THERESE- MTC COLLEGES SEAMANSHIP 6
Iloilo, Philippines (Advance Trim, Stability and Stress)
STUDENT LEARNING MODULE
Revision No. 1 Effectivity date: Reviewed by: Approved by:

10 August 2020 QMR President


TPC SW x 100
WPA = -------------------------------------------------------- (2)
1.025

Where the WPA is that for the Summer load draught waterline.

Substituting equation (2) into equation (1):

TPC SW x 100
0.025 Displacement FW = x FWA
1.025

0.025 x Displ FW x 1.025


FWA (mm) =
100 x TPC SW

to express FWA in mm then:

FWA 0.025 x Displ FW x 1.025


=
1000 100 x TPC SW

FWA 0.025 x Displ SW x1000


Rearranging gives: =
1000 100 x TPC SW

0.25 x DisplSW
therefore: FWA (mm) =
TPC SW

Displacement Summer
thus: FWA (mm) =
4 TPC SW

Process me
Example 1

A ship floats in salt water at the Summer displacement of 1690 tonnes. If the TPC SW is 5.18,
how much will the draught change by if the ship is towed to a berth where the density of
the water is 1.000 t/m3?

Solution:
In moving from SW to FW the ship will experience sinkage by an amount equal to the FWA.

Displacement(¿ tonnes)
FWA (mm) =
4 X TPC
Issue No. 1 Page 91 of 348
ST. THERESE- MTC COLLEGES SEAMANSHIP 6
Iloilo, Philippines (Advance Trim, Stability and Stress)
STUDENT LEARNING MODULE
Revision No. 1 Effectivity date: Reviewed by: Approved by:

10 August 2020 QMR President


1690 1690
FWA= = = 81.56 mm
4 X 5.18 20.72

∴ The draught will increase by 81.56 mm…….

3.3.3 Dock Water Allowance (DWA)

Ship is often load in dock water which is brackish, that is, as having a density of more than
1000 and less than 1025 kg/m3. When the ship is loading in dock water which is of a density
between these two limits “S” maybe submerged such as distance that she will automatically
rise to “S” when the open sea and salt water is reached. The distance by which “S” can be
submerged, and salt water is reached. The distance by which “S” can be submerged, called
the “Dock Water Allowance”.

If the ship were to go from SW to dock water of RD 1.010, the draught would change by the
DWA. The amount of the DWA is simply a fraction of the FWA as shown, in this case 3/5ths
or 15/25ths of the FWA value.

F 1.000 (FW)

05
T
10

FWA 15
DWA
20
S 1.025 (SW)

Figure 3.8

The DWA, as a fraction of the FWA, is found by the formula:

FWA (1025−ρ DW )
DWA =
25
Note: the draft of a ship will be grater in fresh water than in salt water because fresh water
is less dense.

 Fresh Water to Sea Water DRAFT Decreases (Rise)


 Sea Water to Fresh Water DRAFT Increases (Sink)
Issue No. 1 Page 92 of 348
ST. THERESE- MTC COLLEGES SEAMANSHIP 6
Iloilo, Philippines (Advance Trim, Stability and Stress)
STUDENT LEARNING MODULE
Revision No. 1 Effectivity date: Reviewed by: Approved by:

10 August 2020 QMR President

Process me
Example 1

A ship is loading in dock water of density 1010 t/m 3. FWA is 150 mm. Calculate the change
in draft on entering salt water.
F
1000 t/m3

FWA 1.010 t/m3

1.025 t/m3
S

Figure 3.9

Let x = the change in draft in millimeters

x 1025−1010
Then =
FWA 25
15
DWA (x) = 150 x
25
2250
DWA (x) = 25 = 90 mm

Answer: Draft will decrease by 90 mm, i.e. 9 cm

Example 2

A ship floats at a draft of 5.50m in dock water of density 1012 t/m 3. FWA is 165 mm.
Calculate the change in draft on entering salt water.

F
1000 t/m3

FWA 1.012 t/m3


x

1.025 t/m3
S
Figure 3.10

Let x = the change in draft in millimeters


Issue No. 1 Page 93 of 348
ST. THERESE- MTC COLLEGES SEAMANSHIP 6
Iloilo, Philippines (Advance Trim, Stability and Stress)
STUDENT LEARNING MODULE
Revision No. 1 Effectivity date: Reviewed by: Approved by:

10 August 2020 QMR President


FWA (1025−ρ DW )
DWA =
25
165(1025−1012)
DWA =
25
165(13) 2145
DWA = =
25 25
DWA = 85.8 mm

Draft will decrease by 86 mm (convert to meters)

= 0.086 m = 0.09 meter

New draft = Old draft ± DWA

New draft = 5.50m – 0.09 m

New draft = 5.41 m

Example 3

A ship floats at a draft of 7.50m in water of density 1025 t/m 3. What would be her draft in
water of density 1005 t/m3 if her FWA is 180 mm.
1000 t/m3
F

1.005 t/m3
FWA
x

1.025 t/m3
S
Figure 3.11
Let x = the change in draft in millimeters

FWA (1025−ρ DW )
DWA =
25
180(1025−1005) 180(20) 3600
DWA = = =
25 25 25
180(20) 3600
DWA = =
25 25
DWA = 144 mm

Draft will decrease by 144 mm (convert to meters)


= 0.144m
New draft = Old draft ± DWA
Issue No. 1 Page 94 of 348
ST. THERESE- MTC COLLEGES SEAMANSHIP 6
Iloilo, Philippines (Advance Trim, Stability and Stress)
STUDENT LEARNING MODULE
Revision No. 1 Effectivity date: Reviewed by: Approved by:

10 August 2020 QMR President


= 7.50m+ 0.144 m
New draft = 7.644 m

Example 4

A ship floats at a draft of 6.50m in dock water of density 1010 t/m 3. FWA is 160 mm.
Calculate the change in draft on entering salt water.
1000 t/m3
F

FWA 1.010 t/m3


x
S
1.025 t/m3
Figure 3.12

Let x = the change in draft in millimeters

FWA (1025−ρ DW )
DWA =
25
160(1025−1010)
DWA =
25
160(15) 2400
DWA = =
25 25
DWA = 96 mm

Draft will decrease by96 mm (convert to meters)


= 0.096 m = 0.10 meter

New draft = Old draft ± DWA


= 6.50m – 0.10 m
New draft = 6.60 m

Do this

Activity 5 – Dock Water Allowance (DWA)

A ship floats at a draft of 6.75 m in dock water of density 1012 t/m 3. FWA is 155 mm.
Calculate the change in draft when she passes from dock water to salt water.

Write your solutions and answer in the space provided.


Issue No. 1 Page 95 of 348
ST. THERESE- MTC COLLEGES SEAMANSHIP 6
Iloilo, Philippines (Advance Trim, Stability and Stress)
STUDENT LEARNING MODULE
Revision No. 1 Effectivity date: Reviewed by: Approved by:

10 August 2020 QMR President

Assessment

Assessment 1 – Effect of Density on Displacement when the Draft is Constant

A ship of 130 m x 20 m x 15 m has a block coefficient 0.900 and is floating at the load
Summer draft of 8.5 meters in fresh water. Calculate how much more cargo can be loaded
to remain at the same draft in salt water.

a. Calculate the Old Displacement


b. Calculate the New Displacement
c. Calculate the Cargo to discharge
Write your solutions and answer in the space provided.
Issue No. 1 Page 96 of 348
ST. THERESE- MTC COLLEGES SEAMANSHIP 6
Iloilo, Philippines (Advance Trim, Stability and Stress)
STUDENT LEARNING MODULE
Revision No. 1 Effectivity date: Reviewed by: Approved by:

10 August 2020 QMR President

Assessment

Assessment 2 – Effect of Density on Displacement when the Draft is Constant

A ship of 17,350 tonnes displacement is floating in salt water. The ship has to proceed to a
berth where the density of the water is 1010 kg/ m3. Calculate the new displacement and how
much cargo must be discharged if she is to remain at the salt water draft.
Write your solutions and answer in the space provided.
Issue No. 1 Page 97 of 348
ST. THERESE- MTC COLLEGES SEAMANSHIP 6
Iloilo, Philippines (Advance Trim, Stability and Stress)
STUDENT LEARNING MODULE
Revision No. 1 Effectivity date: Reviewed by: Approved by:

10 August 2020 QMR President

Assessment

Assessment 3 – Displacement and TPC Problems

Using the extract hydrostatic particulars determine the displacement and TPC values for the
ship when floating at a draught of 6.20 meters in:
a. salt water (RD 1.025)
b. fresh water (RD 1.000)
c. dock water (RD 1.008)
Write your solutions and answer in the space provided.
Issue No. 1 Page 98 of 348
ST. THERESE- MTC COLLEGES SEAMANSHIP 6
Iloilo, Philippines (Advance Trim, Stability and Stress)
STUDENT LEARNING MODULE
Revision No. 1 Effectivity date: Reviewed by: Approved by:

10 August 2020 QMR President

Assessment

Assessment 4 – Loading Problems – Using Extract Hydrostatic Particulars

MV Esperanza Ace arrives in port with a mean draught of 8.40 m in dock water RD 1.013.
How much cargo may be loaded to ensure that the maximum draught on completion is 8.90
m in the dock water? (Use Method 1 in this problem).

Required:
a. Displacement at 8.40 meters draft
b. Initial displacement dock water density 1.013
c. Displacement at 8.90 meters draft
d. Final displacement dock water density 1.013
e. Cargo to load
Write your solutions and answer in the space provided.
Issue No. 1 Page 99 of 348
ST. THERESE- MTC COLLEGES SEAMANSHIP 6
Iloilo, Philippines (Advance Trim, Stability and Stress)
STUDENT LEARNING MODULE
Revision No. 1 Effectivity date: Reviewed by: Approved by:

10 August 2020 QMR President

Lesson 4: Free Surface Effect

Learning Module 4.1: Free Surface Effect

Competence, Course Outcomes and Learning Outcomes

Competence:

Control trim, stability and stress

Course Outcome:

1. Explain the importance of maintaining stability during loading, unloading


and in-transit in various conditions.
2. Calculate the effect on trim and stability of a ship in the event of damage
to and consequent flooding of a compartment and countermeasures to be
taken.

Learning Outcomes:

At the end of the lesson, the student should be able to:

1. Calculate the free surface of liquids on stability and virtual reduction in GM for liquids
with a free surface in rectangular spaces and with divisional bulkhead.

Overview
Issue No. 1 Page 100 of 348
ST. THERESE- MTC COLLEGES SEAMANSHIP 6
Iloilo, Philippines (Advance Trim, Stability and Stress)
STUDENT LEARNING MODULE
Revision No. 1 Effectivity date: Reviewed by: Approved by:

10 August 2020 QMR President


Most cases of instability in ships are the result of free surface effects. This occurs when
tanks within the ship are only partially full or slack. When a ship heels, liquid within a
partially filled tank will move to the low side. It will be seen in this section that this adversely
affects the transverse statical stability of a ship.

Discussion

4.1 Free Surface

When a tank is completely filled with a liquid, the liquid cannot move within the tank when
the ship heels. For this reason, as far as stability is concerned, the liquid may be considered
a static weight having its center of gravity at the center of gravity of the liquid within the
tank.

Definition

Condition existing when a liquid is free to move in the tank or compartment of a vessel.
Causes a virtual rise of the ship’s center of gravity.

4.2 Free Surface Effect and the loss of Transverse Statical Stability

What is free surface effect and what is its impact on the ship’s stability when the ship is
listed?

Filled Compartment

G G
B Z

B1
g1
Issue No. 1 Page 101 of 348
ST. THERESE- MTC COLLEGES SEAMANSHIP 6
Iloilo, Philippines (Advance Trim, Stability and Stress)
STUDENT LEARNING MODULE
Revision No. 1 Effectivity date: Reviewed by: Approved by:

10 August 2020 QMR President


Figure 4.1

If a tank is completely filled with liquid, the liquid cannot move and acts a static weight. So
the center of gravity of the ship remains unchanged. The graphic shows (figure 4.1) the
listed ship when the tank is filled with liquid.

Partially Filled Compartment

Assume that the ship floats at the same draft with the same KG and with the tank partially
filled (figure 4.2a).

Metacentric
height

a b
Figure 4.2

Now (figure b), if the ship lists, the liquid flows to the lower side causing G of the liquid to
shift to G1.

This results in the center of gravity of the ship G moving to G 1. This effect is called a free
surface effect.

As a result, the vessel suffers an apparent loss of GM which is equal to GG V and hence the
metacentric height.

GZ, the righting lever is also reduced by an amount which is equal to GG 1. This virtual loss
of GM can be calculated. It is called a Free Surface Correction or FSC.

In order to indicate whether FSC has been applied or not the GM before subtracting FSC is
called Solid GM.
Issue No. 1 Page 102 of 348
ST. THERESE- MTC COLLEGES SEAMANSHIP 6
Iloilo, Philippines (Advance Trim, Stability and Stress)
STUDENT LEARNING MODULE
Revision No. 1 Effectivity date: Reviewed by: Approved by:

10 August 2020 QMR President


The GM attained after applying FSC is called a effective or fluid GM.

Thus: GM is termed the solid GM;


GVM is termed the effective or fluid GM.

When calculating the GM of a ship it is important that the effects of free surfaces in slack
tanks are considered. The loss of GZ will be greater as the number of slack tanks increases,
i.e. the cumulative effect of all slack tanks must be accounted for.

Liquid GVM

Solid GM

Figure 4.3

It is always the fluid GM that must be determined to take account of the reduction in GZ
values that arises from liquid movement within the ship as it is heeled.

When the ship returns to the upright condition the center of gravity (G 1) will move back to
its original position at G as the liquid in the tank finds its own level.

Remember G does not actually move up to GV.

Points to remember:

 If the area of the free surface is constant, the weight of liquid in the tank does not
affect the FSE.
 Within the ship, the position of the tank does not affect the FSE.
 When a tank is either empty or full, FSE is zero.
Issue No. 1 Page 103 of 348
ST. THERESE- MTC COLLEGES SEAMANSHIP 6
Iloilo, Philippines (Advance Trim, Stability and Stress)
STUDENT LEARNING MODULE
Revision No. 1 Effectivity date: Reviewed by: Approved by:

10 August 2020 QMR President


 To reduce the total FSE for the ship, the number of slack tanks must be kept to a
minimum since the FSE of each slack tank contributes to the total FSE for the ship.
 By fitting longitudinal divisions in the tank equally spaced, the FSE can be reduced to
1/n2 times the undivided value, n = number of spaces. Example 3 spaces are
produced by two longitudinal divisions and the FSE reduces to 1/9 th value of
undivided tank.
 FSE will make the situation worse before the bottom weight increases to a sufficient
level to bring G down, if time is taken to decide and should be filled first, if the ship
is at an angle of loll.

4.3 Calculating the Effect of Free Surface in a Rectangular Shaped Tank


For a tank has a rectangular free surface the virtual rise of G in meters can be calculated by:
l b3 dt
GGV = x
12V ds

where: GGV is the virtual rise of G in meters;


l is the tank length;
b is the tank breadth;
dt is the density of the liquid in the tank;
ds is the density of the water in which the ship floats (1.025t/m 3);
and V is the volume of displacement of the ship.

Since: Displacement = Volume of Displacement x Density

i.e. W = V x ds

l b3
it follows that: GGV = x dt
12W

Process me
Example 1:

A ship has an initial displacement of 10,600 t and KG 7.60. A rectangular cargo oil tank of
length 30 m and breadth 20 m is partially filled with 9650 t of oil (RD 0.86). If the KG of the
oil is 8.00 m. Calculate the effective GM if the KM for the final displacement is 8.80 m.

Solution:

a. Taking the moments about the keel, calculate the new solid KG.
Issue No. 1 Page 104 of 348
ST. THERESE- MTC COLLEGES SEAMANSHIP 6
Iloilo, Philippines (Advance Trim, Stability and Stress)
STUDENT LEARNING MODULE
Revision No. 1 Effectivity date: Reviewed by: Approved by:

10 August 2020 QMR President


Weight (t) KG (m) Moments (t-m)
(W x KG)
Initial displacement 10,600 7.60 80,560
Cargo oil 9,650 8.00 77,200
Final 20,250 t 7.791m 157,760 t-m

Total moments 157,760t−m


or Final KG= =
Total weights 20,250 t

Final KG= 7.791 meters

b. Calculate the effect of free surface


l b3 30 x 203
GGV = x dt = x 0.86
12W 12 x 20,250

30 x 8000 x 0.86 206,400


GGV = =
243,000 243,000

GGV = 0.849 m

c. Calculate the solid GM and then apply the free surface correction to obtain the fluid GM.

KM 8.800 m
Solid KG ~ 7.791 m
Solid GM 1.009 m
FSE (GGV) ~ 0.849 m
Fluid GM 0.160 m

Do this

Activity 1 – Calculating the Effect of Free Surface in a Rectangular Shaped


Tank

A ship has an initial displacement of 6,676 t and KG 9.00. A rectangular cargo oil tank of
length 20 m and breadth 10 m is partially filled with 200 t of oil (RD 0.89). If the KG of the
oil is 0.786 m. Calculate the effective GM if the KM for the final displacement is 9.20 m.

Write your solution and answer in the space provided.


Issue No. 1 Page 105 of 348
ST. THERESE- MTC COLLEGES SEAMANSHIP 6
Iloilo, Philippines (Advance Trim, Stability and Stress)
STUDENT LEARNING MODULE
Revision No. 1 Effectivity date: Reviewed by: Approved by:

10 August 2020 QMR President

Write your solution and answer in the space provided.


Issue No. 1 Page 106 of 348
ST. THERESE- MTC COLLEGES SEAMANSHIP 6
Iloilo, Philippines (Advance Trim, Stability and Stress)
STUDENT LEARNING MODULE
Revision No. 1 Effectivity date: Reviewed by: Approved by:

10 August 2020 QMR President

4.4 Free Surface Effect when tanks are filled or emptied


When liquid is run into, or removed from a tank so as to cause free surface, it will affect the
ship’s stability in two ways:-
a)
b)
c)
d)
e)
f)
g)
h)
i)
j)
k)
l)
m)
n)
o)
p)
q)
r)
s)
t)
u)
v)
w)
x)
y)
z)
aa)
bb)
Issue No. 1 Page 107 of 348
ST. THERESE- MTC COLLEGES SEAMANSHIP 6
Iloilo, Philippines (Advance Trim, Stability and Stress)
STUDENT LEARNING MODULE
Revision No. 1 Effectivity date: Reviewed by: Approved by:

10 August 2020 QMR President


cc)
dd) The solid KG and displacement will be changed by the weight added or removed
ee) There will be a rise of G caused by the free surface. This will have to be added to the
new solid KG to give the fluid KG.

Example 1:

A ship displaces 5,154 t, floats in sea water and has a KG of 6.10 m when a double bottom
tank is empty. The tank is rectangular, 15 m long, 12 m wide and 1.30 m deep. What would
be the ship’s GM after fresh water has been run into the tank to a depth of 0.80 m, if the
KM is then 7.45 m.
Solution:

Added volume = 15m x 12 x 0.80 = 144 m3 (144 t of fresh water)


Weight (t) KG (m) Moments (t-m)
(W x KG)
Initial displacement 5,154 t 6.10 31,439
Fresh water 144 t 0.40 58
Final 5,298 t 5.95 m 31,497 t-m

1. Taking the moments about the keel, calculate the new solid KG.
Total moments 31,497 t−m
Solid/Final KG= =
Total weights 5,298 t

Solid/Final KG=5.95 meters

2. Calculate the effect of free surface

l b3 15 x 123
GGV = x dt = x 1.000
12W 12 x 5298

15 x 1728 x 1 25,920
GGV = = = 0.41 m
63,576 63,576

3. Calculate the solid GM and then apply the free surface correction to obtain the fluid
GM.
KM 7.45 m
Solid KG ~ 5.95 m
Solid GM 1.50 m
FSE (GGV) ~ 0.41 m
Fluid GM 1.09 m

Do this
Issue No. 1 Page 108 of 348
ST. THERESE- MTC COLLEGES SEAMANSHIP 6
Iloilo, Philippines (Advance Trim, Stability and Stress)
STUDENT LEARNING MODULE
Revision No. 1 Effectivity date: Reviewed by: Approved by:

10 August 2020 QMR President


Activity 2 – Free Surface Effect when tanks are filled or emptied

A ship displaces 10,400 t, floats in sea water and has a KG of 6.45 m when a double bottom
tank is empty. The tank is rectangular, 35 m long, 16 m wide and 1.60 m deep. What would
be the ship’s GM after fresh water has been run into the tank to a depth of 0.90 m, if the
KM is then 7.95 m.

Write your solution and answer in the space provided.


Issue No. 1 Page 109 of 348
ST. THERESE- MTC COLLEGES SEAMANSHIP 6
Iloilo, Philippines (Advance Trim, Stability and Stress)
STUDENT LEARNING MODULE
Revision No. 1 Effectivity date: Reviewed by: Approved by:

10 August 2020 QMR President

4.5 Free Surface in divided tanks

1
W b L
2
1
b
2

Figure 4.4

Figure 4.4 shows a tank having a rectangular free surface and divided at the centerline.

Let l be the length and b the breadth of the tank. The breadth of the free surface on either
side of the tank will thus 1 b.
2
l b3
Rise of G due to free surface on one side = x
12V 8

GGv is due to the free surface on two sides however, so:


l b3
GGv = 2 x 12W x 8

2 l b3
= 8 x
12W
3
1 lb
GGv = 4 x
12W
If we compare this with the formula found in the last section, we shall see that it is one
quarter of the latter. So, in the case of a rectangular free surface, dividing the tank at the
centerline will decrease the rise of G to one quarter of what it would be in an individual
tank. In the case of surfaces other than rectangular ones, the decrease may not be the
same, but it will be considerable.
Issue No. 1 Page 110 of 348
ST. THERESE- MTC COLLEGES SEAMANSHIP 6
Iloilo, Philippines (Advance Trim, Stability and Stress)
STUDENT LEARNING MODULE
Revision No. 1 Effectivity date: Reviewed by: Approved by:

10 August 2020 QMR President


It can be proved, in the same way that dividing the tank into three would decrease the rise
of G due to free surface to one-ninth of that for an undivided tank. Dividing the tank into
four parts will decrease the free surface effect to one-sixteenth (1/16) of the above.

If free surface exists in more than one tank, this will cause a total decrease in the ship’s GM
equal to the sum of the effects for each tank.

Process me
Example 1:

A ship is floating in salt water and displaces 6,000 t, has a solid KG of 5.810 m and KM of
6.424 m. Free surface exists in a rectangular bunker tank, 3.0 m long and 12.0 m wide,
divided at the center line and containing oil of density 0.890 t/m 3. What is the ship’s fluid
GM?

a. Calculate the effect of free surface

1 l b3
GGV = x x dt
4 12W

1 3 x 123
GGV = x x 0.890
4 12 x 6000
1 3 x (1728)
GGV = x x 0.890
4 72000
1 5184
GGV = x x 0.890
4 72000

1 5184 x 0.890
GGV = x
4 72000

1 4613.76
GGV = x
4 72000

4613.76
GGV = = 0.016 m
288,000

b. Calculate the solid GM and then apply the free surface correction to obtain the fluid GM.

KM 6.424 m
Solid KG ~ 5.810 m
Solid GM 0.614m
FSE (GGV) ~ 0.016 m
Issue No. 1 Page 111 of 348
ST. THERESE- MTC COLLEGES SEAMANSHIP 6
Iloilo, Philippines (Advance Trim, Stability and Stress)
STUDENT LEARNING MODULE
Revision No. 1 Effectivity date: Reviewed by: Approved by:

10 August 2020 QMR President


Fluid GM 0.598 m

Example 2:

A ship is floating in salt water and displaces 8,440 t, has a KG of 6.21 m and KM of 7.21 m,
when a double bottom tank is full of fresh water. The tank is rectangular 18 m long, 20 m
wide, 1.5 meters deep and is divided into 4 equal tanks by fore and aft watertight divisions.
Calculate the ship’s GM after 360 t of fresh water have been pumped out, leaving it slack.

Displacement 8440 t

Full tank fresh water

Figure 4.5 Tank full

Depth of water 360 t of FW pumped out


removed 1 m
c.g 0.5 m

Figure 4.6 Tank pumped out FW

Solution:
360
Depth of water removed = = 1.00 m
18 x 20
Center of gravity (c.g.) water removed =0.5 m from top tank

KG of water removed = tank height – c.g. of water removed

= 1.5 m – 0.5 m = 1.00 meter

Weight Height (KG) Moment


Ship displacement 8.440 t 6.21 52,412 t-m
Water discharged ~ 360 t 1.0 ~ 360 t- m
Final 8,080t 6.442 m 52,052 t-m

a. Calculate the Solid KG:

Total moments 52,052t−m


Solid/Final KG= =
Total weights 8,080 t
Issue No. 1 Page 112 of 348
ST. THERESE- MTC COLLEGES SEAMANSHIP 6
Iloilo, Philippines (Advance Trim, Stability and Stress)
STUDENT LEARNING MODULE
Revision No. 1 Effectivity date: Reviewed by: Approved by:

10 August 2020 QMR President


Solid/Final KG= 6.442 meters

The surface is divided into 4 parts.

So, calculate the effect of free surface effect:

1 l b3
GGV = x x dt
16 12W
Fresh water density (d)
3
1 18 x 20
GGV = x x 1.000
16 12 x 8,080
1 18 x 8,000
GGV = x
16 96,960
144,000
GGV =
16 x 96,960

144,000
GGV =
1,551,360

GGV = 0.09 m

c. Calculate the solid GM and then apply the free surface correction to obtain the fluid GM.

KM 7.21 m
Solid KG ~ 6.44m
Solid GM 0.77m
FSE (GGV) ~ 0.09 m
Fluid GM 0.68 m
Issue No. 1 Page 113 of 348
ST. THERESE- MTC COLLEGES SEAMANSHIP 6
Iloilo, Philippines (Advance Trim, Stability and Stress)
STUDENT LEARNING MODULE
Revision No. 1 Effectivity date: Reviewed by: Approved by:

10 August 2020 QMR President

Do this

Activity 3 – Free Surface in divided tanks

A ship is floating in salt water and displaces 16,676 t, has a KG of 8.20 m and KM of 9.25 m,
when a double bottom tank is full of fresh water. The tank is rectangular 20 m long, 24 m
wide, 2.0 meters deep and is divided into 4 equal tanks by fore and aft watertight divisions.
Calculate the ship’s GM after 720 t of fresh water have been pumped out, leaving it slack.

Write your solution and answer in the space provided.

4.6 Free Surface Moments


The moment of inertia (i), often termed the second moment of area, of the free liquid
surface of a rectangular tank may be determined by:
Issue No. 1 Page 114 of 348
ST. THERESE- MTC COLLEGES SEAMANSHIP 6
Iloilo, Philippines (Advance Trim, Stability and Stress)
STUDENT LEARNING MODULE
Revision No. 1 Effectivity date: Reviewed by: Approved by:

10 August 2020 QMR President


3
lb
I= (m4)
12
If the value of I is multiplied by the liquid density then a value of “Free Surface Moments”
(FSM’s) (t – m) is obtained.
3
lb
FSM (t-m) = x dt
12
I b3
Consider the previous free surface effect formula: GGV = x dt
12W
FSMs
Therefore: GGV =
Displacement

Since: Final KG =
∑ of moments
Displacement

It is evident from the above that the greater the value of the free surface moments, the
greater the value of the effective KG and the greater the loss of GM (GG V).

In calculating the effective GM it is usual to make allowance for free surfaces by


incorporating the FSMs in the KG table where they must always be ADDED.

Using Free Surface Moments in Stability Calculations 


Another method of representing the free surface effect is to elevate the ship's center
of gravity by a suitable amount.

This simplifies the calculation of righting moments as a function of heel since the
tank's contribution to the center of gravity is considered to be fixed.

The amount by which the C.G. is elevated may be chosen such that the additional
righting moment produced by a small change of heel is the same as would be
produced by the shifting of the tank's contents.

This elevation of the C.G., multiplied by the weight of the ship, is called the free
surface moment, or FSM.

The primary disadvantage of using the FSM is that it does not accurately represent
the tank's effect on stability beyond a small increment of heel, since the FSM itself
can be very different at different heel angles.

Process me
Issue No. 1 Page 115 of 348
ST. THERESE- MTC COLLEGES SEAMANSHIP 6
Iloilo, Philippines (Advance Trim, Stability and Stress)
STUDENT LEARNING MODULE
Revision No. 1 Effectivity date: Reviewed by: Approved by:

10 August 2020 QMR President


Consider the previous example.

Example 1:

A ship has an initial displacement of 10,600 t and KG 7.60. A rectangular cargo oil tank of
length 30 m and breadth 20 m is partially filled with 9650 t of oil (RD 0.86). If the KG of the
oil is 8.00 m. Calculate the effective GM if the KM for the final displacement is 8.80 m.

Solution:
1. Calculate the FSMs using:
lb 3
FSM (t-m) = x dt
12
30 x 203
FSM (t-m) = x 0.86
12

FSM (t-m) = 30 x 8000 x 0.86 = 206,400 = 17,200 t-m


12 12
2. Taking moments about the keel, also adding the FSMs in the “moments” column,
calculate the fluid KG.
Weight (t) KG (m) Moments (t-m)
(W x KG)
Initial displacement 10,600 7.60 80,560
Cargo oil 9,650 8.00 77,200
FSMs 17,200
Final 20,250 t 8.64m 174,960 t-m

3. Apply the fluid KG value to the final KM to obtain the final fluid GM.

KM 8.800 m
Fluid KG ~ 8.640 m
Fluid GM 0.160 m

Example 2:

A ship has a solid GM of 1.02 m and displaces 7,600 t when a tank is partly filled with oil of
relative density 0.850. if the free surface moment of this tank is 1,250 m 4, find the fluid GM?

dt
Formula: Loss of GM = F.S.M. x
W
0.850
= 1250 t/m4 x
7,600t

1250 x 0.850
Loss of GM =
7,600 t
Issue No. 1 Page 116 of 348
ST. THERESE- MTC COLLEGES SEAMANSHIP 6
Iloilo, Philippines (Advance Trim, Stability and Stress)
STUDENT LEARNING MODULE
Revision No. 1 Effectivity date: Reviewed by: Approved by:

10 August 2020 QMR President


1062.5
Loss of GM = = 0.1398 = 0.14 m
7,600 t
Solid GM = 1.02 m
Loss of GM = - 0.14 m
Fluid GM = 0.88m

4.4 Free Surface Data


For a rectangular shaped tank, the calculation of the effects of free surface is
straightforward. However, not all tanks are this convenient regular shape and data relating
to tanks of all shapes on board are included in the ship’s Tank Sounding Data tables. This
data can be provided in a number of ways, the person on board conducting the ship’s
stability calculations must be familiar with the data supplied.

Representation of Free Surface Data in Tank Sounding/ullage Table

In tank sounding or ullage table free surface data for use in calculating the ship’s effective
KG and GM can be represented using alternative methods.

4.4.1 Method 1 – Free surface moments for an assumed density value


Consider the extract from a tank sounding table shown paying particular attention to the
column headings.
Table 4.1: Extract Tank Sounding Table

The table is for a cargo oil tank in a tanker – No. 2 Cargo Oil tank Starboard.

Free Surface Moments (FSMs) in tones-meters are tabulated for an assumed liquid density
of 0.740 t/m3.
Issue No. 1 Page 117 of 348
ST. THERESE- MTC COLLEGES SEAMANSHIP 6
Iloilo, Philippines (Advance Trim, Stability and Stress)
STUDENT LEARNING MODULE
Revision No. 1 Effectivity date: Reviewed by: Approved by:

10 August 2020 QMR President


Weight of liquid in the tank is tabulated against sounding for an assumed liquid density of
0.740 t/m3.

VCG (Vertical Center of Gravity or KG) indicates the vertical position of the oil within the ship
in terms of meters above the keel for the appropriate sounding.

Process me
Example 1:

A ship displaces 5,500 t and has a KG of 7.860 m No. 2 Cargo Oil tank starboard is filled to a
sounding of 150 cms with cargo oil RD 0.740. Calculate the final effective KG and GM if the
KM for the final condition is 8.00 m (Use the sounding table 1 extract given).
Table 4.2: Extract Tank Sounding Table

Solution:

1. Obtain cargo data from table.


Soundings 150 cms:

Weight = 107.46 t
KG /VCG = 2.302 m
FSMs = 373.7 t-m
2. Calculate the final effective KG and the final effective GM.
Weight (t) KG (m) Moments (t-m)
(W x KG)
Ship 5,500 7.860 43,230.0
Cargo oil 107.46 2.302 247.4
Issue No. 1 Page 118 of 348
ST. THERESE- MTC COLLEGES SEAMANSHIP 6
Iloilo, Philippines (Advance Trim, Stability and Stress)
STUDENT LEARNING MODULE
Revision No. 1 Effectivity date: Reviewed by: Approved by:

10 August 2020 QMR President


FSMs 373.7
Final 5607.46 7.820 43,851.1

KM 8.000
Fluid KG ~ 7.820
Fluid GM 0.18
For intermediate soundings, interpolation would be necessary to extract the values.

The previous example was straightforward, whereby the tank was loaded with liquid of the
same tabulated density.

However, if liquid of a different density to that assumed by the tables had been run into the
tank the tabulated values of both weight and FSMs would be in error.

Therefore, tabulated values must be corrected for the appropriate density. (Both mass
(weight) and the value of the FSMs are directly proportional to density).

Consider the previous example again. This time salt water ballast (RD 1.025) is loaded into
the tank to same level as before.

Example 2:

A ship displaces 5500 t and has a KG of 7.860 m No. 2 Cargo Oil tank starboard is filled to a
sounding of 150 cms with salt water ballast RD 1.025. Calculate the final effective KG and
GM if the KM for the final condition is 8.00 m (Use the sounding table 1 extract given).
Table 4.3: Extract Tank Sounding Table

Solution:
a. Obtain cargo data from table:
Soundings 150 cms:

Weight = 107.46 t
KG /VCG = 2.302 m
FSMs = 373.7 t-m
Issue No. 1 Page 119 of 348
ST. THERESE- MTC COLLEGES SEAMANSHIP 6
Iloilo, Philippines (Advance Trim, Stability and Stress)
STUDENT LEARNING MODULE
Revision No. 1 Effectivity date: Reviewed by: Approved by:

10 August 2020 QMR President

b. Calculate Actual mass of salt water ballast loaded is found by:

RD SW
Actual mass of SW = W x
Cargo oil density
1.025
= 107.46 x
0.740
107.46 x 1.025 110.1456
Actual mass of SW =
0.740
= 0.740
= 148.85 tonnes

c. Actual FSMs for the salt water ballast loaded is found by:

RD SW
Actual FSMs for SW = FSMs x
Cargo oil density

1.025
= 373.7 x
0.740

373.7 x 1.025 383.0425


Actual FSMs for SW =
0.740
= 0.740
= 517.625 tonnes

The KG of the liquid is above as before – not being affected by density….

d. Calculate the final effective KG and hence the final effective GM.

Weight KG (m) Moments (t-m)


(t) (W x KG)
Ship 5,500 7.860 43,230.0
SW Ballast 148.85 2.302 342.6
FSMs 517.6
Final 5648.85 7.805 44,090.2

Total moments 44,090.2 t−m


Final KG= = = 7.805 m
Total weights 5648.85 t
KM 8.000
Fluid KG ~ 7.805
Fluid GM 0.195

Do this
Activity 4 – Free Surface Moments for an Assumed Density Value
Issue No. 1 Page 120 of 348
ST. THERESE- MTC COLLEGES SEAMANSHIP 6
Iloilo, Philippines (Advance Trim, Stability and Stress)
STUDENT LEARNING MODULE
Revision No. 1 Effectivity date: Reviewed by: Approved by:

10 August 2020 QMR President


A ship displaces 7,800 t and has a KG of 8.500 m No. 2 Cargo Oil tank starboard is filled to a
sounding of 200 centimeters with cargo oil RD 0.876. Calculate the final effective KG and
GM if the KM for the final condition is 8.80 m (Use the sounding table 1 extract below).

0.876

Write your solution and answer in the space provided.

Write your solution and answer in the space provided.


Issue No. 1 Page 121 of 348
ST. THERESE- MTC COLLEGES SEAMANSHIP 6
Iloilo, Philippines (Advance Trim, Stability and Stress)
STUDENT LEARNING MODULE
Revision No. 1 Effectivity date: Reviewed by: Approved by:

10 August 2020 QMR President

Activity 5 – Free Surface Moments for an Assumed Density Value


A ship displaces 7,800 t and has a KG of 8.500 m No. 2 Cargo Oil tank starboard is filled to a
sounding of 225 centimeters with salt water ballast RD 1.025. Calculate the final effective
KG and GM if the KM for the final condition is 8.80 m (Use the sounding table 1 extract in
activity 4).

Write your solution and answer in the space provided.

Write your solution and answer in the space provided.


Issue No. 1 Page 122 of 348
ST. THERESE- MTC COLLEGES SEAMANSHIP 6
Iloilo, Philippines (Advance Trim, Stability and Stress)
STUDENT LEARNING MODULE
Revision No. 1 Effectivity date: Reviewed by: Approved by:

10 August 2020 QMR President

4.4.1 Method 2 –Moments of inertia (m4)


Consider the alternative extract from a tank sounding table for the same tank shown.

Table 4.4: Extract Tank Sounding Table


Issue No. 1 Page 123 of 348
ST. THERESE- MTC COLLEGES SEAMANSHIP 6
Iloilo, Philippines (Advance Trim, Stability and Stress)
STUDENT LEARNING MODULE
Revision No. 1 Effectivity date: Reviewed by: Approved by:

10 August 2020 QMR President

Moments of Inertia in meters4 (m4) are tabulated.


Volume of liquid in the tank in cubic meters (m3) is tabulated.
Density of the liquid is not considered.
To obtain the weight (mass) of the liquid and the Free Surface Moments which are to be
incorporated into the KG moments table both volume and I values as tabulated must be
multiplied by the density of the liquid in the tank.

Process me
Example 1:

A ship displaces 5500 t and has a KG of 7.860 m No. 2 Cargo Oil tank starboard is filled to a
sounding of 150 cms with cargo oil RD 0.74. Calculate the final effective KG and GM if the
KM for the final condition is 8.00 m (Use the sounding table 1 extract given).

a. Calculate the mass of oil in the tank.

Mass = Volume x Density


Mass = 145.22 x 0.740
Mass = 107.46 tonnes
Issue No. 1 Page 124 of 348
ST. THERESE- MTC COLLEGES SEAMANSHIP 6
Iloilo, Philippines (Advance Trim, Stability and Stress)
STUDENT LEARNING MODULE
Revision No. 1 Effectivity date: Reviewed by: Approved by:

10 August 2020 QMR President


b. Calculate the FSMs for the oil:

FSMs = I x Density
FMSs = 505.0 x 0.740
FMSs = 373.7 tonnes-meter

c. Calculate the final effective KG and hence the final effective GM.
Weight KG (m) Moments (t-m)
(t) (W x KG)
Ship 5,500 7.860 43,230.0
Cargo oil 107.46 2.302 247.4
FSMs 373.7
Final 5607.46 7.820 43,851.1

Total moments 43,851,1 t−m


Final KG= = = 7.820 m
Total weights 5607.46 t
KM 8.000
Fluid KG ~ 7.820
Fluid GM 0.180

Do this

Activity 6 – Method 2: Moments of inertia (m4)

A ship displaces 7,800 t and has a KG of 8.500 m No. 2 Cargo Oil tank Starboard is filled to
a sounding of 200 centimeters with cargo RD 0.976. Calculate the final effective KG and GM
if the KM for the final condition is 8.80 m (Use the extract sounding table below).

Tank 2CO Stbd


Sounding Volume LCG TCG VCG I
(cms) (m3) (m foap) (m) (m) (m4)
0 0.00 78.145 0.063s 1.503 0.0
25 17.57 78.290 2.963s 1.656 364.3
50 41.66 78.345 3.242s 1.785 405.8
75 66.64 78.366 3.365s 1.914 450.4
100 92.49 78.379 3.457s 2.044 497.6
125 118.81 78.389 3.525s 2.174 501.2
150 145.22 78.398 3.569s 2.302 505.0
175 171.69 78.406 3.602s 2.430 508.8
200 198.24 78.414 3.628s 2.557 512.6
225 224.88 78.421 3.648s 2.684 516.4

Write your solution and answer in the space provided.


Issue No. 1 Page 125 of 348
ST. THERESE- MTC COLLEGES SEAMANSHIP 6
Iloilo, Philippines (Advance Trim, Stability and Stress)
STUDENT LEARNING MODULE
Revision No. 1 Effectivity date: Reviewed by: Approved by:

10 August 2020 QMR President

Write your solution and answer in the space provided.


4.5 Factors Influencing Free Surface Effect
l b3
Consider the free surface formula for loss of GM GGV: GGV (m) = x dt
12W
It is clear that the breadth of the tank is the most important factor.

4.5.1 Tank Breadth

If a tank is subdivided, the loss of GM can be greatly reduced.


Issue No. 1 Page 126 of 348
ST. THERESE- MTC COLLEGES SEAMANSHIP 6
Iloilo, Philippines (Advance Trim, Stability and Stress)
STUDENT LEARNING MODULE
Revision No. 1 Effectivity date: Reviewed by: Approved by:

10 August 2020 QMR President

Process me
Example 1:

A ship has a displacement of 12,500 t and initial KG of 7.850 m. A rectangular double


bottom tank has the following dimensions: length 20 m, breadth 15 m and is filled with salt
water ballast (RD 1.025) to a sounding of 2.00 m. If the KM for the final condition is 8.00 m,
calculate the final effective GM.

Solution:

a. Calculate the mass and KG of the ballast water.

Mass = Volume x Density


Mass = (l x b x sounding) x density
Mass = (20 x 15 x 2) x 1.025 = 615 tonnes

Since it is a double bottom tank the KG of the ballast water will be half the sounding:
KG = 0.5 X 2.0 = 1.00 m

b. Calculate the FSMs for the rectangular free liquid surface.


3
lb
FSM (t-m) = x dt
12
3
20 x 15
FSM (t-m) = x 1.025
12

20 x 3375 x 1.025 69187.5


FSM (t-m) = = = 5765.6 t-m
12 12

Taking moments about the keel calculate the final KG and the final KM.

Weight (t) KG (m) Moments (t-m)


(W x KG)
Ship 12,500 7.850 98,125.0
SW Ballast 615 1.000 615.0
FSMs 5765.6
Final 13,115 7.968 104,505.6

Total moments 104,505.6t−m


Final KG= = = 7.968 m
Total weights 13,115 t

KM 8.000
Fluid KG ~ 7.968
Fluid GM 0.032
Issue No. 1 Page 127 of 348
ST. THERESE- MTC COLLEGES SEAMANSHIP 6
Iloilo, Philippines (Advance Trim, Stability and Stress)
STUDENT LEARNING MODULE
Revision No. 1 Effectivity date: Reviewed by: Approved by:

10 August 2020 QMR President

To satisfy the IMO intact stability requirements the minimum GM requirement for a ship is
0.15 m. this ship clearly does not satisfy that requirement.

Process me
Consider the same example but this time the tank will be equally subdivided into two tanks.

Example 2:

A ship has a displacement of 12,500 t and initial KG of 7.850 m. A rectangular double


bottom tank has the following dimensions: length 20 m, breadth 15 m and is filled with salt
water ballast (RD 1.025) to a sounding of 2.00 m. If the KM for the final condition is 8.00 m
calculate the final effective GM. It can be seen that there are now two tanks (Figure 4.7)
each having a breadth of 7.5 m.

7.5 m 7.5 m

15 m
Figure 4.7
Solution:

a. Calculate the mass and KG of the ballast water


Mass = Volume x Density
Mass = (l x b x sounding) x density
Mass = (20 x 15 x 2) x 1.025 = 615 tonnes

Alternatively:
Mass per tank = Volume x Density
Mass per tank = (l x b x sounding) x density
Mass = (20 x 7.5 x 2) x 1.025 = 307.5 tonnes

Total mass = 307.5 x 2 tanks = 615.0 tonnes

Since it is a double bottom tank the KG of the ballast water will be half the sounding:
KG = 0.5 X 2.0 = 1.00 m
Issue No. 1 Page 128 of 348
ST. THERESE- MTC COLLEGES SEAMANSHIP 6
Iloilo, Philippines (Advance Trim, Stability and Stress)
STUDENT LEARNING MODULE
Revision No. 1 Effectivity date: Reviewed by: Approved by:

10 August 2020 QMR President


b. Calculate the FSMs per tank for the rectangular free liquid surface.
lb 3
FSM (t-m) = x dt
12
3
20 x 7.5
FSM (t-m) = x 1.025
12

20 x 421.875 x 1.025 8648.43


FSM (t-m) = = = 720.7 t-m
12 12

Total FSM = 720.7 x 2 tanks = 1441.4 t-m

c. Taking moments about the keel, calculate the final KG and the final GM.

Weight KG (m) Moments (t-m)


(t) (W x KG)
Ship 12,500 7.850 98,125.0
SW Ballast 615 1.000 615.0
FSMs 1,441.4

Final 13,115 7.639 100,181.4 t-m

Total moments 100,181.4 t−m


Final KG= = = 7.639 m
Total weights 13,115 t

KM 8.000
Fluid KG ~ 7.639
Fluid GM 0.361

Subdividing the tank has resulted in the final GM being much improved.

This is a direct of the reduced free surface moments.

For the undivided tank the total FSMs where 5,765.6 t-m.

For the subdivided tank the total FSMs where 1441.4 t-m.

5,765.6
i.e = 1441.4 t-m
4
Issue No. 1 Page 129 of 348
ST. THERESE- MTC COLLEGES SEAMANSHIP 6
Iloilo, Philippines (Advance Trim, Stability and Stress)
STUDENT LEARNING MODULE
Revision No. 1 Effectivity date: Reviewed by: Approved by:

10 August 2020 QMR President

Process me
Example 3:

A ship has a displacement of 12,500 t and initial KG of 7.850 m. A rectangular double


bottom tank, which is equally subdivided into three compartments has a length 20 m,
breadth 15 m and is filled with salt water ballast (RD 1.025) to a sounding of 2.00 m. If the
KM for the final condition is 8.00 m calculate the final effective GM. It can be seen that there
are now three tanks (Figure 4.8) each having a breadth of 5.0 m.

5.0 m 5.0 m 5.0 m

15 m
Figure 4.8
Solution:
a. Calculate the mass and KG of the ballast water
Mass = Volume x Density
Mass = (l x b x sounding) x density
Mass = (20 x 15 x 2) x 1.025 = 615 tonnes

Alternatively:
Mass per tank = Volume x Density
Mass per tank = (l x b x sounding) x density
Mass = (20 x 5.0 x 2) x 1.025 = 205.0 tonnes
Total mass = 205.0 x 3 tanks = 615.0 tonnes

Since it is a double bottom tank the KG of the ballast water will be half the sounding:
KG = 0.5 X 2.0 = 1.00 m

b. Calculate the FSMs per tank for the rectangular free liquid surface.
lb 3
FSM (t-m) = x dt
12
3
20 x 5
FSM (t-m) = x 1.025
12

20 x 125 x 1.025 2562.5


FSM (t-m) = = = 213.542 t-m
12 12
Total FSM = 213.542 x 3 tanks = 640.625 t-m
c. Taking moments about the keel, calculate the final KG and the final KM.
Issue No. 1 Page 130 of 348
ST. THERESE- MTC COLLEGES SEAMANSHIP 6
Iloilo, Philippines (Advance Trim, Stability and Stress)
STUDENT LEARNING MODULE
Revision No. 1 Effectivity date: Reviewed by: Approved by:

10 August 2020 QMR President


Weight KG (m) Moments (t-m)
(t) (W x KG)
Ship 12,500 7.850 98,125.0
SW Ballast 615 1.000 615.0
FSMs 640.6

Final 13,115 7.578 99,380.6 t-m

Total moments 99,380.6 t−m


Final KG= = = 7.578 m
Total weights 13,115 t

KM 8.000
Fluid KG ~ 7.578
Fluid GM 0.422

As a result of subdividing the tank into three it is evident that the final GM is further
improved.
This is a direct result of the reduced free surface moments.
For the undivided tank the total FSMs where 5765.6 t-m.
For the tank equally subdivided into three compartments the total FSMs where 640.6 t-m.
Free surface moments have been reduced to one ninth of their original value…
5,765.6
i.e = 640.6 t-m
9
The following conclusions may be drawn from the previous examples on subdivided tanks.

Equally subdividing a tank has the following effects on the free surface:

a. No subdivision

Figure 4.9
b. a single subdivision

Figure 4.10
Issue No. 1 Page 131 of 348
ST. THERESE- MTC COLLEGES SEAMANSHIP 6
Iloilo, Philippines (Advance Trim, Stability and Stress)
STUDENT LEARNING MODULE
Revision No. 1 Effectivity date: Reviewed by: Approved by:

10 August 2020 QMR President


A single subdivision will reduce the free surface moments (and loss of GM) to one quarter of
the original value.

c. Two subdivisions (creating three compartments)

Figure 4.11

4.5.2 Tank Length

Free surface moments (and loss of GM) are directly proportional to the length of the tank
i.e. if the tank is doubled so will be the value of the free surface moments (and loss of GM).

4.5.3 Density

Free surface moment (and loss of GM) are directly proportional to the density of the liquid in
the tank, the greater the density of the liquid in the tank, the greater the FSMs and
subsequent loss of GM.

4.5.4 Ship Displacement

Free surface moment (and loss of GM) are inversely proportional to the displacement of the
ship. For a given tank, the loss of GM will be smaller as the displacement increases and vice-
versa. It should be noted that the actual free surface moments for any tank are not affected
by the ship’s displacement (since ship displacement is not included in the formula for their
calculation anyway).

Do this

Activity 7 – Free Surface Effect on Sub-divided Tank

A ship has a displacement of 7,800 t and initial KG of 8.50 m. A rectangular double bottom
tank has the following dimensions: length 25 m, breadth 20 m and is filled with salt water
ballast (RD 1.025) to a sounding of 2.30 m. If the KM for the final condition is 8.80 m
calculate the final effective GM. It can be seen that there are now two tanks each having a
breadth of 10.0 m.
Issue No. 1 Page 132 of 348
ST. THERESE- MTC COLLEGES SEAMANSHIP 6
Iloilo, Philippines (Advance Trim, Stability and Stress)
STUDENT LEARNING MODULE
Revision No. 1 Effectivity date: Reviewed by: Approved by:

10 August 2020 QMR President


Write your solution and answer in the space provided.

Assessment – Free Surface Effect Problem

A ship has a displacement of 13,000 t and initial KG of 7.55 m. A rectangular double bottom
tank, which is equally subdivided into three compartments has a length 25 m, breadth 18 m
and is filled with salt water to a sounding of 2.20 m. If the KM for the final condition is 7.75
m calculate the final effective GM. It can be seen that there are now three tanks each
having a breadth of 6.0 m.
Calculate the following:
1. Mass and KG of the ballast water
2. FSMs per tank for the rectangular free liquid surface.
3. Final KG
4. Final effective GM
Write your solutions and answer in the space provided.
Issue No. 1 Page 133 of 348
ST. THERESE- MTC COLLEGES SEAMANSHIP 6
Iloilo, Philippines (Advance Trim, Stability and Stress)
STUDENT LEARNING MODULE
Revision No. 1 Effectivity date: Reviewed by: Approved by:

10 August 2020 QMR President

Write your solutions and answer in the space provided.


Issue No. 1 Page 134 of 348
ST. THERESE- MTC COLLEGES SEAMANSHIP 6
Iloilo, Philippines (Advance Trim, Stability and Stress)
STUDENT LEARNING MODULE
Revision No. 1 Effectivity date: Reviewed by: Approved by:

10 August 2020 QMR President

Lesson 5: List and Heel

Learning Module 5.1: List and Heel

Competence, Course Outcomes and Learning Outcomes

Competence:

Control trim, stability and stress

Course Outcome:

1. Explain the importance of maintaining stability during loading, unloading


Issue No. 1 Page 135 of 348
ST. THERESE- MTC COLLEGES SEAMANSHIP 6
Iloilo, Philippines (Advance Trim, Stability and Stress)
STUDENT LEARNING MODULE
Revision No. 1 Effectivity date: Reviewed by: Approved by:

10 August 2020 QMR President


and in-transit in various conditions.
2. Calculate the effect on trim and stability of a ship in the event of damage
to and consequent flooding of a compartment and countermeasures to be
taken.

Learning Outcomes:

At the end of the lesson, the student should be able to:

1. Differentiate between the heel and list with respect to stability of the ship.

Discussion

5.1 Difference between List and Heel

So far stability has only been considered for a ship that is upright, whereby G is on the
centerline and the ship floats upright in still water. It is necessary to consider the position of
G in the transverse sense as well as the vertical.

There is a distinction to be made between the terms list and heel, this often being
overlooked or ignored completely.

List is the term used to describe a ship is inclined due to the distribution of weights within
it.

Heel is the term used to describe a ship that has been forcibly inclined by external forces
(wind, waves, etc.).

A “temporary” inclination of a ship, caused by outside forces such as winds, waves or during
ships turn.

When a list is corrected by increasing the displacement mass, the additional weight should
be placed as low as possible in the vessel. A vessel is said to be heeled when it is inclined by
an external force, e.g. from waves or wind.
Issue No. 1 Page 136 of 348
ST. THERESE- MTC COLLEGES SEAMANSHIP 6
Iloilo, Philippines (Advance Trim, Stability and Stress)
STUDENT LEARNING MODULE
Revision No. 1 Effectivity date: Reviewed by: Approved by:

10 August 2020 QMR President

Figure 5.1: Ship is heeled due to external force Figure 5.2: Listed Ship

Oral Assessment (Prelim)

Explain the following and make a video clip wearing upper uniform.

1. Simpson’s First, Second and Third Rule.


2. What is the effect of change of density when displacement is constant?
3. In a filled compartment, what is free surface effect and what is its impact on the ship’s stability when the
ship is listed?
4. In a partially filled compartment, what is free surface effect and what is its impact on the ship’s stability
when the ship is listed?
5. Differentiate heel and list

Oral presentation on the chosen research topic directions (this is a video oral presentation):

1. In this activity, you will work a one person only.


2. On a paper, please write down what you think is your idea regarding your chosen
research topic. Please accomplish this task in 3 minutes.
3. When done, please post your output on Power Point Slides and prepare to present.
4. Each person will be given at least 2 minutes to present their outputs.
5. During the presentation, please explain briefly why you think you listed items that
constitute the terms in your chosen research topic.

RUBRICS FOR GRADING ORAL ASSESSMENT

CRITERIA SCORE
Content (40%)
 The contetnt clearly incorporates salient points discussed about the
topic
Creativity and overall presentation (30%)
 The presentation showcases learner’s creativity and originality
Other elements (15%)
 The group effectively use gestures, eye contact, movements, and
facial expressions to convey the message.
 The speaker speaks audibly and clearly.
 The speaker uses props, and costumes appropriately and effectively.
Mastery (15%)
 The pair has mastered the concepts and delivered it well.

Research/Output
Issue No. 1 Page 137 of 348
ST. THERESE- MTC COLLEGES SEAMANSHIP 6
Iloilo, Philippines (Advance Trim, Stability and Stress)
STUDENT LEARNING MODULE
Revision No. 1 Effectivity date: Reviewed by: Approved by:

10 August 2020 QMR President


Research your OUTPUT for the following topics and explain it:

1. Righting lever (GZ)


2. Initial metacenter height above the keel (KM)
3. Transverse distance between the Center of Buoyancy and the Metacenter (BM).
4. Longitudinal Centre of Gravity (LCG) and Longitudinal Centre of Buoyancy (LCB)
5. Trimming moment

Power Point Presentation, make a 5 minutes presentation of the chosen research topic. The
format of the slides must be:

- Slide 1: Title of the chosen research topic and members


– Slide 2: Justification for choosing the topic
– Slide 3: Existing problems, issues, gaps relevant to the chosen topic
- Font Size and Style: Title: Arial 36-40” Body: 24-28”
- Animation and Design: Free Style
- Follow 7 x 7 rule (7 words and 7 lines in a slide)

RUBRIC FOR GRADING POWER POINT PRESENTATION OF A CHOSEN RESEARCH


TOPIC

CRITERIA SCORE
CONTENT (40%)
 The content clearly incorporates salient points discussed about
the topic.
SLIDE CREATION (15%)
 Presentation flows well and logically. Presentation reflects
extensive use of tools in a creative way.
SLIDE TRANSITIONS (15%)
 Transitions are smooth. Transitions enhance the presentation
PICTURES, CLIP ART BACKGROUND (15%)
 Images are appropriate. Layout is pleasing to the eye.
MECHANICS (15%
 No spelling errors. No grammar errors. Text is in authors’ own
words

RESEARCH WORK
Issue No. 1 Page 138 of 348
ST. THERESE- MTC COLLEGES SEAMANSHIP 6
Iloilo, Philippines (Advance Trim, Stability and Stress)
STUDENT LEARNING MODULE
Revision No. 1 Effectivity date: Reviewed by: Approved by:

10 August 2020 QMR President


Submit a well-written literature review of your research topic and explain it:

1. Tonnes Per Centimeter Immersion (TPC)


2. Fresh Water Allowance (FWA)
3. Dock Water Allowance (DWA)
4. List and loll
5. Wind effects

Submit a well-written literature review of your research topic. The table provided below is a
guide in making your literature review. You may add extra pages.

1. Draft article summary paragraph: 3-4 paragraphs of these will become your short
literature review.
2. Create an order for your paragraph summaries (try this with just 3-4 paragraphs that
relate). You have now created a draft of a short literature review.
3. Revise your short literature review to include paragraph introduction, body, and
conclusions/synthesis.

ARTICLE SUMMARY TABLE (Related Literature and Studies)

1 2
 Reference
 Aim/Objective/Scope
 Participants and sampling
 Context and framework
 Results or findings
 Implications Practice to Research
 Your Comments

Rubric for grading research work on reviewing literature

CRITERIA SCORE
INTRODUCTION (10%)
 Clear overview of paper, demonstrates importance of topic.
BODY (40%)
 Balanced viewpoint: Objective, balanced view from various
perspectives. Coherent theme: Each cited study related to
the topic and to other studies.
Issue No. 1 Page 139 of 348
ST. THERESE- MTC COLLEGES SEAMANSHIP 6
Iloilo, Philippines (Advance Trim, Stability and Stress)
STUDENT LEARNING MODULE
Revision No. 1 Effectivity date: Reviewed by: Approved by:

10 August 2020 QMR President


 Depth and breadth of research: Variety of studies and
attention to detail about the topic.
 Analysis: Collection of studies analyzed for differences and
commonalities about the topic.
CONCLUSION AND SYNTHESIS (10%)
 Information synthesized and brought to a logical
conclusion.
ORGANIZATION AND ALIGNMENT (15%)
 Organization and Alignment: Information logically
organized with good flow. Issues threaded throughout
paper.
MECHANICS, APA, REFERENCES (25%)
 Correct spelling, punctuation, sentence structure, word
usage.
 Correct use of APA in body of paper References correctly.
typed, appropriate number and quality

Lesson 6: List

Learning Module 6.1: List

Competence, Course Outcomes and Learning Outcomes

Competence:

Control trim, stability and stress

Course Outcome:

1. Explain the importance of maintaining stability during loading, unloading


and in-transit in various conditions.
2. Calculate the effect on trim and stability of a ship in the event of damage
to and consequent flooding of a compartment and countermeasures to be
taken.
Issue No. 1 Page 140 of 348
ST. THERESE- MTC COLLEGES SEAMANSHIP 6
Iloilo, Philippines (Advance Trim, Stability and Stress)
STUDENT LEARNING MODULE
Revision No. 1 Effectivity date: Reviewed by: Approved by:

10 August 2020 QMR President

Learning Outcomes:

At the end of the lesson, the student should be able to:

1. Create a table of moments:


– About the keel to determine final KG and
– About the center line to determine GG H to solve for final list of multiple weight
problems e.g. ship initially upright, ship initially listed and loading weights about
the centerline to complete upright).

Discussion

6.1 Loll, or List

A ship may develop a list for one of two reasons:

a) If the center of gravity is out of the center line of the ship;


b) If the ship has a negative GM.

These conditions are usually caused by faulty loading of the cargo and are generally
avoidable if the weights in the ship are properly distributed. They are not necessarily
dangerous, provided that the ship has an adequate range of stability, but are
obviously bad seamanship. In either case the ship will heel over until she is in
neutral equilibrium, that is, until B has moved out sufficiently to come vertically
under G. the first condition can occur in either stiff or tender ships and the list will
always be towards that side of the centerline to which G has moved. In second
condition the list may be to either side or may, under the influence of external
Issue No. 1 Page 141 of 348
ST. THERESE- MTC COLLEGES SEAMANSHIP 6
Iloilo, Philippines (Advance Trim, Stability and Stress)
STUDENT LEARNING MODULE
Revision No. 1 Effectivity date: Reviewed by: Approved by:

10 August 2020 QMR President


forces, change from one side to the other. It may also increase or decrease if
weights are taken away from, added to the ship.

6.2 Angle of List


Consider a ship floating upright as shown in Figure 6.1. The centers of gravity and buoyancy
are on the centerline. The resultant force acting on the ship is zero and the resultant
moment about the center of gravity is zero. b

W L
G
B

K
w
Figure 6.1

Now let a weight already on board the ship be shifted transversely such that G
moves to G1 as in Figure 6.2(a). This will produce a listing moment of W x GG 1, and
the ship will list until G1 and the center of buoyancy are in the same vertical line, as
in Figure 6.2(b).

b b

M
W

W L W1 θ L1
G G1 G
B G1
B B1 L

K
w

(b)
(a)
Figure 6.2
On this position G1 will also lie vertically under M so long as the angle of list is small.
Therefore, if the final positions of the metacenter and the center of gravity are
known, the final list can be found, using trigonometry, in the triangle GG 1M, which is
right-angled at G.

The final position of the center of gravity is found by taking moments about the keel
and about the centerline.
Issue No. 1 Page 142 of 348
ST. THERESE- MTC COLLEGES SEAMANSHIP 6
Iloilo, Philippines (Advance Trim, Stability and Stress)
STUDENT LEARNING MODULE
Revision No. 1 Effectivity date: Reviewed by: Approved by:

10 August 2020 QMR President


6.3 Multiple Weight Problems – Ship Initially Upright
List problems are solved by taking moments about the keel to determine final KG and then
GM and then taking moments about the center line to determine GG H.

Remember rules.
 If a weight is loaded G will move directly towards the center of gravity of the loaded
weight.
 If a weight is discharged G will move directly away the center of gravity of the
discharged weight.

The procedure is as follows:

1. Take moments about the keel to determine the final KG:

Final KG =
∑ of moments about keel (t−m)
Final Displacement ( t)
2. Calculate the final GM: GM = KM – KG

3. Take moments about the center line to calculate the final distance that G is off the
centerline, GGH:

(GG1) = GGH =
∑ of moments about centerline (t−m)
Final Displacement( t)

¿H
4. Calculate the list: Tan θ LIST =
GM FINAL

Process me
Example 1

A ship displaces 8000 tonnes, KG 7.60m and is initially upright. The following cargo is
worked:

Load: 300t at Kg 0.60, 6.1 m to port of CL;


250t at Kg 6.10, 7.6 m to starboard of CL:

Discharge: 50t from Kg 1.20 m, 4.6 m to port of CL;


500t from Kg 12.60 m, 4.6 m to starboard of CL.
Issue No. 1 Page 143 of 348
ST. THERESE- MTC COLLEGES SEAMANSHIP 6
Iloilo, Philippines (Advance Trim, Stability and Stress)
STUDENT LEARNING MODULE
Revision No. 1 Effectivity date: Reviewed by: Approved by:

10 August 2020 QMR President

Required:

Create a table and calculate the final angle of list on completion of cargo if the KM for the
final displacement is 9.36 m.

Weight discharged Weight loaded

500 t
M
4.6 m M

G GH G
50 t 7.6 m

4.6 m
W
6.1 m 250 t

300 t
K K
Portside Starboard side

Figure 6.3a Loading and discharging weights Figure 6.3b After completion ship listed 6.1° to Port

a. Take the moments about the keel to determine the final KG.

Weight (t) KG (m) Moments (t-m)


(W x KG)
Initial Ship 8,000 7.60 60,800.00
Load 300 0.60 180.00
Load 250 6.10 1,525.00
Discharge - 50 1.20 - 60.00
Discharge - 150 12.60 - 6,300.00
Final 8,000 t 7.018 m 56,145.00 t-m

Total moments 56,145t−m


Final KG= = = 7.018 m
Total weights 8,000 t
b. Calculate the final GM. KM 9.360 m
GM = KM ~ KG KG ~ 7.018 m
GMFINAL 2.342 m
Issue No. 1 Page 144 of 348
ST. THERESE- MTC COLLEGES SEAMANSHIP 6
Iloilo, Philippines (Advance Trim, Stability and Stress)
STUDENT LEARNING MODULE
Revision No. 1 Effectivity date: Reviewed by: Approved by:

10 August 2020 QMR President


c. Take moments about the centerline to determine GG H.

Weight (t) Dist off CL Port Starboard


Moments (t-m) Moments (t-m)
8,000 0.00 0.0
+ 300 6.10 1830.0
+ 250 7.60 1900.0
- 50 4.60 230.0
- 500 4.60 2300.0
4,130.0 t-m 2130.0 t-m
~ 2,130.0 t-m
2,000.0 t-m

GGH =
∑ of moments about centerline (t−m) = 2,000 = 0.25 m
Final Displacement(t) 8,000

d. Calculate the final list.


¿H 0.250
Tan θ LIST = = = 0.10675
GM FINAL 2.342
List θ = 0.10675 tan-1

List = 6° 5’ 35.7” (6’/60= 0.1°) = List = 6.1° Port

6.2 Multiple Weight Problems – Ship Initially Listed


M
If a ship is initially listed G must be off the center line as shown:

Since: ᶿ
¿H
Tan θ LIST =
GM FINAL
G
Then: GGH = Tanθ LISTx GM
Where GG1 is the initial listing lever to be incorporated into GH
the moments table for the ship. Consider the following example.
Figure 6.4

Process me
Issue No. 1 Page 145 of 348
ST. THERESE- MTC COLLEGES SEAMANSHIP 6
Iloilo, Philippines (Advance Trim, Stability and Stress)
STUDENT LEARNING MODULE
Revision No. 1 Effectivity date: Reviewed by: Approved by:

10 August 2020 QMR President

Example 2

A ship has a displacement of 15,000 tonnes, KG 8.600 m, KM 9.400 m and is listed 6° to


starboard. Cargo is worked as follows:

Load: 150 t at Kg 7.60, 5.0 m to port of CL;


305 t at Kg 8.00 on the CL:
95 t at Kg 8.00 m, 4.2 m to starboard of CL;

Calculate the final angle of list. (Assume KM remains constant).

Solution:
a. Calculate the initial GM.
GM = KM ~ KG
GM = 9.400 m ~ 8.400 m= 0.800 m

b. Calculate GGH:
GGH = Tanθ LISTx GM
GGH = Tan 6° x 0.800
GGH = 0.10510 x 0.800
GGH= 0.084 m

4° List
6° List to stbd
M to stbd M
W L
G GH G GH
4.2 m
305 t

150 t 95 t
5.0 m

K K
Issue No. 1 Page 146 of 348
ST. THERESE- MTC COLLEGES SEAMANSHIP 6
Iloilo, Philippines (Advance Trim, Stability and Stress)
STUDENT LEARNING MODULE
Revision No. 1 Effectivity date: Reviewed by: Approved by:

10 August 2020 QMR President

Figure 6.5a Ship listed 6° to starboard Figure 6.5b Ship listed 4° to starboard after loading

c. Take the moments about the keel to determine the final KG.

Weight (t) KG (m) Moments (t-m)


(W x KG)
Initial Ship 15,000 8.60 129,000.0
Load 150 7.60 1,140.0
Load 305 8.00 2,440.0
Load 95 8.00 760.0
Final 15,550 t 8.575 m 133,340.0 t-m

Total moments 133,340t−m


Final KG= = = 8.575 m
Total weights 15,550 t

d. Calculate the final GM.


KM 9.400 m
GMF = KM ~ KG KG ~ 8.575 m
GMFINAL 0.825 m

e. Take moments about the centerline to determine GG H.

Weight (t) Dist off CL Port Starboard


Moments (t-m) Moments (t-m)
15,000 0.084 1,260.0
+ 150 5.000 750.0
+ 305 0.000 0.0 0.0
+95 4.200 399.0
750.0 t-m 1,659.0 t-m
~ 750.0
909.0

GGH =
∑ of listing moments about centerline( t−m)
Final Displacement (t )
Issue No. 1 Page 147 of 348
ST. THERESE- MTC COLLEGES SEAMANSHIP 6
Iloilo, Philippines (Advance Trim, Stability and Stress)
STUDENT LEARNING MODULE
Revision No. 1 Effectivity date: Reviewed by: Approved by:

10 August 2020 QMR President


909
GGH = = 0.058 m
15,550

f. Calculate the final list.


¿H
Tan θ LIST = Tan θ LIST =
GM FINAL

0.058
Tan θ LIST = = 0.07030
0.825

List θ = 0.07030 tan-1

List = 4° 1’ 17.22” (1’/60= 0.0°)

Final List = 4.0° Starboard

6.3 Loading Weights about the Center Line to Complete Upright

A common question arises where the ship is near completion of loading and the remaining
cargo has to be distributed between two compartments that are either side of the center
line in such a way that ship completes upright.

To complete upright: Port moments MUST EQUALT to starboard moments .

Process me
Example 3

The ship initial displacement 17,500 tonnes, KG 8.800 m, KM 9.400 m and listed 3º to
starboard. 400 tonnes of cargo remains to be loaded where space is available in a tween
deck Kg 10.50 m, 7.0 m to port of CL and 10.0 m to starboard of CL. Assume KM remains
constant. Calculate the final GM and the amount of cargo to load in each space so that the
ship will complete loading upright.

Solution: METHOD 1

a. Calculate the initial GM.

Initial GM = KM ~ Initial KG
Initial GM = 9.40 m ~ 8.800 m= 0.600 m
Issue No. 1 Page 148 of 348
ST. THERESE- MTC COLLEGES SEAMANSHIP 6
Iloilo, Philippines (Advance Trim, Stability and Stress)
STUDENT LEARNING MODULE
Revision No. 1 Effectivity date: Reviewed by: Approved by:

10 August 2020 QMR President


b. Calculate GGH:
GGH = Tanθ LISTx GM
GGH = Tan 3° x 0.600 m
GGH= 0.031 m

c. Take the moments about the keel to determine the final KG and GM. (note that all 400 t
of cargo is loaded at KG 10.5 m so treat as a single weight).

Weight (t) KG (m) Moments (t-m)


Initial Ship 17,500 8.80 154,000
Load 400 10.50 4,200
Final 17,900 t 8.837 m 158,200 t-m

Total moments 158,200t−m


Final KG= = = 8.837 m
Total weights 117,900 t

d. Calculate the final GM.

GM Final = KM ~ KG KM 9.400 m
KG ~ 8.837 m
GMFINAL 0.563 m

e. Take moments about the centerline to determine GG H.

Weight (t) Dist. off CL Port Starboard


Moments (t-m) Moments (t-m)
17,500 0.031 542.5
+ 400 7.000 2,800
2,800 t-m 542.5 t-m
~ 542.5
2,257.5 t-m

If all 400 tonnes loaded onto the port side space the ship would complete with an excess
of 2,257.5 t-m moments to port. Therefore some of this 400 t must now be shifted to the
space on the starboard side a distance of 17.0 meters (7.0 m + 10.0 m).

Where:

2,257.5 t-m = w x d
2,257.5 t-m = w x (7.0 m + 10.0m)
Issue No. 1 Page 149 of 348
ST. THERESE- MTC COLLEGES SEAMANSHIP 6
Iloilo, Philippines (Advance Trim, Stability and Stress)
STUDENT LEARNING MODULE
Revision No. 1 Effectivity date: Reviewed by: Approved by:

10 August 2020 QMR President


2,257.5 t-m = 17w
2,257.5t−m
w= = 132.8 tonnes to be shifted from port to starboard
17 m
To complete the upright:
Load = 400 tonnes – 132.8 tonnes = 267.2 tonnes to port
Solution: METHOD 2
a. Calculate the initial GM.

Initial GM = KM ~ Initial KG
Initial GM = 9.40 m ~ 8.800 m= 0.600 m

b. Calculate GGH:
GGH = Tanθ LISTx GM
GGH = Tan 3° x 0.600 m = 0.031 m
c. Take the moments about the keel to determine the final KG and GM. (note that all 400 t
of cargo is loaded at KG 10.5 m so treat as a single weight).

Weight (t) KG (m) Moments (t-m)


Initial Ship 17,500 8.80 154,000
Load 400 10.50 4,200
Final 17,900 t 8.837 m 158,200 t-m

Total moments 158,200t−m


Final KG= = = 8.837 m
Total weights 117,900 t

d. Calculate the final GM.


KM 9.400 m
GM Final = KM ~ KG KG ~ 8.837 m
GMFINAL 0.563 m

e. Take moments about the centerline: Let x = cargo to load to port; (400 – x) = cargo to
load to starboard.

Weight (t) Dist off CL Port Starboard


Moments (t-m) Moments (t-m)
17,500 0.031 542.5
x 7.000 7x
(400-x) 10.000 (4000 – 10x)
7x 542.5 + (4000 – 10x)
Issue No. 1 Page 150 of 348
ST. THERESE- MTC COLLEGES SEAMANSHIP 6
Iloilo, Philippines (Advance Trim, Stability and Stress)
STUDENT LEARNING MODULE
Revision No. 1 Effectivity date: Reviewed by: Approved by:

10 August 2020 QMR President

To complete upright:
Port moments MUST EQUALT to starboard moments. Therefore:
7x = 542.5 + (4000 – 10x)
7x = 542.5 + 4000 – 10x
7x + 10x = 542.5 + 4000
17x = 4542.5
4542.5
x= = 267.2 tonnes to port
17
Load = 400 t – 267.2 t = 132.8 tonnes to starboard

6.4 List and Free Surface Effect


List with no free surface

List with no free surface

GV GH

G GH

CL Figure 6.4

The basic list triangle is GGHM. GM is the solid metacentric height, the GM that would exist if
the ship had no slack tanks.

GGH is the distance that G is off the centerline.

GGV is the virtual rise of G due to tank free surfaces. Since GM is reduced to G VM (the Fluid
GM) it can be seen that the angle of list has increased for the same distance that G is off the
center line (GGH).

The greater the free surface moments/free surface effect, the greater will be the list for the
same listing moments.

Process me
Example 4
Issue No. 1 Page 151 of 348
ST. THERESE- MTC COLLEGES SEAMANSHIP 6
Iloilo, Philippines (Advance Trim, Stability and Stress)
STUDENT LEARNING MODULE
Revision No. 1 Effectivity date: Reviewed by: Approved by:

10 August 2020 QMR President


A ship displacement 15,200 tonnes with a KG 0f 10.6 m and is initially upright. Ballast water
RD 1.022 is pumped into a rectangular Double Bottom tank length 24 m, breadth 10 m to a
sounding of 4.10 m. If the KG of the ballast water is 2.10 m and its transverse center of
gravity (TCG) is 5.15 m to the starboard of the center line calculate the final angle of list.
Assume the KM for the final displacement is 11.75 m.

1. Assuming no free surface moments;


2. Assuming for free surface moments.

Solution:

Mass of ballast water loaded = L x B x Depth x RD SW


Mass of ballast water loaded = 24 m x 10 m x 4 m x 1.025 = 984 tonnes

1. No Free Surface Moments


Weight (t) KG (m) Moments (t-m)
Initial Displacement 15,200 10.6 161,120
Load + 984 2.10 2,066.4
Final 16,184 10.08 163,186.4

KG = 10.08 m
Take the moments about the keel to determine final KG and GM.
GM Final = KM ~ KG
GM Final = 11.75 ~ 10.08
GM Final = 1.67 m

Take the moments about the center line to calculate the GG H.

Weight (t) Dist off CL Port Starboard


Moments (t-m) Moments (t-m)
15,200 0.00 0.0 0.0
+ 984 5.15 5,067.6
0.0 5,067.6

Net Listing moments 5,067.6t−m


GGH = = = 0.313 m
Final displacement 16,184 t
¿H
Calculate the final list: Tan θ LIST =
GM FINAL
0.313 m
Tan θ LIST =
1.67 m
θ LIST = 0.18743 tan-1
θ LIST = 10º 36’ 56.81”
θ LIST = 10.6º to Starboard
2. Including Free Surface Moments:
Issue No. 1 Page 152 of 348
ST. THERESE- MTC COLLEGES SEAMANSHIP 6
Iloilo, Philippines (Advance Trim, Stability and Stress)
STUDENT LEARNING MODULE
Revision No. 1 Effectivity date: Reviewed by: Approved by:

10 August 2020 QMR President


3
lb
FSMs = x dt
12
3
24 x 10 24 x 1000 x 1.025 24,600
FSMs = x 1.025 = = = 2,050 t-m
12 12 12

Take the moments about the keel to determine the final KG and GM:
Weight (t) KG (m) Moments (t-m)
Initial Displacement 15,200 10.6 161,120
Load + 984 2.10 2,066.4
FSMs 2,050.0
Final 16,184 10.21 165,236.4
GM Final = KM ~ KG
GM Final = 11.75 m ~ 10.21 m
GM Final = 1.54 m
The value of GGH is not affected by free surface moments.
¿H
Calculate the final list: Tan θ LIST =
GM FINAL
0.313 m
Tan θ LIST =
1.54 m
θ LIST = 0.20325 tan-1
θ LIST = 11º 29’ 17.95”
θ LIST = 11.4º to Starboard

Do This

Activity 1 – List Calculation Multiple Weights


A ship has a displacement of 8,820 t, KG 6.73 m and is upright. She then loads:
540 tonnes at 3.00 meters above the keel and on the centerline;
300 tonnes at 9.00 meters above keel and 2.5 m to port of centerline
200 tonnes at 5.00 meters above keel and 6.8 m to port of centerline
280 tonnes at 7.50 meters above keel and 4.5 m to starboard of centerline
If KM is then 7.81 m. Calculate the angle of heel.

Write your solution and answer in the space provided.

Write your solution and answer in the space provided.


Issue No. 1 Page 153 of 348
ST. THERESE- MTC COLLEGES SEAMANSHIP 6
Iloilo, Philippines (Advance Trim, Stability and Stress)
STUDENT LEARNING MODULE
Revision No. 1 Effectivity date: Reviewed by: Approved by:

10 August 2020 QMR President

Activity 2 – Loading Weights about the Center Line to Complete Upright


A ship displaces 6,400 tonnes, has a KG of 5.97 m, KM 7.62 m and is listed 5º to starboard,
500 tonnes of cargo are to be loaded into the wings of a tween deck at a distances of 5.00
Issue No. 1 Page 154 of 348
ST. THERESE- MTC COLLEGES SEAMANSHIP 6
Iloilo, Philippines (Advance Trim, Stability and Stress)
STUDENT LEARNING MODULE
Revision No. 1 Effectivity date: Reviewed by: Approved by:

10 August 2020 QMR President


off the center line to port and 7.00 m off the center line to starboard. Calculate the weight
to load in each wing in order that the ship may then be upright. (You can use any method).

Write your solution and answer in the space provided.

Write your solution and answer in the space provided.


Issue No. 1 Page 155 of 348
ST. THERESE- MTC COLLEGES SEAMANSHIP 6
Iloilo, Philippines (Advance Trim, Stability and Stress)
STUDENT LEARNING MODULE
Revision No. 1 Effectivity date: Reviewed by: Approved by:

10 August 2020 QMR President

Assessment 1 – List Calculation Multiple Weights

A ship of 9,500 tonnes displacement is listed 3.5º to starboard and has KM of 9.50 m and
KG 9.30 m. She loads 300 tonnes of bunkers in No. 3 DB tank portside (KG 0.60 m and
center of gravity 6.00 m from the centerline) and discharges two parcels of cargo each of 50
Issue No. 1 Page 156 of 348
ST. THERESE- MTC COLLEGES SEAMANSHIP 6
Iloilo, Philippines (Advance Trim, Stability and Stress)
STUDENT LEARNING MODULE
Revision No. 1 Effectivity date: Reviewed by: Approved by:

10 August 2020 QMR President


tonnes from the port side of No.2 shelter deck (KG 11 m and center of gravity 5.00 m from
the centerline). Calculate the final list.

Write your solution and answer in the space provided.

Assessment No. 2– Loading Weights About the Center


of Line to Complete Upright

A ship of 12,500 tonnes displacement KM of 7.00 m and KG 6.40 m has a list 3º list to
starboard and has yet to load 500 tonnes of cargo. There is space available in the tween
decks, centers of gravity 6.00 m each side of the centerline. Find how much cargo to load
on each side if the ship is to complete loading upright.
Issue No. 1 Page 157 of 348
ST. THERESE- MTC COLLEGES SEAMANSHIP 6
Iloilo, Philippines (Advance Trim, Stability and Stress)
STUDENT LEARNING MODULE
Revision No. 1 Effectivity date: Reviewed by: Approved by:

10 August 2020 QMR President

Write your solution and answer in the space provided.

Write your solution and answer in the space provided.


Issue No. 1 Page 158 of 348
ST. THERESE- MTC COLLEGES SEAMANSHIP 6
Iloilo, Philippines (Advance Trim, Stability and Stress)
STUDENT LEARNING MODULE
Revision No. 1 Effectivity date: Reviewed by: Approved by:

10 August 2020 QMR President

Lesson 7: Trim

Learning Module 7.1: Trim

Competence, Course Outcomes and Learning Outcomes


Issue No. 1 Page 159 of 348
ST. THERESE- MTC COLLEGES SEAMANSHIP 6
Iloilo, Philippines (Advance Trim, Stability and Stress)
STUDENT LEARNING MODULE
Revision No. 1 Effectivity date: Reviewed by: Approved by:

10 August 2020 QMR President


Competence:

Control trim, stability and stress

Course Outcome:

1. Explain the importance of maintaining stability during loading, unloading


and in-transit in various conditions.
2. Calculate the effect on trim and stability of a ship in the event of damage
to and consequent flooding of a compartment and countermeasures to be
taken.

Learning Outcomes:

At the end of the lesson, the student should be able to:

1. Calculate the following:


– Trim;
– Change of Trim;
– Moment to change trim by one centimeter (MCTC);
– Change of Draft aft and forward due to change of trim;
– Bodily sinkage or rise ;
– Displacement and weight added or removed.

Discussion

7.1 Trim
This is the longitudinal equivalent of heel, but whereas the latter is measured in angle, trim
is measured by the difference of drafts fore and aft as measured at the forward and aft
perpendiculars.

If the drafts are the same, fore and aft, or difference is zero then the ship said to be “on
even keel”.
Issue No. 1 Page 160 of 348
ST. THERESE- MTC COLLEGES SEAMANSHIP 6
Iloilo, Philippines (Advance Trim, Stability and Stress)
STUDENT LEARNING MODULE
Revision No. 1 Effectivity date: Reviewed by: Approved by:

10 August 2020 QMR President

Aft draft 5.00 m Forward draft 5.00 m

Figure 7.1 Even Keel

If the forward draft is greater that aft draft, the vessel is trimmed by the bow.

Aft draft 3.00 m Forward draft 5.50 m

Forward Trim

Figure 7.2 2.50 m Trimmed by the bow.

If the aft draft is greater than the forward draft, the vessel is trimmed by the stern.

Aft draft Forward draft 3.00 m


8.00 m
Aft Trim
Figure 7.3 5.00 m Trimmed by the stern

Causes of Trim:
1. Moving loads/mass fore and aft from one point to another changes the trim of the
ship but not the mean draft
2. Adding or removing loads to and from the ship will involve a decrease or increase of
the mean draft as well as a change in trim.
3. The change in water density where the ship floating changes the mean draft as well
as the trim of the ship.

7.2 Change of Trim


If a weight is shifted longitudinally the ship will experience a change of trim.

Process me
Example 1:
Issue No. 1 Page 161 of 348
ST. THERESE- MTC COLLEGES SEAMANSHIP 6
Iloilo, Philippines (Advance Trim, Stability and Stress)
STUDENT LEARNING MODULE
Revision No. 1 Effectivity date: Reviewed by: Approved by:

10 August 2020 QMR President


A ship is floating at a drafts forward 6.100 m and aft 6.300 m. A weight is then shifted
forward. The final drafts are forward 6.090 m and aft 6.310 m. Calculate the change of trim.

6.300 m 6.100 m

Figure 7.4a: Initial Trim 0.200 m by the stern


Solution:

a. Calculate the initial trim:

Formula: Initial Trim = Forward draft ~ Aft draft

Initial Trim = 6.100 m ~ 6.300 m


Initial Trim = 0.200 m by the stern

b. Calculate the Final trim:

Formula: Final Trim = Forward draft ~ Aft draft

Final Trim = 6.090 m ~ 6.310 m


Final Trim = 0.220 m by the stern

W1

6.310 m 6.090 m

Figure 7.4b: Final trim 0.220 meters by the stern

c. Calculate the change of trim (COT):

Initial Trim = 0.200 m by the stern


Final Trim = ~ 0.220 m by the stern
Final Trim = 0.020 m by the STERN
Note: trim by the stern has increased by 0.020 m.
Example 2:
Issue No. 1 Page 162 of 348
ST. THERESE- MTC COLLEGES SEAMANSHIP 6
Iloilo, Philippines (Advance Trim, Stability and Stress)
STUDENT LEARNING MODULE
Revision No. 1 Effectivity date: Reviewed by: Approved by:

10 August 2020 QMR President


A ship is floating at a draughts forward 6.000 m and aft 5.640 m. A weight is then shifted
aft. The final drafts are forward 5.680 m and aft 5.960 meters. Calculate the change of trim.

5.640 m 6.000 m

Figure 7.5a: Initial trim 0.360 meters by the head

Solution:

a. Calculate the initial trim:

Formula: Initial Trim = Forward draft ~ Aft draft


Initial Trim = 6.000 m ~ 5.640 m
Initial Trim= 0.360 m by the head

b. Calculate the initial trim:

Formula: Final Trim = Forward draft ~ Aft draft


Final Trim = 5.680 m ~ 5.960 m
Final Trim = 0.280 m by the stern

W1

5.960 m 5.680 m

Figure 7.5b: Final trim 0.280 meters by the stern

c. Calculate the change of trim (COT):

Initial Trim = 0.360 m by the head


Final Trim = + 0.280 m by the stern

Final Trim = 0.640 m by the STERN

Note: The ship initially trimmed by the head, completes trimmed by the stern.
Issue No. 1 Page 163 of 348
ST. THERESE- MTC COLLEGES SEAMANSHIP 6
Iloilo, Philippines (Advance Trim, Stability and Stress)
STUDENT LEARNING MODULE
Revision No. 1 Effectivity date: Reviewed by: Approved by:

10 August 2020 QMR President


7.3 Moment to Change Trim by One Centimeter (MCT1C)
This is the trimming moment required to change the ships trim by exactly one centimeter.

It is tabulated in the ships hydrostatic particulars and is used to determine the change of
trim that takes place when weights are shifted, loaded or discharged.
Change of trim can be calculated by the formula:

Trmmingmoment
COT (cms) =
MCT 1 C

where the trimming moment is: w x d

“w” being the weight shifted and


“d” being the distance through which the weight is shifted longitudinally

w xd
Thus: COT (cms) =
MCT 1C

Process me
Example 1

A weight of 170 tonnes is moved aft by distance of 25 m. If the MCTC for the current
draught is 250 t-m determine the final trim of the ship if the initial trim was 0.20 m by the
stern.

25 m W
W1

Figure 7.6

Solution:

a. Calculate the change of trim (COT):


Issue No. 1 Page 164 of 348
ST. THERESE- MTC COLLEGES SEAMANSHIP 6
Iloilo, Philippines (Advance Trim, Stability and Stress)
STUDENT LEARNING MODULE
Revision No. 1 Effectivity date: Reviewed by: Approved by:

10 August 2020 QMR President


w xd
COT (cms) =
MCT 1C
170t x 25 m
COT (cms) =
250 t−m
4250
COT (cms) =
250
COT (cms) = 17 cms – convert to meter (17/100)

COT = 0.17 m by the stern

b. Calculate the final trim:

Initial Trim= 0.200 m by the stern


COT = + 0.170 m further by the stern

Final Trim = 0.370 m by the STERN

7.4 Formula for Calculating MCT1C


In practice the MCT1C value will always be found for the draught in question in the ship’s
hydrostatic particulars. However, in examinations it may have to be calculated and the
formula for calculating MCT1C is:

W x GML
MCT1C =
100 LBP
Where: “W” is the displacement
“GML” is the longitudinal metacentric height
“LBP” is the length between perpendiculars

The derivation of this formula is as follows:

The ship in Figure 7.7 is on even keel with a weight on deck.

W G L
B
Issue No. 1 Page 165 of 348
ST. THERESE- MTC COLLEGES SEAMANSHIP 6
Iloilo, Philippines (Advance Trim, Stability and Stress)
STUDENT LEARNING MODULE
Revision No. 1 Effectivity date: Reviewed by: Approved by:

10 August 2020 QMR President

Figure 7.7
The weight is shifted aft along the deck through distance “d” meters.in accordance with the
formula:
wxd
GG1 =
W
G and B become horizontally separated creating a trimming lever. This causes the ship to
trim by the stern until B attains a new position vertically below the new longitudinal center
of gravity, G1 (Figure 7.9).

W1 W

G1 G

Figure 7.8
GG1 ML is a right angled triangle where:

OPP
Tan θ =
ADJ

¿1
Therefore: Tan θ =
GML

wxd ¿1
Since: GG1 = andTan θ = then:
W GML

wxd
Tan θ =
W x GML

TRIM (m)
Also in Figure 7.10: Tan θ =
LBP (m)

ML
θ

W
W1
Issue No. 1 Page 166 of 348
ST. THERESE- MTC COLLEGES SEAMANSHIP 6
Iloilo, Philippines (Advance Trim, Stability and Stress)
STUDENT LEARNING MODULE
Revision No. 1 Effectivity date: Reviewed by: Approved by:

10 August 2020 QMR President

G1 G
L
W1
L1
B1 B
W

Figure 7.9

AP FP

W1

TRIM
LBP

Figure 7.10
If the change of trim due to the weight shifted is exactly 1 cm, then:

TRIM (m) 0.01(m) 1


Tan θ =
LBP(m)
=Tan θ =
LBP(m)
= Tan θ =
100 LBP
(1)

wxd MCTC
Since: Tan θ = equals: Tan θ = (2)
W x GML W x GML

(Because (w x d) is the moment to change the trim by exactly 1 cm)

Bringing formulae (1) and (2) together gives:

1 MCTC
=
100 LBP W x GML

W x GML
Rearranging this gives: MCT1C =
100 LBP

Process me
Example
Issue No. 1 Page 167 of 348
ST. THERESE- MTC COLLEGES SEAMANSHIP 6
Iloilo, Philippines (Advance Trim, Stability and Stress)
STUDENT LEARNING MODULE
Revision No. 1 Effectivity date: Reviewed by: Approved by:

10 August 2020 QMR President


A ship is 125 meters long. Has a displacement of 3,650 tonnes and a longitudinal
metacentric height of 200 meters. Calculate her Moment to Change Trim by 1 Centimeter
(MCT1C).
W x GML 3,650t x 200 m 730,000
MCT1C = = = = 58.4 tonne-meters
100 LBP 100 x 125 m 12,500

7.5 Longitudinal Center of Flotation (LCF or F)


This is at the geometric center of the ship’s water plane area and is the point about which
the ship will trim.

Consider the ship where a weight is shifted longitudinally.

W1 W

W1
L
F
W
L1
Figure 7.11 (a)

It can be seen that the LCF is in the same position in the ship’s length as the point where
the initial and final waterlines intersect.

Because the water plane area changes shape and size with draught the position of the LCF
will also change with draught.

The position of the LCF is normally quoted in hydrostatic data as being so many meters
forward of the after perpendicular (foap).

The position of the LCF is important because if a ship experiences a change of trim, some of
that change of trim must be applied to the aft draught and the remainder applied to the
forward draught as can be seen.

W1

L
W1
Issue No. 1 Page 168 of 348
ST. THERESE- MTC COLLEGES SEAMANSHIP 6
Iloilo, Philippines (Advance Trim, Stability and Stress)
STUDENT LEARNING MODULE
Revision No. 1 Effectivity date: Reviewed by: Approved by:

10 August 2020 QMR President

F1 L1
W

CL
Figure 7.11 (b)

7.6 Change of Draft Forward and Aft Due to Change of Trim

When a ship changes trim it will obviously cause a change in the drafts forward and aft. One
of these will be increased and the other decreased. A formula must now be found that will
give the change in drafts due to change in trim.

Consider a ship floating upright, as shown in Figure 7.12 (a). F1represents the position of
the center of flotation, which is l meters from aft. The ship’s length is L meters and a weight
“w” is on deck forward. Let this weight now be shifted aft a distance of “d” meters. The ship
will trim about F1 and change the trim “t” cm by the stern, as shown in Figure 7.11 (b).
W1C is a line drawn parallel to the keel. “A” represents the new draft aft and “F” the new
draft forward. The trim is therefore equal to A – F and, since the original trim was zero, this
must also be equal to the change of trim.

L = LBP

W F1 L

AP FP
w
Figure 7.12 (a)

W1 W
C
W1 F1 L t
Issue No. 1 Page 169 of 348
ST. THERESE- MTC COLLEGES SEAMANSHIP 6
Iloilo, Philippines (Advance Trim, Stability and Stress)
STUDENT LEARNING MODULE
Revision No. 1 Effectivity date: Reviewed by: Approved by:

10 August 2020 QMR President

L1
A W

CL F
X l
Figure 7.12 (b)

Let “x” represent the change of draft aft due to the change of trim and let “y” represent the
change forward. In the triangles WW1F1 and W1L1C, using the property of similar triangles:

x cm t cm lm x t cm
= or x cm =
1m L m Lm
l
Therefore Change of draft aft in cm = x Change of trim in cm
L

where l = the distance of center of flotation from aft in meters and


L = the ships length in meters
It will also be noticed that x + y = t.

Therefore: Change of draft Forward in cm = Change of trim – Change of draft Aft

Process me
Example 1: The Effect of Shifting a Weight

A ship is 126 m long is floating at draft of 6.50 m forward and 7.50 m aft. The center of
flotation is 3 m aft of amidships. MCT1C is 250 tonnes-meter. Displacement is 6000 tonnes.
Calculate the new drafts if a weight of 120 tonnes already on board is shifted forward a
distance of 50 meters.
L =126 m

d=45m
W W1

W l = 60 m l = 63 + 3 m = 66m L
F 3m

AP FP
Amidship
Issue No. 1 Page 170 of 348
ST. THERESE- MTC COLLEGES SEAMANSHIP 6
Iloilo, Philippines (Advance Trim, Stability and Stress)
STUDENT LEARNING MODULE
Revision No. 1 Effectivity date: Reviewed by: Approved by:

10 August 2020 QMR President

Figure 7.13
a. Trimming moment = w x d
= 120 t x 50 m
Trimming moment = 6000 t-m by the head

Trimming moment 6000t−m


b. Change of trim (CT) = =
MTC 1 C 250t−m

Change of trim (CT) = 24 cm by the head


l
c. Change of draft aft = x Change of trim (CT)
L

L 126 m
Where: l= 2 = 2
= 63m – 3m = 60 m
60 m
Change of draft aft = x 24 cm
126 m
60 x 24 cm
Change of draft aft =
126
Change of draft aft = 11.43 cm (convert to meters)
Change of draft aft = 0.11 m

l
d. Change of draft forward = x Change of trim (CT)
L
Where: l = 63m + 3m = 66 m
66 m
Change of draft forward = x 24 cm
126 m
66 x 24 cm
Change of draft forward =
126
Change of draft forward = 12.57 cm (convert to meters)

Change of draft forward = 0.13 m

e. Calculate the Final draughts:


Aft Forward
Initial draughts 7.50 m 6.50 m
Change of trim _- 0.11 m + 0.13 m
New Draughts 7.39 m 6.63 m

Example 2
Issue No. 1 Page 171 of 348
ST. THERESE- MTC COLLEGES SEAMANSHIP 6
Iloilo, Philippines (Advance Trim, Stability and Stress)
STUDENT LEARNING MODULE
Revision No. 1 Effectivity date: Reviewed by: Approved by:

10 August 2020 QMR President


Calculate the new drafts in a ship of 125 m length, when a weight of 100 tonnes is shifted
aft for a distance of 60 m from the tipping center. The initial drafts were 4.80 m forward
and 4.76 m aft. The center of flotation is 2.5 m abaft amidships and the MCT1C is 120 t-m.

L =125 m

d=60m

W1 W

W l= 62.5 – 2.5 m = 60m L


F 2.5 m l = 62.5 + 2.5 = 65 m

AP FP
Amidship
Figure 7.14
wxd 100t x 60 m
a. Change of trim (CT) =
MTC 1C
=
120 t−m
= 50 cm by the stern

LCF (F) is 65 m from forward and 60 m from aft


l
b. Change of draft aft = x Change of trim (CT)
L

L 125 m
Where: l= 2= 2 = 62.5m – 2.5m = 60 m
60 m
Change of draft aft = x 50 cm
125 m
60 x 50 cm
Change of draft aft =
125
Change of draft aft = 24 cm (convert to meters)
Change of draft aft = 0.24 m

c. Change of draft forward = Change of trim (CT) – Change of draft aft

Change of draft forward = 0.50m – 0.24m


Change of draft forward = 0.26 m

d. Calculate the Final draughts:


Aft Forward
Initial draughts 4.76 m 4.80 m
Change of trim _+0.24 m - 0.26 m
New Draughts 5.00 m 4.54 m
Issue No. 1 Page 172 of 348
ST. THERESE- MTC COLLEGES SEAMANSHIP 6
Iloilo, Philippines (Advance Trim, Stability and Stress)
STUDENT LEARNING MODULE
Revision No. 1 Effectivity date: Reviewed by: Approved by:

10 August 2020 QMR President


Example 3

A box-shaped vessel 100m x 15m x 6 m depth floats in salt water on an even keel at 4.00 m
draft (see Figure 7.15). Calculate the final drafts if a weight of 65 tonnes already on board is
shifted a distance of 45 meters aft.
64t
40 m 64t
L
W LCF L W1 LCF L1

W
l F
A F
A
Figure 7.15

Salt water density: 1.025


a. Calculate for BML:
2
L
Formula: BML =
12d
100 x 100 10,000
BML = =
12 x 4 48

BML = 208.3 m

b. Calculate for W:
W = L X B x d x dt
W = 100 x 15 x 4 x 1.025
W = 6,150 tonnes

c. Calculate for MCT1C:


Since BG is small compared with GML. BML can be used instead of GML:

W x BML 6,150t x 208.3 m


MCT1C = =
100 LBP 100 x 100 m
1,281,045
MCT1C = =128.1 t-m/cm
10,000

d. Calculate for Change of Trim:


wxd
Change of trim (CT) =
MTC 1C
65 t x 45 m
=
128.1t−m
Issue No. 1 Page 173 of 348
ST. THERESE- MTC COLLEGES SEAMANSHIP 6
Iloilo, Philippines (Advance Trim, Stability and Stress)
STUDENT LEARNING MODULE
Revision No. 1 Effectivity date: Reviewed by: Approved by:

10 August 2020 QMR President


2,925 m
=
128.1t−m/cm

Change of trim (CT) = 22.83 cm by the stern


Change of trim (CT) = 23 cm by the stern

e. Calculate for Change of drafts:


l
Change of draft aft = x Change of trim (CT)
L

1
Change of draft aft = x 23 cm
2
23 cm
Change of draft aft =
2
Change of draft aft = 11.5 cm (convert to meters)
Change of draft forward = 0.12 m

f. Calculate the Final draughts:


Aft Forward
Initial draughts 4.00 m 4.00 m
Change of trim _+ 0.12 m - 0.12 m
New Draughts 4.115 m 3.885 m

7.7 Trim Calculations – Changing of Loading


7.7.1 The effect of loading, Discharging and Moving Weights
When a weight is loaded at the center of flotation it will produce no trimming moment, but
the ship’s drafts will increase uniformly so that the ship displaces an extra weight of water
equal to the weight loaded. If the weight is now shifted forward or aft away from the center
of flotation, it will cause a change of trim. From this it can be seen that when a weight is
loaded away from the center of flotation, it will cause both a bodily sinkage and a change of
trim.

Similarly, when a weight is being discharged, if the weight is first shifted to the center of
flotation it will produce a change of trim, and if it is then discharged from the center of
flotation the ship will rise bodily. Thus, both a change of trim and bodily rise must be
considered when a weight is being discharged away from the center of flotation.

If weights are loaded or discharged the effect to bodily rise or sinkage must also be
considered.

The following procedure should be followed when loading a weight.


Issue No. 1 Page 174 of 348
ST. THERESE- MTC COLLEGES SEAMANSHIP 6
Iloilo, Philippines (Advance Trim, Stability and Stress)
STUDENT LEARNING MODULE
Revision No. 1 Effectivity date: Reviewed by: Approved by:

10 August 2020 QMR President


1. First, assume the weight to be added at the center of flotation and calculate the
sinkage by the formula:
w
Bodily Sinkage/Rise cms=
TPC

2. Next, assume the weight to be shifted from the center of flotation, forward or aft, to
its new position. Calculate the change of trim caused by this by the formula:
wxd
Chang of trim (CT) =
MTC 1C
3. Calculate the change of draft at either end, due to the above change of trim, as
described in “Change of draft due to change of trim”.
4. Add the results of (a) to the draft at each end: then apply the results of (c). This will
give the ship’s new drafts after the weight has been added.
5. When weights are discharged, we follow the same procedures as for weights loaded.
Remember, however, that they will cause a bodily rise instead of sinkage: also be
careful about which way will cause the trim to change.
6. If a number of weights are loaded or discharged, use the total effects of all of them.
The bodily rise or sinkage will equal the net weight, divided by the TPC. The moment
changing trim will be the algebraic sum of all the moments.

Process me
Example 1

A ship 90 m long is floating at draft of 5.50 m forward and 6.00 m aft. The center of
flotation is 1.5 m aft of amidships, TPC = 12 tonnes and MCT1C is 120 t-m (see figure
7.15). Calculate the new drafts if a total weight of 450 tonnes is loaded in a position 14 m
forward of amidships.

L =90 m

l= 45 m
450 t

d=1.5+14=15.5m
W L
14m
Issue No. 1 Page 175 of 348
ST. THERESE- MTC COLLEGES SEAMANSHIP 6
Iloilo, Philippines (Advance Trim, Stability and Stress)
STUDENT LEARNING MODULE
Revision No. 1 Effectivity date: Reviewed by: Approved by:

10 August 2020 QMR President


la= 43.5 m F 1.5 m

AP FP
D Aft = 6.00 m Figure 7.16 D Fwd = 5.50
m
a. Calculate bodily sinkage:
w
Bodily Sinkage =
TPC
450 t
Bodily Sinkage =
12t /cm
Bodily Sinkage = 37.5 cm (convert to meters)

Bodily Sinkage= 0.375 m

b. Calculate Change of Trim:

d= 1.5 m + 14 m = 15.5 m
wxd
Chang of trim (CT) =
MTC 1C
450 t x 15.5 m 6,975t−m
Change of trim (CT) = =
120 t−m/cm 120t−m/ cm
Change of trim (CT) = 58.13 cm by the head

c. Calculate Change of drafts:


l
Change of draft aft = x Change of trim (CT)
L
where: l = 43.5 m (from aft to LCF)
43.5 m
Change of draft aft = x 58.13 cm
90 m
2,528.66 cm
Change of draft aft =
90
Change of draft aft = 28.10 cm (convert to meters)

Change of draft aft =0.281 m

d. Change of draft forward =Change of trim (CT) – Change of draft aft

Change of draft forward = 0.581m –0.281m


Change of draft forward = 0.300 m
e. Calculate the Final draughts:
Issue No. 1 Page 176 of 348
ST. THERESE- MTC COLLEGES SEAMANSHIP 6
Iloilo, Philippines (Advance Trim, Stability and Stress)
STUDENT LEARNING MODULE
Revision No. 1 Effectivity date: Reviewed by: Approved by:

10 August 2020 QMR President


Aft Forward
Initial draughts 6.000 m 5.500 m
Bodily sinkage + 0.375 m + 0.375 m
6.375 m 5.875 m
Change due trim - 0.281 m + 0.300 m
New Draughts = 6.094 m 6.175 m

Example 2

A box-shaped vessel 40 m x 6 m x 3 m depth floats in salt water on an even keel at 3.00 m


draft (see Figure 7.17). Calculate the final drafts if a weight of 45 tonnes is discharged from
a position 6 m from forward. MCT1C IS 8.4 t-m.

F1 L

F
A
Figure 7.17
a. Calculate for BML:
WPA
Formula: TPC=
97.56
40 m X 6 m 240 m
TPC= =
97.56 97.56

TPC = 2.46 tonnes/cm

b. Calculate bodily rise:


w
Bodily rise=
TPC
45 t
Bodily rise= = 18.29 cm = 0.183 m
2.46 t/cm
c. Calculate for Change of Trim:
wxd
Change of trim (CT) =
MTC 1C
45 t x 14 m
=
8.4 t−m
630 m
=
8.4 t−m/cm
Change of trim (CT) = 75 cm by the stern
Issue No. 1 Page 177 of 348
ST. THERESE- MTC COLLEGES SEAMANSHIP 6
Iloilo, Philippines (Advance Trim, Stability and Stress)
STUDENT LEARNING MODULE
Revision No. 1 Effectivity date: Reviewed by: Approved by:

10 August 2020 QMR President


d. Calculate Change of drafts:
l
Change of draft aft = x Change of trim (CT)
L
1 75 cm
Change of draft aft = x 75 cm =
2 2

Change of draft aft = 37.5 cm (convert to meters)

Change of draft aft = 0.375 m


l
Change of draft forward = x Change of trim (CT)
L
1 75 cm
= x 75 cm=
2 2
Change of draft forward= 37.5 cm (convert to meters)
Change of draft forward = 0.375 m

e. Calculate the Final draughts:


Aft Forward
Initial draughts 3.000 m 3.000 m
Bodily rise - 0.183 m - 0.183 m
2.817 m 2.817 m
Change due trim + 0.375 m - 0.375 m
New Draughts 3.192 m 2.442 m

Example 3

A ship 100 m long is floating at draft of 7.00 m forward and 6.80 m aft. The longitudinal
center of flotation is 45 m forward of aft perpendicular, TPC is15 tonnes and MCT1C is 150
t-m (see figure 7.18). Calculate the new drafts if a total weight of 200 tonnes is loaded 20 m
forward of aft perpendicular.

L =100 m

200t

d?
W 20m L
Aft l= 45 m F Fwd
7.00m
6.80m
AP FP
Figure 7.18
Issue No. 1 Page 178 of 348
ST. THERESE- MTC COLLEGES SEAMANSHIP 6
Iloilo, Philippines (Advance Trim, Stability and Stress)
STUDENT LEARNING MODULE
Revision No. 1 Effectivity date: Reviewed by: Approved by:

10 August 2020 QMR President


a. Calculate bodily sinkage:
w
Bodily Sinkage =
TPC
200t
Bodily Sinkage =
15t /cm
Bodily Sinkage = 13.33 cm (convert to meters)

Bodily Sinkage = 0.133m

b. Calculate Change of Trim:


d=(l – distance from AP) 45 m -20 m = 25 m
wxd
Change of trim (CT) =
MTC 1C
200 t x 25 m
Change of trim (CT) =
150t−m/cm
5000t−m
=
150t −m/cm
Change of trim (CT) = 33.3 cm by the stern

c. Calculate Change of drafts aft and forward:


l
Change of draft aft = x Change of trim (CT)
L
where: l = 45 m (from aft perpendicular to LCF )
45 m
Change of draft aft = x 33.3 cm
100 m
1,498.5 cm
Change of draft aft =
100
Change of draft aft = 14.99 cm (convert to meters)
Change of draft aft = 0.150 m

Change of draft forward =Change of trim (CT) – Change of draft aft


Change of draft forward = 0.333 m –0.150 m
Change of draft forward = 0.183 m

Note: Weight is loaded aft of the LCF (F) so the ship will trim by the STERN.
Issue No. 1 Page 179 of 348
ST. THERESE- MTC COLLEGES SEAMANSHIP 6
Iloilo, Philippines (Advance Trim, Stability and Stress)
STUDENT LEARNING MODULE
Revision No. 1 Effectivity date: Reviewed by: Approved by:

10 August 2020 QMR President

d. Calculate the Final draughts:


Aft Forward
Initial draughts 6.800 m 7.000 m
Bodily sinkage + 0.133 m + 0.133 m
6.933 m 7.133 m
Change due trim + 0.150 m - 0.183 m
New Draughts 7.083 m 6.950 m

7.7.2 Multiple Weights Problems

Problems involving multiple weights require a tabular approach to be adopted where


moments are taken about the LCF.

Process me
Example 1

A ship 120 m in length floats at draughts 6.24m forward and 6.36 m aft. LCF is 54 m
forward of AP, TPC 14.2 and MCT1C 116 t-m.
Note: Longitudinal center of gravity (lcg) Forward Aft Perpendicular (foap)

The following cargo is worked:


Load 120 t longitudinal center of gravity (lcg) 10.0 foap
Load 68 t longitudinal center of gravity lcg 86 foap
Discharge 36 t longitudinal center of gravity lcg 22 foap
Discharge 48 t longitudinal center of gravity lcg 60 foap
Calculate the final draughts.

Procedures:

1. The weights loaded and discharged are summed to find the net weight loaded or
discharged to be entered in column 1 of the table.
2. The weights are listed as positive values, regardless of whether the weight is being
loaded or discharged to be entered in column 2 of the table.
3. The distance that each weight is loaded or discharged from the LCF is listed in
column 3 of the table.
4. Trimming moments are calculated (Column 2 being multiplied by column 3) and
entered in column 4 or 5 depending on whether they are head or stern moments.(It
is here that mistakes are commonly made whereby the moments are often applied
the wrong way).
Issue No. 1 Page 180 of 348
ST. THERESE- MTC COLLEGES SEAMANSHIP 6
Iloilo, Philippines (Advance Trim, Stability and Stress)
STUDENT LEARNING MODULE
Revision No. 1 Effectivity date: Reviewed by: Approved by:

10 August 2020 QMR President


Solution:

120m

60m 48 t
22m 36t d=32m d=6m
54m .F 86m

10m d=44m d=32m


120 68 t

AP l = 60m FP

Figure 7.19

a. Calculate the distances each cargo from F:

Longitudinal Center of Flotation (LCF or F) = 54 m foap.


Distance of Load 120 t (54m – 10 m) = 44 m
Distance of Load 68 t (86 m – 54 m) = 32m
Distance of Load 36 t (54m – 22 m) = 32 m
Distance of Load 48 t (60m – 54 m) = 6m
1 2 3 4 5
Weight (t) Weight (t) Dist. From Head Stern
LCF (m) moments moments
(t-m) (t-m)
120 (L) 120 44 m 5,280
68 (L) 68 32 m 2,176
- 36 (D) 36 32 m 1,152
- 48 (D) 48 6m 288
104 Load 3,328 5,568
2,240
b. Calculate bodily sinkage:
w
Bodily Sinkage=
TPC
104 t
Bodily Sinkage=
14.2t /cm
Bodily Sinkage= 7. 3 cm (convert to meters)

Bodily Sinkage= 0.073 m


c. Calculate Change of Trim:
Issue No. 1 Page 181 of 348
ST. THERESE- MTC COLLEGES SEAMANSHIP 6
Iloilo, Philippines (Advance Trim, Stability and Stress)
STUDENT LEARNING MODULE
Revision No. 1 Effectivity date: Reviewed by: Approved by:

10 August 2020 QMR President


Trimming moments
Change of trim (CT) =
MTC 1C
2240 t
Change of trim (CT) =
166 t−m/cm
Change of trim (CT) = 19.31 cm by the STERN

d. Calculate Change of drafts aft and forward:


l
Change of draft aft = x Change of trim (CT)
L
where: l = 54 m (from aft perpendicular to LCF )
54 m
Change of draft aft = x 19.31 cm
120 m
1,042.7 cm
Change of draft aft =
120
Change of draft aft = 8.68 cm (convert to meters)

Change of draft aft =0.087 m

Change of draft forward =Change of trim (CT) – Change of draft aft


Change of draft forward = 0.193 m – 0.087 m
Change of draft forward = 0.106 m

e. Calculate the Final draughts:


Aft Forward
Initial draughts 6.360 m 6.240 m
Bodily sinkage + 0.073 m + 0.073 m
6.433 m 6.313 m
Change due trim + 0.087m - 0.106 m
New Draughts 6.520 m 6.207 m

Example 2

A ship 220 m in length (LBP) with a TPC of 58 t and MCT1C of 650 t-m while the center of
flotation is 3.8 m forward of amidships. The drafts are 10.57 m forward and 11.00 m aft.
The following cargoes is then worked:
Load 450 t 85 m forward of amidships
Load 1,555 t 43 m abaft of F (Center of flotation)
Discharge 824 t 28 m forward of amidships
Discharge 669 t 35 m abaft of F (Center of flotation)
Illustrate and calculate the final draughts.
Issue No. 1 Page 182 of 348
ST. THERESE- MTC COLLEGES SEAMANSHIP 6
Iloilo, Philippines (Advance Trim, Stability and Stress)
STUDENT LEARNING MODULE
Revision No. 1 Effectivity date: Reviewed by: Approved by:

10 August 2020 QMR President

220m

669 d=35m
28m
824
.F d=24.2
3.8m
d=43m d=81.2
1555 85m 450

L/2= 110m FP
AP l = 110 + 3.8= 113.8m
Figure 7.20
Solution:
a. Calculate the distances each cargo from F:
Dist. of Loaded weight 450 t (85 m – 3.8 m) = 81.2 m
Dist. of Loaded weight 1,555 t = 43 m
Dist. of Discharged weight 824 t (28 m – 3.8 m) = 24.2 m
Dist. of Discharged weight 669 t = 35 m

1 2 3 4 5
Weight (t) Weight (t) Dist. From Head Stern
LCF (m) moments moments
(t-m) (t-m)
450 (L) 450 81.2 m 36,540
1,555 (L) 1,555 43 m 66,865
- 824 (D) 824 24.2 m 19,940.8
- 669 (D) 669 35 m 23,415
512 Loaded 59,955 86,805.8
26,850.8
b. Calculate bodily sinkage:
w
Bodily Sinkage=
TPC
512t
Bodily Sinkage=
58t /cm
Bodily Sinkage= 8.83 cm (convert to meters)

Bodily Sinkage = 0.09 m

c. Calculate Change of Trim:


Issue No. 1 Page 183 of 348
ST. THERESE- MTC COLLEGES SEAMANSHIP 6
Iloilo, Philippines (Advance Trim, Stability and Stress)
STUDENT LEARNING MODULE
Revision No. 1 Effectivity date: Reviewed by: Approved by:

10 August 2020 QMR President


Trimming moments
Change of trim (CT) =
MTC 1C
26,850.8 t
Change of trim (CT) =
650 t−m/cm
Change of trim (CT) = 41.31 cm (convert to meters)

Change of trim (CT) = 0.41 m by the STERN

d. Calculate Change of drafts aft and forward:


l
Change of draft aft = x Change of trim (CT)
L
where: l = 54 m (from aft perpendicular to LCF )
113.8 m
Change of draft aft = x 0.41 m
220 m

46.658 m
Change of draft aft =
220
Change of draft aft = 0.21 m

Change of draft forward =Change of trim (CT) – Change of draft aft


Change of draft forward = 0.41 m –0.21 m
Change of draft forward = 0.20 m

e. Calculate the Final draughts:


Aft Forward
Initial draughts 11.10 m 10.57 m
Bodily sinkage + 0.09 m + 0.09 m
11.19 m 10.66 m
Change due trim + 0.21m - 0.20 m
New Draughts 11.40 m 10.46 m

Example 3

A ship 6500 tonnes displacement has a drafts 7.00 m forward and 8.00 m aft. MCT1C is 100
tonnes-m, TPC is 20 tonnes, center of flotation is amidships, 500 tonnes of cargo are then
discharged from each of the following holds (see Figure 7.21).
No. 1 hold, center of gravity 40 m forward of amidships
No. 2 hold, center of gravity 25 m forward of amidships
No. 3 hold, center of gravity 20 m aft of amidships
No. 4 hold, center of gravity 50 m aft of amidships
Issue No. 1 Page 184 of 348
ST. THERESE- MTC COLLEGES SEAMANSHIP 6
Iloilo, Philippines (Advance Trim, Stability and Stress)
STUDENT LEARNING MODULE
Revision No. 1 Effectivity date: Reviewed by: Approved by:

10 August 2020 QMR President


The following bunkers are also loaded:
140 tonnes at 15 m forward of
60 tonnes at 12 m aft of
Calculate new draughts forward and aft.

d=50m
d=20m .F d=25m
d=40m
500t 500t 500t
500t
60t d=12m 140t d=15m
AP FP
Solution: Figure 7.21
a. Calculate the Net Weight discharged/loaded:
Total cargo discharged = 4 holds x 500 t = 2,000 tonnes
Total bunkers loaded = 140 t + 60 t = - 200 tonnes
Net weight =1,800 tonnes -discharged

b. Calculate bodily sinkage:


w
Bodily Rise =
TPC
1,800t
Bodily Rise= = 90cm = 0.90 meters
20 t/cm
1 2 3 4 5
Weight (t) Weight (t) Dist. From Head Stern
LCF (m) moments moments
(t-m) (t-m)
-500 (D) 500 40 20,000
-500 (D) 500 25 12,500
-500 (D) 500 20 10,000
-500 (D) 500 50 25,000
140 (L) 140 15 2,100
60 (L) 60 12 720
1,800 Discharged 37,100 33,220
3,880
c. Calculate Change of Trim:
Trimming moments
Change of trim (COT) =
MTC 1C
Issue No. 1 Page 185 of 348
ST. THERESE- MTC COLLEGES SEAMANSHIP 6
Iloilo, Philippines (Advance Trim, Stability and Stress)
STUDENT LEARNING MODULE
Revision No. 1 Effectivity date: Reviewed by: Approved by:

10 August 2020 QMR President


3,880 t
Change of trim (COT) =
100t−m/cm
Change of trim (COT) = 38.8 cm by the HEAD

d. Calculate Change of drafts aft and forward:


Note: Since the center of flotation is amidships,

Change of draft aft = Change of draft forward


1
=
change of trim
2
38.8 cm
Change of draft aft = = 19.4 cm = 0.19 m
2

e. Calculate the Final draughts:


Aft Forward
Initial draughts 8.00 m 7.00 m
Bodily rise - 0.90 m - 0.90 m
7.10 m 6.10 m
Change due trim - 0.19 m + 0.19 m
New Draughts 6.91 m 6.29 m

Do This

Activity 1 – Trim Calculation Multiple Weights


A ship if floating at drafts of 7.10 meters forward and 7.70 meters. The following cargo is
loaded in the flowing holds:

No. 1 hold, 20 tonnes, center of gravity 30 m forward of amidships


No. 2 hold, 45 tonnes, center of gravity 25 m forward of amidships
No. 3 hold, 60 tonnes, center of gravity 15 m aft of amidships
No. 4 hold, 30 tonnes, center of gravity 3 m aft of amidships
The center of floatation is amidships, MCT1C 200 t-m and TPC 35 tonnes. Calculate the new
drafts forward and aft.

Write your solution and answer in the space provided.


Issue No. 1 Page 186 of 348
ST. THERESE- MTC COLLEGES SEAMANSHIP 6
Iloilo, Philippines (Advance Trim, Stability and Stress)
STUDENT LEARNING MODULE
Revision No. 1 Effectivity date: Reviewed by: Approved by:

10 August 2020 QMR President

Write your solution and answer in the space provided.

Do This

Activity 2 – Trim Calculation Multiple Weights

A ship 100 m long has a center of flotation 3.00 meters aft of amidships and is floating at
drafts 3.20 m forward and 4.40 meters aft. A TPC 10 tonnes, MCT1C 150 t-m. A 30 tonnes
of cargo is discharged from 20 m forward of amidships and 40 tonnes is discharged from 12
m aft of amidships. Calculate the new final drafts.
Write your solution and answer in the space provided.
Issue No. 1 Page 187 of 348
ST. THERESE- MTC COLLEGES SEAMANSHIP 6
Iloilo, Philippines (Advance Trim, Stability and Stress)
STUDENT LEARNING MODULE
Revision No. 1 Effectivity date: Reviewed by: Approved by:

10 August 2020 QMR President

Write your solution and answer in the space provided.


Issue No. 1 Page 188 of 348
ST. THERESE- MTC COLLEGES SEAMANSHIP 6
Iloilo, Philippines (Advance Trim, Stability and Stress)
STUDENT LEARNING MODULE
Revision No. 1 Effectivity date: Reviewed by: Approved by:

10 August 2020 QMR President

Do This

Activity 3 – Trim Calculation – Removing Weights

A vessel floats at drafts of 5.12 m forward and 4.69 m aft. Her T.P.C. is 14.4, MCT1C IS 102
and F is amidships. What will be the new drafts after 98 tonnes of water has been pumped
out of the fore peak tank, 52 meters forward of amidships?

Write your solution and answer in the space provided.


Issue No. 1 Page 189 of 348
ST. THERESE- MTC COLLEGES SEAMANSHIP 6
Iloilo, Philippines (Advance Trim, Stability and Stress)
STUDENT LEARNING MODULE
Revision No. 1 Effectivity date: Reviewed by: Approved by:

10 August 2020 QMR President

Assessment 1 – Trim Calculation Multiple Weights

A ship arrives Iloilo International Port with drafts 6.80 m forward and 7.20 m aft. 500 tonnes
of cargo is then discharged from each of 4 holds.

The center of gravity of No. 1 hold is 40 m forward of amidships


The center of gravity of No. 2 hold is 25 m forward of amidships
The center of gravity of No. 3 hold is 20 m aft of amidships
The center of gravity of No. 4 hold is 50 m aft of amidships

Also, 50 tonnes of cargo is loaded in a position whose center of gravity is 15.00 m aft of
amidships and 135 tonnes of cargo center of gravity 40 m forward of amidships. TPC 15
tonnes, MCT1C is 400 t-m. The center of flotation is amidships. Calculate the final drafts.

Write your solution and answer in the space provided.


Issue No. 1 Page 190 of 348
ST. THERESE- MTC COLLEGES SEAMANSHIP 6
Iloilo, Philippines (Advance Trim, Stability and Stress)
STUDENT LEARNING MODULE
Revision No. 1 Effectivity date: Reviewed by: Approved by:

10 August 2020 QMR President

Write your solution and answer in the space provided.


Issue No. 1 Page 191 of 348
ST. THERESE- MTC COLLEGES SEAMANSHIP 6
Iloilo, Philippines (Advance Trim, Stability and Stress)
STUDENT LEARNING MODULE
Revision No. 1 Effectivity date: Reviewed by: Approved by:

10 August 2020 QMR President

Assessment 2 – Trim Calculation Multiple Weights

MV St Therese is 140 meters long, has a TPC 0f 20 and an MCT1C of 120, while the center
of flotation is 3.00 meters abaft amidships. The draft is 7.10 meters forward and 7.25
meters aft. The following cargo is then worked:

Loaded 120 tonnes, 50 meters forward of F.


Loaded 70 tonnes, 20 meters abaft F.
Discharged 90 tonnes , 30 meters forward of F.

Calculate the new displacement and new drafts.


Write your solution and answer in the space provided.
Issue No. 1 Page 192 of 348
ST. THERESE- MTC COLLEGES SEAMANSHIP 6
Iloilo, Philippines (Advance Trim, Stability and Stress)
STUDENT LEARNING MODULE
Revision No. 1 Effectivity date: Reviewed by: Approved by:

10 August 2020 QMR President

Write your solution and answer in the space provided.


Issue No. 1 Page 193 of 348
ST. THERESE- MTC COLLEGES SEAMANSHIP 6
Iloilo, Philippines (Advance Trim, Stability and Stress)
STUDENT LEARNING MODULE
Revision No. 1 Effectivity date: Reviewed by: Approved by:

10 August 2020 QMR President

Lesson 8: Curve of Statical Stability

Learning Module 8.1: Curve of Statical Stability

Competence, Course Outcomes and Learning Outcomes

Competence:

Control trim, stability and stress

Course Outcome:

1. Explain the importance of maintaining stability during loading, unloading


and in-transit in various conditions.
2. Calculate the effect on trim and stability of a ship in the event of damage
to and consequent flooding of a compartment and countermeasures to be
taken.

Learning Outcomes:

At the end of the lesson, the student should be able to:


Issue No. 1 Page 194 of 348
ST. THERESE- MTC COLLEGES SEAMANSHIP 6
Iloilo, Philippines (Advance Trim, Stability and Stress)
STUDENT LEARNING MODULE
Revision No. 1 Effectivity date: Reviewed by: Approved by:

10 August 2020 QMR President


1. Construct a curve of statical stability and
2. Calculate the area under the curve in meter-radians up to a stated angle using
Simpson’s Rules

Introduction

The curve of statical stability, or GZ curve as it is most commonly referred to, is a graphical
representation of the ship’s transverse statical stability.

Transverse statical stability is the term used to describe the ability of a ship to return to the
upright, when it has been forcibly heeled by an external force and is and is momentarily at
rest when floating in still water.

Righting Moment (t-m) = GZ (M) X Displacement

At any angle of heel, it is the horizontal disposition of G and B that determines the GZ value.

As the ship progressively heels over the righting lever, GZ, increases to some maximum
value and then decreases until at some angle of heel it becomes negative i.e. it becomes a
capsizing lever.

Calculating the value of GZ at specified angles of heel for a ship’s particular condition of
loading, will allow a curve of statical stability, or GZ curve, to be produced.

The greater the value of GZ, the greater will be the area under the curve. Minimum
standards with respect to the area under the curve (and other criteria) are specified in the
“Code on Intact Stability (IMO)” and these are incorporated in the government legislation of
most countries that adopt the IMO conventions.

It is aim of this module to review the method of actually producing a curve of statical
stability and be able to extract basic information from it.

8.1 Curves of Statical Stability (GZ Curves)

In order to illustrate the length of the righting arms at each angle, these levers are
displayed as a curve, the stability curve. The curve shows the stability of the ship at heeling
angles from zero upwards (usually up to 60°).

The curve of the righting arms applies to a specific draft and weight distribution. Any other
condition gives another curve.
Issue No. 1 Page 195 of 348
ST. THERESE- MTC COLLEGES SEAMANSHIP 6
Iloilo, Philippines (Advance Trim, Stability and Stress)
STUDENT LEARNING MODULE
Revision No. 1 Effectivity date: Reviewed by: Approved by:

10 August 2020 QMR President


The righting arm must be sufficiently large at each angle for the ship to right itself during
various conditions such as bad weather.

The curve must be derived and evaluated:

 During loading or discharging and before sailing


 For changes of weight distribution during the voyage due to consumption of fuel
and/or drinking water. This has to be considered before leaving port.

The calculation and appraisal of the curve can be produced by computer.

With each new entry the ship’s loading computer immediately recalculates the righting arms
and thus, the curve.

The responsible officers must be thoroughly aware of the basis of this calculation. They
must know which rules to apply if the ships stability decreases to such an extent that action
is required.

Remarks about stability curve:

– As long as the weight of the ship does not change, the shape of the curve is
determined by the ships form and the water-plane area. With a small freeboard or
small draft the water-plane area can change considerably under heel.
– The curve is only applicable if the ship lies in calm water. If the ship sails in waves
the water-plane area changes and thus, a continuously changing curve results.
– The curve shows:
 the righting moment or the righting arm at each angle of heel;
 the energy produced by the righting moment to resist a heel from 0° to any
chosen angle
– The magnitude of the levers and thus the area under the curve, will decrease if:
 weight (cargo, ballast, etc.) is placed above G;
 weight is discharged under G.
– Changes in the direction of the GZ curve are caused when:
 The deck edge is submerged
 The coaming is submerged
 The bilge rises above water

These are all due to the waterline breadth changing positively or negatively.
– The horizontal scale is the distance calculated from 0° to the point where the
righting arm is negative (more than 83°). That is, the point at which the ship will
dynamically overturn.
Issue No. 1 Page 196 of 348
ST. THERESE- MTC COLLEGES SEAMANSHIP 6
Iloilo, Philippines (Advance Trim, Stability and Stress)
STUDENT LEARNING MODULE
Revision No. 1 Effectivity date: Reviewed by: Approved by:

10 August 2020 QMR President


– The ship will overturn at the top of the curve with a static load.

0.6

0.5
0.4
Deck Immersion

0.3

0.2

0.1
0.0

10.0 20.0 30.0 40.0 50.0 60.0 70.0 80.0 90.0


Heeling angle (Degrees)
Figure 8.1
= point of inflection

Figure 8.2

8.2 Procedures for Calculating GZ Values


Example 1

MV Almar completes loading with a displacement of 29,000 t and a KG corrected for free
surfaces of 8.92 m. Calculate the GZ values and GM if the KM for the loaded displacement is
9.46 m. (Use extract tabulated KN Values for MV Almar)

Solution:

Using the formula: GZ = KN – (KG Sin θ )


Issue No. 1 Page 197 of 348
ST. THERESE- MTC COLLEGES SEAMANSHIP 6
Iloilo, Philippines (Advance Trim, Stability and Stress)
STUDENT LEARNING MODULE
Revision No. 1 Effectivity date: Reviewed by: Approved by:

10 August 2020 QMR President


a. Calculate the GZ values for the loaded condition.

To calculate the KG Sin θ :


8.92 m x sin 10° = 1.55 meters
8.92 m x sin 20° = 3.05 meters
8.92 m x sin 30° = 4.46 meters
8.92 m x sin 40° = 5.73 meters
8.92 m x sin 60° = 7.72 meters
8.92 m x sin 80° = 8.78 meters

Heel 10° 20° 30° 40° 60° 80°


KN 1.65 3.31 4.93 6.30 7.86 8.07
KG* Sin Heel - 1.55 - 3.05 - 4.46 - 5.73 - 7.72 - 8.78

GZ (arm) 0.10 0.26 0.47 0.57 0.14 - 0.71

Table 1: KN Values Table


Issue No. 1 Page 198 of 348
ST. THERESE- MTC COLLEGES SEAMANSHIP 6
Iloilo, Philippines (Advance Trim, Stability and Stress)
STUDENT LEARNING MODULE
Revision No. 1 Effectivity date: Reviewed by: Approved by:

10 August 2020 QMR President

b. Calculate the GM: KM 9.460 m


GM = KM - KG
KG ~ 8.920 m
GM 0.54 m

8.3 Procedures for Constructing the Curve of Statical Stability

The following steps should always be undertaken when producing a curve of statical
stability.

1. Determine the ship’s displacement and effective KG for the condition being
considered (effective KG being that taking into account free liquid surfaces in tanks).
2. From the hydrostatic data find the value of KM for the ship’s displacement.
3. Find the GMFLUID using: GM = KM – KGFLUID

4. Enter KN tables (or curves) and obtain KN value in meters for each angle of heel
given.
5. Using: GZ = KN – (KG Sin θ ) determine the GZ values for the angles of heel
given.
6. Plot the GZ values
7. Before joining all points on the curve construct a vertical at 5.73°and from the base
upwards mark off the value of the effective GM (using the GZ scale). From this point
draw a straight line to the origin of the curve to be drawn. This will indicate the
initial trend of the curve at small angles of heel and will assist in sketching the actual
curve between the origin and the first plotted GZ value.

(GZ and GM are closely related at small angles of heel)

The curve for the GZ values calculated in the example is shown. Note the construction using
the initial GM value of 0.54m.

Heel 10 20 30 40 60 80
KN 1.65 3.31 4.93 6.30 7.86 8.07
KG* Sin Heel 1.55 3.05 4.46 5.73 7.72 8.78
GZ (arm) 0.10 0.26 0.47 0.57 0.14 - 0.71

0.54
Issue No. 1 Page 199 of 348
ST. THERESE- MTC COLLEGES SEAMANSHIP 6
Iloilo, Philippines (Advance Trim, Stability and Stress)
STUDENT LEARNING MODULE
Revision No. 1 Effectivity date: Reviewed by: Approved by:

10 August 2020 QMR President

57.3

Heel (deg.)

Figure 8.3 Curve of static stability

Example 2

MV St. Therese completes loading with a displacement of 3,500 t and a KG of 4.9 m.


Calculate the GZ values and draw the stability curve for this loading condition.

ANGLE OF HEEL – DEGREES


DISPLACEMENT
10° 20° 30° 40° 50° 60°
(t)
4500 0.894 1.782 2.611 3.453 4.103 4.552

4250 0.891 1.791 2.630 3.489 4.150 4.552

4000 0.890 1.793 2.658 3.521 4.192 4.584

3750 0.892 1.799 2.699 3.551 4.228 4.614


3500 0.919 1.862 2.853 3.672 4.257 4.684
3250 0.905 1.831 2.804 3.626 4.277 4.666

Figure 8.4: Cross Curves of Stability (KN Values)

Heeling angle 10° 20° 30° 40° 50° 60°


KN 0.919 1.862 2.853 3.672 4.257 4.684

KG* Sin Heel 0.851 1.676 2.450 3.150 3.754 4.244


GZ (arm) 0.068 0.186 0.403 0.522 0.555 0.440
Issue No. 1 Page 200 of 348
ST. THERESE- MTC COLLEGES SEAMANSHIP 6
Iloilo, Philippines (Advance Trim, Stability and Stress)
STUDENT LEARNING MODULE
Revision No. 1 Effectivity date: Reviewed by: Approved by:

10 August 2020 QMR President

Deck Immersion

GMO

Heeling angle (Degrees)


Figure 8.5: Curves of Static Stability

8.4 Information Available from the Curve of Statical Stability


Based on the curve (Figure 8.3) the following information can be obtained from it.

1. The GZ value for any angle of heel;


This can be used to calculate the moment of statical stability for the ship at that
particular angle of heel if the formula”
Righting moment (t-m) = GZ (m) x Displacement (t) is applied.
2. The maximum GZ and the angle of heel at which it occurs.
3. The range of positive stability and the angle of vanishing stability (AVS);
4. The approximate angle of deck edge immersion (θ DEI).

From the curve:

The maximum GZ value is 0.57 m and occurs at an approximate angle of heel of 39°.

θ DEI

Max. GZ

AVS

Range of positive stability


Issue No. 1 Page 201 of 348
ST. THERESE- MTC COLLEGES SEAMANSHIP 6
Iloilo, Philippines (Advance Trim, Stability and Stress)
STUDENT LEARNING MODULE
Revision No. 1 Effectivity date: Reviewed by: Approved by:

10 August 2020 QMR President

Heel (deg.)

Figure 8.6: Curves of Static Stability

8.5 Determining the area under the curve

The area under the curve represents the energy involved in the dynamic stability. The
problem with calculating the area under the curve is that the levers are given in cm or
meters on the vertical axis and the angles are given in degrees on the horizontal axis.

To overcome this, the heeling angles have to be expressed in terms of distance. This is done
by converting the heeling angles to radians, by dividing the number of heeling degrees by
57.3°. When the area below the curve is so calculated, centimeter radians or meter radians
are obtained.

The area under the curve can now be calculated in the following ways:

– Make a triangle with the same area as the area under the curve. This is less
accurate, but still a good way to obtain a quick impression of the area (surface
area= ½ x base x height). This can be applied to regulation a1, a2 and a3.
– Make a rectangle of the area. (surface area = base x height).
This can be applied to regulations b1 and b2.
– Using the Simpson’s Rules

The area under the stability curve, representing the energy a ship can exert against a
moment caused by a list, can be represented as ΣGZφ , whereby φ is reported in radians.

As long as the curve runs evenly, the calculation of the area can be estimated by regarding
the area 0 – 30° as a triangle and the area 30 – 40°, as trapezium.

Area 0 – 30° is then: 0.5 x (30°/57.3°) x GZ 30° mrad = 0.262 x GZ 30° mrad

Area 0 – 40° is then: 0.5 x (30°/57.3°) x (GZ 30° + GZ 40°) mrad = 0.873 x GZ 30°+GZ
40 °) mrad
For more accurate calculation of the area, especially if the curve is less regular, such as
ships with low freeboard and a high coaming, Simpson’s Rules can be used assuming that
the curve is of the second degree.

The substantiation of this will not be addressed here. The resulting area is a reasonably
accurate estimate.

The practical application is (see figure 1 and 2):


Issue No. 1 Page 202 of 348
ST. THERESE- MTC COLLEGES SEAMANSHIP 6
Iloilo, Philippines (Advance Trim, Stability and Stress)
STUDENT LEARNING MODULE
Revision No. 1 Effectivity date: Reviewed by: Approved by:

10 August 2020 QMR President


The curve is divided into a number of equal parts (h) on the X axis, in this case four,
resulting in an even number of vertical ordinates (Y 0-Y4).

According to Simpson’s 1st rule, the area is now:


1
x h x (Y0 + 4 x Y1 + 2 + Y2 + 4 x Y3 + Y4).
3

Y Y0 Y1 Y2 Y3 Y4
a-
s h
Figure 8.7 X-as

GZ 10° 20° 30° 40° 50° 60°

GZ10 GZ20 GZ30 GZ40

10° =
0.175 rad Figure 8.8

The calculation of the area under the curve using the 1 st rule of Simpson works as follows:

– The area from 0° - 30°, 0° - 40° and the increase from 30° - 40° must be calculated.
– There is one problem in calculating the area 0 - 30°.
Issue No. 1 Page 203 of 348
ST. THERESE- MTC COLLEGES SEAMANSHIP 6
Iloilo, Philippines (Advance Trim, Stability and Stress)
STUDENT LEARNING MODULE
Revision No. 1 Effectivity date: Reviewed by: Approved by:

10 August 2020 QMR President


– Because this part of the curve cannot be divided in an equal number of parts on the
X axis (from which the GZ value is determined), an intermediate step is used.

First, the area 0° - 10° has to be calculated and this part of the curve regarded as a triangle
(area shaded in blue).

Next, the area from 10° - 30° (in red) has to be calculated; together, they indicate the area
0° - 30°.

The area from 0° - 40° can be directly figured using Simpson’s 1 st Rule.

To reduce the area from 0° - 30°, an increase from 30° - 40° is obtained (in green)10° is
equal to 10°/57.3° = 0.175 radians.

The calculation now includes the following steps:

Area 0° - 10° = 0.5 x 0.175 x GZ1 = A

1
Area 10°-30° = x 0.175 x (GZ 10° + 4 x GZ 20° + GZ 30°) = B
3
Area 0° - 30° = A + B

1
Area 0° - 40° = x 0.175 x (GZ 0° + 4 x GZ 10° + 2 x GZ 20° + 4 x GZ 30° + GZ 40°)=C
3
Increase 30° - 40° = C – (A + B).

We need a method for determining the areas under parts of the GZ curve, and hence the
dynamical stability between two heel angles. A fairly accurate method of doing this is by
using “Simpson’s Rules.

An “ordinate” is the distance from a fixed reference line. In this case the reference line is
the horizontal heel axis. The GZ curve is then defined using ordinates at regular intervals. In
the above diagram the interval (h) is 10°. However, degrees of arc are of little use when
calculating an area, so it is necessary to convert degrees into radians (a linear measure of
arc as well as another method of measuring an angles). One radian is the equivalent of 57.3
degrees of arc, therefore to convert degrees into radians we must divide the angle in
degrees by 57.3.

10
For example 10° = radians = 0.17452 radians
57.3

Example 1:

A ship’s Curve of Statical Stability has the following ordinates:


0°=0 10°=0.20m 20°=0.52m 30°=0.95m 40°=1.22m 50°=1.16m 60°=0.75m
Issue No. 1 Page 204 of 348
ST. THERESE- MTC COLLEGES SEAMANSHIP 6
Iloilo, Philippines (Advance Trim, Stability and Stress)
STUDENT LEARNING MODULE
Revision No. 1 Effectivity date: Reviewed by: Approved by:

10 August 2020 QMR President


Construct curve of statical stability and calculate the area under the curve using the
Simpson’s Rule.

1. Plot the GZ values

Figure 8.9

2. Join all points on the curve

h = 10°

Figure 8.10
Issue No. 1 Page 205 of 348
ST. THERESE- MTC COLLEGES SEAMANSHIP 6
Iloilo, Philippines (Advance Trim, Stability and Stress)
STUDENT LEARNING MODULE
Revision No. 1 Effectivity date: Reviewed by: Approved by:

10 August 2020 QMR President

There are 6 common interval (h).

AREA UNDER
THE CURVE

Figure 8.11

Solution: Use Simpson’s First Rule.

10
Common Interval (h) = 10° = radians
57.3

1
Formula: Area = x h x (1a + 4b + 2c + 4d + 2e + 4f + 1g)
3
1
or Area = h (Sum of Products for Area)
3

Heel (°) Ordinate (m) Simpson’s Products for Area


Multiplier
0° 0 1 0
10° 0.20 4 0.80
20° 0.52 2 1.04
30° 0.95 4 3.80
Issue No. 1 Page 206 of 348
ST. THERESE- MTC COLLEGES SEAMANSHIP 6
Iloilo, Philippines (Advance Trim, Stability and Stress)
STUDENT LEARNING MODULE
Revision No. 1 Effectivity date: Reviewed by: Approved by:

10 August 2020 QMR President


40° 1.22 2 2.44
50° 1.16 4 4.64
60° 0.76 1 0.76
Sum of product 13.48
for area (Σ1)

1
Area = h (Sum of Products for Area)
3

1 10
Area = x x 13.48
3 57.3

10 x 13.48 134.8
Area = = = 0.7842 meter-radians
3 x 57.3 171.9

Do This

Activity 1 – Curve of Statical Stability

MV St Prestige completes loading with a displacement of 19,000 t and a KG corrected for


free surfaces of 8.85 m. KM for the loaded displacement is 9.48 m. (Use the extract
tabulated KN Values)

1. Calculate the GZ values.


2. Calculate the area under the curve from 0º to 40º using the Simpson’s Rule.
3. Construct curve of statical stability using a graphing paper.
Issue No. 1 Page 207 of 348
ST. THERESE- MTC COLLEGES SEAMANSHIP 6
Iloilo, Philippines (Advance Trim, Stability and Stress)
STUDENT LEARNING MODULE
Revision No. 1 Effectivity date: Reviewed by: Approved by:

10 August 2020 QMR President


Issue No. 1 Page 208 of 348
ST. THERESE- MTC COLLEGES SEAMANSHIP 6
Iloilo, Philippines (Advance Trim, Stability and Stress)
STUDENT LEARNING MODULE
Revision No. 1 Effectivity date: Reviewed by: Approved by:

10 August 2020 QMR President


Write your solution and answer in the space provided.

Write your solution and answer in the space provided.


Issue No. 1 Page 209 of 348
ST. THERESE- MTC COLLEGES SEAMANSHIP 6
Iloilo, Philippines (Advance Trim, Stability and Stress)
STUDENT LEARNING MODULE
Revision No. 1 Effectivity date: Reviewed by: Approved by:

10 August 2020 QMR President

Assessment – Curve of Statical Stability


Issue No. 1 Page 210 of 348
ST. THERESE- MTC COLLEGES SEAMANSHIP 6
Iloilo, Philippines (Advance Trim, Stability and Stress)
STUDENT LEARNING MODULE
Revision No. 1 Effectivity date: Reviewed by: Approved by:

10 August 2020 QMR President

MV St Therese completed loading with a displacement of 31,000 t and a KG corrected for


free surfaces of 7.54 m. KM for the loaded displacement is 8.10 m. (Use your extract
tabulated KN Values).

1. Calculate the GZ values.


2. Calculate the area under the curve using the Simpson’s Rule.
3. Construct curve of statical stability using a graphing paper.

Write your solution and answer in the space provided.


Issue No. 1 Page 211 of 348
ST. THERESE- MTC COLLEGES SEAMANSHIP 6
Iloilo, Philippines (Advance Trim, Stability and Stress)
STUDENT LEARNING MODULE
Revision No. 1 Effectivity date: Reviewed by: Approved by:

10 August 2020 QMR President

Lesson 9: Dynamical Stability

Learning Module 9.1: Dynamical Stability

Competence, Course Outcomes and Learning Outcomes


Issue No. 1 Page 212 of 348
ST. THERESE- MTC COLLEGES SEAMANSHIP 6
Iloilo, Philippines (Advance Trim, Stability and Stress)
STUDENT LEARNING MODULE
Revision No. 1 Effectivity date: Reviewed by: Approved by:

10 August 2020 QMR President

Competence:

Control trim, stability and stress

Course Outcome:

1. Explain the importance of maintaining stability during loading, unloading


and in-transit in various conditions.
2. Calculate the effect on trim and stability of a ship in the event of damage
to and consequent flooding of a compartment and countermeasures to be
taken.

Learning Outcomes:

At the end of the lesson, the student should be able to:

1. Explain the dynamical stability at a given angle of heel.


2. Calculate the dynamical stability at a given angle of heel.

Figure 9.1

Discussion

9.1 Definition

The ‘dynamical stability” of a ship at any particular angle of inclination may be defined as
‘the work required by the external forces (wind, waves etc.) to heel the ship to that angle’ of
heel.
Issue No. 1 Page 213 of 348
ST. THERESE- MTC COLLEGES SEAMANSHIP 6
Iloilo, Philippines (Advance Trim, Stability and Stress)
STUDENT LEARNING MODULE
Revision No. 1 Effectivity date: Reviewed by: Approved by:

10 August 2020 QMR President


9.2 Work

Suppose that we wish to push a weight across the deck of a ship. The weight will resist our
efforts to move it on account of inertia, friction with the deck, etc., and we shall have to
exert force in order to start it moving. If we then stop pushing, the friction between the
deck and the weight will soon cause the latter to stop moving, so we must continue to push
until it is the desired position. The greater the weight, the harder we must push and the
greater the distance, the longer we must push. In other words, we must do work and the
amount of work done depends on the distance we have to move the weight and the amount
of force we have to exert in order to move it. Thus, work done is equal to the force exerted,
multiplied by the distance over which it is exerted.
b

θ
W1 g L1
h1
h G g1
Z

B P R L
B1

W
Figure 9.2
9.3 Dynamical Stability

Consider a ship which is being heeled by some external force. As soon as she heels to a
small angle, her moment of statical stability will try to force her back to the upright. In order
to heel her further, sufficient force must be exerted to overcome this statical stability and
must continue to be exerted foe as long as the ship continues to heel. We can liken this
case so that of the weight mentioned in the last paragraph and say that the work done to
heel the ship to any given angle is equal to all the force exerted, over all the distance
through which the ship has heeled. This is obviously only another way of expressing the
definition of dynamical stability, which is given above.

9.4 Dynamical Stability from a Curve of Statical Stability


Figure 9.3 shows a curve of statical stability, in which the moment of statical stability (W x
GZ) is plotted against the angle of heel. The statical stability at any angle is found by the
perpendicular distance from the base line to the curve at the angle. For instance, the
moment of statical stability at 30° of heel is found by drawing the perpendicular line, AB,
and then the horizontal one, BD. The required moment is then CD – in this case about
13,000 tone-meters.
Issue No. 1 Page 214 of 348
ST. THERESE- MTC COLLEGES SEAMANSHIP 6
Iloilo, Philippines (Advance Trim, Stability and Stress)
STUDENT LEARNING MODULE
Revision No. 1 Effectivity date: Reviewed by: Approved by:

10 August 2020 QMR President


We have just said that dynamical stability is equal to all the force exerted over all the
distance through which the ship has heeled. This can be taken to mean the sum of all the
moments of statical stability, for every small angle of heel, up to the given angle.

D
B

Figure 9.3

Figure 9.4

Consider Figure 9.4, we shall see that the sum of all the moments of statical stability up to
30° of heel will be equal to the shaded area ABC. Similarly for other angle. This means that
the dynamical stability at any angle is equal to the area under the curve of statical stability
up to that angle. For this purpose, the vertical distances to the curve are always measured
in terms of statical stability and the length along the base line in terms of circular measure
(or “radians”). A radian is equal to 57.3°, so that the length along the base line becomes:

Angle of heel(°)
57.3
Issue No. 1 Page 215 of 348
ST. THERESE- MTC COLLEGES SEAMANSHIP 6
Iloilo, Philippines (Advance Trim, Stability and Stress)
STUDENT LEARNING MODULE
Revision No. 1 Effectivity date: Reviewed by: Approved by:

10 August 2020 QMR President


Thus, given a scale or curve of righting levers or moments, we can use Simpson’s Rules to
find the Dynamical Stability.

We should always use moments of statical stability (W x GZ) to obtain dynamical stability,
which would be measured in “tonne meter-radians”. For the purpose of comparison,
however, we often use righting levers (GZ), to give a quantity called “meter radians”. The
latter can be converted to actual dynamical stability by multiplying it by the ship’s
displacement (W).

Therefore, the dynamically stability to any angle of heel is found by multiplying the area
under the stability curve to that angle by the displacement.

Formula:
Dynamical stability = W x Area under the stability curve

It should be noted that in finding the area under the stability curve by the use of Simpson’s
Rules, the common interval must be expressed in radians:

57.3° = 1 radian

1
1° = radians
57.3

x
or x° = radians
57.3

Therefore, to convert degrees to radians, simply divide the number of degrees by 57.3.

Process me
Example 1

Calculate the dynamical stability at 40° heel, for a ship which displaces 6,600 tonnes and
which has the following righting levers:
Issue No. 1 Page 216 of 348
ST. THERESE- MTC COLLEGES SEAMANSHIP 6
Iloilo, Philippines (Advance Trim, Stability and Stress)
STUDENT LEARNING MODULE
Revision No. 1 Effectivity date: Reviewed by: Approved by:

10 August 2020 QMR President


Angle of 10° 20° 30° 40°
Heel

Figure 9.4
GZ (m) 0.18 0.41 0.67 0.85

10
Common interval (h) = 10° = radians
57.3

Inclination (°) GZ (m) / Simpson’s Area Product


Ordinate Multiplier
0 0.00 1 0.00
10 0.18 4 0.72
20 0.41 2 0.82
30 0.67 4 2.68
40 0.85 1 0.85
Sum of product for
area (Σ1) 5.07

1
Area = h (Sum of Products for Area)
3

1 10
Area = x x 5.07
3 57.3

10 x 5.07 50.7
Area = = = 0.2949 meter-radians
3 x 57.3 171.9

Dynamical stability = W x Area under the stability curve

= 6,600 t x 0.2949 meter-radians

Dynamical stability = 1,946.34 tonne meter-radians

Example 2

Illustrate and calculate the dynamical stability at 40° heel, for a ship which displaces 5,200
tonnes and which has the following righting levers:
Heel 0° 10° 20° 30° 40°

GZ (m) 0 0.21 0.35 0.45 0.48


Issue No. 1 Page 217 of 348
ST. THERESE- MTC COLLEGES SEAMANSHIP 6
Iloilo, Philippines (Advance Trim, Stability and Stress)
STUDENT LEARNING MODULE
Revision No. 1 Effectivity date: Reviewed by: Approved by:

10 August 2020 QMR President

Inclination (°) GZ (m) / Simpson’s Area Product


Ordinate Multiplier
0 0.00 1 0.00
10 0.21 4 0.84
20 0.35 2 0.70
30 0.45 4 1.80
40 0.48 1 0.48
Sum of product for
area (Σ1) 3.82
Solution:
1
Area under the stability curve = h (Sum of Products for Area)
3
1 10
Area = x x 3.82
3 57.3

10 x 3.82 38.2
Area = = = 0.2222 meter-radians
3 x 57.3 171.9

Dynamical stability = W x Area under the stability curve

= 5,200 t x 0.2222 meter-radians

Dynamical stability = 1,155.44 tonne meter-radians

Example 3

A box-shaped vessel 55 m x 15 m x 8 m is floating in salt water at a draft of 5.00 m forward


and aft. The GM is 0.70. Calculate the dynamical stability to 20° heel.

Formula: Displacement = L x B x Draft


B2
BM =
12d
Issue No. 1 Page 218 of 348
ST. THERESE- MTC COLLEGES SEAMANSHIP 6
Iloilo, Philippines (Advance Trim, Stability and Stress)
STUDENT LEARNING MODULE
Revision No. 1 Effectivity date: Reviewed by: Approved by:

10 August 2020 QMR President


GZ = GM x Sin θ

Solution:

1. Displacement = L x B x Draft
Displacement = 55 m x 15 m x 5.00 m x 1.025 t/m3
Displacement = 4,228 tonnes

B2
2. BM =
12d

15 m x 15 m
BM =
12 x 5 m

225 m2
BM = = 3.75 m
60 m

Note: When calculating the GZs 10° may be considered a small angle of heel, but 20° is a
large angle of heel and therefore the wall sided formula must be used to find the GZ.

3. GZ ordinates are derived below:

At 10° heel: GZ = GM x Sin θ


GZ = 0.7 m x Sin 10°
GZ = 0.7 m x 0.17365 (R.O. to 5 decimal)
GZ = 0.122 meters (R.O. to 3 decimal)
1
At 20° heel: GZ = (GM + BM tan2θ ) Sin θ
2
1
= (0.7 + x 3.75 tan2 20°) Sin 20°
2
1
= (0.7 + x 3.75 x 0.363972) Sin 20°
2
1
= (0.7 + x 3.75 x 0.13247) (0.34202)
2
0.49676
= (0.7 + ) (0.34202)
2
= (0.7 + 0.24838) (0.34202)

= (0.94838 x 0.34202)

GZ= 0.324 m (R.O. to 3 decimal)


Issue No. 1 Page 219 of 348
ST. THERESE- MTC COLLEGES SEAMANSHIP 6
Iloilo, Philippines (Advance Trim, Stability and Stress)
STUDENT LEARNING MODULE
Revision No. 1 Effectivity date: Reviewed by: Approved by:

10 August 2020 QMR President


Inclination (°) GZ (m) / Simpson’s Area Product
Ordinate Multiplier
0 0.000 1 0.00
10 0.122 4 0.488
20 0.324 1 0.324
Sum of product for
area (Σ1) 0.830
1
Area under the stability curve = h (Σ1)
3

1 10
Area = x x 0.830
3 57.3

10 x 0.830 m 8.30
Area = = = 0.04828 meter-radians
3 x 57.3 171.9

Dynamical stability = W x Area under the stability curve

= 4,228 t x 0.04828 meter-radians

Dynamical stability = 204.13 tonne meter-radians

Dynamical stability is important in ship stability for two reasons. Since it is the measure of
the work that must be done to heel a ship:

1. It is a big factor in deciding how a ship will roll, in this case the waves are doing the
work;

2. It determines the ability of a sailing ship to stand-up under sail; the pressure of the
wind on the sails supplying the work in this case. This is not of much interest to the
average merchant seaman today, but is important to yachtsmen and other who have
to deal with sailing craft.

It must always be remembered that anything which reduces statical stability will also reduce
dynamical stability. This is yet another count against our old enemy the free surface of
liquids.

Do this

Activity 1: Dynamical Stability Calculation


Issue No. 1 Page 220 of 348
ST. THERESE- MTC COLLEGES SEAMANSHIP 6
Iloilo, Philippines (Advance Trim, Stability and Stress)
STUDENT LEARNING MODULE
Revision No. 1 Effectivity date: Reviewed by: Approved by:

10 August 2020 QMR President


Calculate the dynamical stability at 40° heel of a ship which displaces 6,876 tonnes and has
the following GZs:

Heel 0° 10° 20° 30° 40°


GZ (m) 0.000 0.115 0.228 0.331 0.429

Write your solution and answer in the space provided.

Write your solution and answer in the space provided.


Issue No. 1 Page 221 of 348
ST. THERESE- MTC COLLEGES SEAMANSHIP 6
Iloilo, Philippines (Advance Trim, Stability and Stress)
STUDENT LEARNING MODULE
Revision No. 1 Effectivity date: Reviewed by: Approved by:

10 August 2020 QMR President

Do this

Activity 2: Dynamical Stability Calculation

Calculate the dynamical stability at 45° heel of a ship which displaces 5,000 tonnes and has
the following GZs:

Heel 15° 30° 45°


GZ (m) 0.205 0.482 0.611

Write your solution and answer in the space provided.


Issue No. 1 Page 222 of 348
ST. THERESE- MTC COLLEGES SEAMANSHIP 6
Iloilo, Philippines (Advance Trim, Stability and Stress)
STUDENT LEARNING MODULE
Revision No. 1 Effectivity date: Reviewed by: Approved by:

10 August 2020 QMR President

Write your solution and answer in the space provided.


Issue No. 1 Page 223 of 348
ST. THERESE- MTC COLLEGES SEAMANSHIP 6
Iloilo, Philippines (Advance Trim, Stability and Stress)
STUDENT LEARNING MODULE
Revision No. 1 Effectivity date: Reviewed by: Approved by:

10 August 2020 QMR President

Assessment – Calculation of Dynamic Stability

Calculate the area under the curve and dynamical stability at 50° heel, for a ship which
displaces 10,000 tonnes displacement and which has the following righting levers when
inclined:

Angle of Heel 0º 10° 20° 30° 40° 50º


GZ (m) 0.0 0.02 0.12 0.21 0.30 0.33

Write your solution and answer in the space provided.


Issue No. 1 Page 224 of 348
ST. THERESE- MTC COLLEGES SEAMANSHIP 6
Iloilo, Philippines (Advance Trim, Stability and Stress)
STUDENT LEARNING MODULE
Revision No. 1 Effectivity date: Reviewed by: Approved by:

10 August 2020 QMR President

Write your solution and answer in the space provided.

Lesson 10: Intact Stability Code

Learning Module 10.1: Intact Stability Code

Competence, Course Outcomes and Learning Outcomes

Competence:

Control trim, stability and stress


Issue No. 1 Page 225 of 348
ST. THERESE- MTC COLLEGES SEAMANSHIP 6
Iloilo, Philippines (Advance Trim, Stability and Stress)
STUDENT LEARNING MODULE
Revision No. 1 Effectivity date: Reviewed by: Approved by:

10 August 2020 QMR President


Course Outcome:

1. Explain the importance of maintaining stability during loading, unloading


and in-transit in various conditions.
2. Calculate the effect on trim and stability of a ship in the event of damage
to and consequent flooding of a compartment and countermeasures to be
taken.

Learning Outcomes:

At the end of the lesson, the student should be able to:

1. Explain the purpose of International Code on Intact Stability, 2008 (2000 IS Code)
2. Explain the integration of IMO knowledge on Intact Stability Code 2008 concerning
ship stability

Discussion

10.1 Introduction

This Code has been assembled to provide, in a


single document, mandatory requirements
in the introduction and in part A and
recommended provisions in part B relating to
intact stability based primarily on existing IMO
instruments. Where recommendations in this
Issue No. 1 Page 226 of 348
ST. THERESE- MTC COLLEGES SEAMANSHIP 6
Iloilo, Philippines (Advance Trim, Stability and Stress)
STUDENT LEARNING MODULE
Revision No. 1 Effectivity date: Reviewed by: Approved by:

10 August 2020 QMR President

10.2 Purpose

The purpose of the Code is to present mandatory


and recommendatory stability criteria and other
measures for ensuring the safe operation of ships,
to minimize the risk to such ships to the personnel
Figure 10.1
on board and to the environment.

This introduction and part A of the Code address the mandatory criteria and part B contains
recommendations and additional guidelines.

This Code contains intact stability criteria for the following types of ships and other marine
vehicles of 24 m in length and above, unless otherwise stated:
1. cargo ships;
2. cargo ships carrying timber deck cargoes;
3. passenger ships;
4. fishing vessels;
5. special purpose ships;
6. offshore supply vessels;
7. mobile offshore drilling units;
8. pontoons; and
9. cargo ships carrying containers on deck and containerships.

10.2.2 Administrations may impose additional requirements regarding the design aspects of
ships of novel design or ships not otherwise covered by the Code.

The IS Code includes fundamental principles such as:

 general precautions against capsizing (criteria regarding metacentric height (GM) and
righting lever (GZ);
 weather criterion (severe wind and rolling criterion);
Issue No. 1 Page 227 of 348
ST. THERESE- MTC COLLEGES SEAMANSHIP 6
Iloilo, Philippines (Advance Trim, Stability and Stress)
STUDENT LEARNING MODULE
Revision No. 1 Effectivity date: Reviewed by: Approved by:

10 August 2020 QMR President


 effect of free surfaces and icing; and watertight integrity

The IS Code also addresses related operational aspects like information for the master,
including stability and operating booklets and operational procedures in heavy weather.

10.3 General Criteria

10.3.1 Other ships stability criteria

Criteria regarding righting lever curve properties:

1. The area under the righting lever curve (GZ curve) shall not be less
than 0.055 meter-radians up to φ = 30° angle of heel and not less than 0.09 meter-
radians up to φ = 40° or the angle of down-flooding φ if this angle is less than 40°.
Additionally, the area under the righting lever curve (GZ curve) between the angles
of heel of 30° and 40° or between 30° andφ , if this angle is less than 40°, shall not
be less than 0.03 meter-radians.
2. The righting lever GZ shall be at least 0.2 m at an angle of heel equal to or greater
than 30°.
3. The maximum righting lever shall occur at an angle of heel not less than 25°. If this
is not practicable, alternative criteria, based on an equivalent level of safety, may be
applied subject to the approval of the Administration.
4. The initial metacentric height GMo shall not be less than 0.15 m.

10.3.2 Additional Criteria for Passenger Ships

5. In addition, the angle of heel on account of crowding of passengers to one side as


defined below shall not exceed 10°.

A minimum weight of 75 kg shall he assumed for each passenger except that this value may
be increased subject to the approval of the Administration. In addition, the mass and
distribution of the luggage shall be approved by the Administration.

The height of the center of gravity for passengers shall be assumed equal to:
1. 1 m above deck level for passengers standing upright. Account may be taken, if
necessary, of camber and sheer of deck; and;
2. 0.3 m above the seat in respect of seated passengers.

6. Passengers and luggage shall be considered to be in the spaces normally at their


disposal, when assessing compliance with the criteria given in 1 to 4.

In addition, the angle of heel on account of turning shall not exceed 10° when calculated
using the following formula:
Issue No. 1 Page 228 of 348
ST. THERESE- MTC COLLEGES SEAMANSHIP 6
Iloilo, Philippines (Advance Trim, Stability and Stress)
STUDENT LEARNING MODULE
Revision No. 1 Effectivity date: Reviewed by: Approved by:

10 August 2020 QMR President

where:
MR = heeling moment (kNm)
Vo = service speed (m/s)
LWL = length of ship at waterline (m)
= displacement (t)
d = mean draught (m)
KG = height of center of gravity above baseline (m)

As Per Intact Stability Code Part- A Chapter 3.1.

10.3.3 Stability Criteria for Ships Carrying Timber Deck Cargo

Scope

The provisions given hereunder apply to all ships of 24 m in length and over engaged in the
carriage of timber deck cargoes. Ships that are provided with, and make use of, their timber
load line shall also comply with the requirements of regulations 41 to 45 of the 1966 Load
Lines Convention and the Protocol of 1988 relating thereto.
Alternative stability criteria

For ships loaded with timber deck cargoes and provided that the cargo extends
longitudinally between superstructures (where there is no limiting superstructure at the after
end, the timber deck cargo shall extend at least to the after end of the aftermost hatchway)
transversely for the full beam of ship, after due allowance for a rounded gunwale, not
exceeding 4% of the breadth of the ship and/or securing the supporting uprights and which
remains securely fixed at large angles of heel may be:

1. The area under the righting lever curve (GZ curve) shall not be less than 0.08 meter-
radians up to φ = 40° or the angle of flooding if this angle is less than40°.
2. The maximum value of the righting lever (GZ) shall be at least 0.25 m.
3. At all times during a voyage, the metacentric height GM shall not be less than 0.1 m,
taking into account the absorption of water by the deck cargo and/or ice accretion
on the exposed surfaces.
4. When determining the ability of the ship to withstand the combined effects of beam
wind and rolling according to Heeling Weather Criterion (IS Code Chap.2.3, the 16°
limiting angle of heel under action of steady wind shall be complied with, but the
additional criterion of 80% of the angle of deck edge immersion may be ignored.
5. The cargoes extends longitudinally between superstructures.

10.3.4 Stability Criteria for Grain in Bulk


Issue No. 1 Page 229 of 348
ST. THERESE- MTC COLLEGES SEAMANSHIP 6
Iloilo, Philippines (Advance Trim, Stability and Stress)
STUDENT LEARNING MODULE
Revision No. 1 Effectivity date: Reviewed by: Approved by:

10 August 2020 QMR President


1. The angle of heel due to the shift of the grain is to be not greater than 12 degrees or
the angle at which the deck cargo is immersed, whichever is the lesser.
2. The net or residual area between the heeling arm curve and the righting arm curve
up to the angle of heel of minimum difference between the ordinates of two curves,
or 40 degrees or the angle of flooding ( θ F), whichever is the least, is in all conditions
of loading to be not less than 0.075 meter radians.
3. The initial metacentric height (GM O), after correction for free surface effects of
liquids in tanks, is to be not less than 0.30 m.
4. After loading, the master shall ensure that the ship is upright before proceeding to
sea.
5. Master must demonstrate the ability of the ship at all stages of any voyage to comply
with the stability criteria required by this section.

10.3.5 Containerships greater than 100 m

Application

These requirements apply to containerships greater than 100 m in length as defined in


section 2(Definitions) of the Introduction. They may also be applied to other cargo ships in
this length range with considerable flare or large water plane areas. The Administration may
apply the following criteria instead of those in part A, 2.2.

Intact stability

1. The area under the righting lever curve (GZ curve) should not be less than 0.009/C
meter-radians up toφ = 30° angle of heel, and not less than 0.016/C meter-radians
up to φ = 40° or the angle of flooding φ f(as defined in part A, 2.2 of the IS Code) if
this angle is less than 40°.
2. Additionally, the area under the righting lever curve (GZ curve) between the angles
of heel of 30° and 40° or between 30° and φ f, if this angle is less than 40°, should
not be less than 0.006/C meter-radians.
3. The righting lever GZ should be at least 0.033/C m at an angle of heel equal or
greater than 30°.
4. The maximum righting lever GZ should be at least 0.042/C m.
5. The total area under the righting lever curve (GZ curve) up to the angle of flooding φ
f should not be less than 0.029/C meter-radians.

In the criteria the form factor should be calculated using the formula and figure 10.2:

where:
d = mean draught (m)
D' = moulded depth of the ship, corrected for defined parts of volumes within the
Issue No. 1 Page 230 of 348
ST. THERESE- MTC COLLEGES SEAMANSHIP 6
Iloilo, Philippines (Advance Trim, Stability and Stress)
STUDENT LEARNING MODULE
Revision No. 1 Effectivity date: Reviewed by: Approved by:

10 August 2020 QMR President


hatch coamings according to the formula:
D = moulded depth of the ship (m);
BD = moulded breadth of the ship (m);
KG = height of the center of mass above base, corrected for free surface effect, not
be taken as less than d (m);
CB = block coefficient;
CW = water plane coefficient;
lH = length of each hatch coaming within Ll4 forward and aft from amidships (m)
(see figure 10.2);
b = mean width of hatch coamings within Ll4 forward and aft from amidships (m)
(see figure 10.2);
h = mean height of hatch coamings within Ll4 forward and aft from amidships (m)
(see figure 10.1);
L = length of the ship (m);
B = breadth of the ship on the waterline (m);
Bm= breadth of the ship on the waterline at half mean draught (m).

b
IH

B/4
D1
D
d KG
d/2

BM
B
BD

lH lH lH

L/4 L/4

L
Figure 10.2

The shaded areas in figure 10.1represent partial volumes within the hatch coamings
considered contributing to resistance against capsizing at large heeling angles when the ship
is on a wave crest 2,3,2,7. The use of electronic loading and stability instrument is
encouraged in determining the ship's trim and stability during different operational
conditions.
Issue No. 1 Page 231 of 348
ST. THERESE- MTC COLLEGES SEAMANSHIP 6
Iloilo, Philippines (Advance Trim, Stability and Stress)
STUDENT LEARNING MODULE
Revision No. 1 Effectivity date: Reviewed by: Approved by:

10 August 2020 QMR President

Checkpoint
Activity – Identification. Intact Stability Code 2008

Directions: Give the corresponding Intact Stability Criteria requirements for different type of
ships based on Intact Stability Code 2008.

_______1. The metacentric height (GM) intact stability criteria requirement for ships
carrying timber deck cargoes
_______2. The initial metacentric height (GM) intact stability criteria requirement for
container ships greater than 100 meters.
_______3. The initial metacentric height (GM) intact stability criteria requirement for
general cargo ships.
_______4. The righting lever GZ intact stability criteria requirement in meters for general
cargo ships.
_______5. The righting lever GZ intact stability criteria requirement in meters for container
ships greater than 100 meters.
_______6. The limiting angle of heel under action of steady wind requirement for ships
carrying timber deck cargoes.
_______7. The maximum righting lever GZ requirement for ships carrying timber deck
cargoes.
_______8. The maximum righting lever GZ requirement in meters for container ships
greater than 100 meters.
_______9. The maximum righting lever GZ requirement in meters for ships carrying timber
deck cargoes.
_______10. The area under GZ curve requirement for ships carrying timber deck cargoes
in meter radians.

Assessment – Intact Stability Code

Multiple Choice: Select the correct answer from the given choices.
Issue No. 1 Page 232 of 348
ST. THERESE- MTC COLLEGES SEAMANSHIP 6
Iloilo, Philippines (Advance Trim, Stability and Stress)
STUDENT LEARNING MODULE
Revision No. 1 Effectivity date: Reviewed by: Approved by:

10 August 2020 QMR President


1. When was the International Code on Intact Stability, 2008 adopted?
A. 02 December 2008 C. 04 December 2008
B. 04 December 2008 D. 06 December 2008
2. Under what resolution of the IMO can you find information about intact stability
criteria of your vessel?
A. Resolution MSC 75 (69) C. Resolution MSC 269 (85)
B. Resolution MSC 267 (85) D. Resolution MSC 270 (85)
3. When was the International Code for Intact Stability, 2008 entered into force?
A. 01 May 2010 C. 01 July 2010
B. 01 June 2010 D. 01 August 2010

Intact Stability Criteria for General Cargo, Tankers and Passenger Ships

4. What is the general intact stability criteria requirement on the initial GM or


metacentric height?
A. Not less than 0.10 meters C. Not more than 0.10 meters
B. Not less than 0.15 meters D. Not more than 0.15 meters
5. What is the general intact stability criteria requirement on the righting lever GZ?
A. Shall at least 0.10 meters C. Shall at least 0.20 meters
B. Shall at least 0.15 meters D. Shall at least 0.25 meters
6. What is the general intact stability criteria requirement on the righting lever GZ angle
of heel?
A. ≤ 20 degrees C. ≥ 20 degrees
B. ≤ 30 degrees D. ≥ 30 degrees
7. What is the general intact stability criteria requirement on the maximum righting
lever should occur?
A. At heel > 30 preferably but not less than 25 degrees
B. At heel > 30 preferably but not less than 20 degrees
C. At heel > 25 preferably but not less than 30 degrees
D. At heel > 25 preferably but not less than 25 degrees
8. What is the general intact stability criteria requirement on the Area of the GZ curve?
A. Should be at least 0.055 m radian up to 10 degrees
B. Should be at least 0.055 m radian up to 20 degrees
C. Should be at least 0.055 m radian up to 30 degrees
D. Should be at least 0.055 m radian up to 40 degrees

9. What is the general intact stability criteria requirement on the Area of the GZ curve?
A. Should be at least 0.090 m radian up to 10 degrees
B. Should be at least 0.090 m radian up to 20 degrees
C. Should be at least 0.090 m radian up to 30 degrees
D. Should be at least 0.090 m radian up to 40 degrees
10. What is the general intact stability criteria requirement on the Area of the GZ curve?
Issue No. 1 Page 233 of 348
ST. THERESE- MTC COLLEGES SEAMANSHIP 6
Iloilo, Philippines (Advance Trim, Stability and Stress)
STUDENT LEARNING MODULE
Revision No. 1 Effectivity date: Reviewed by: Approved by:

10 August 2020 QMR President


A. Should be at least 0.02 m radian between 30 and 40 or between 30 and
angle of down flooding.
B. Should be at least 0.03 m radian between 30 and 40 or between 30 and
angle of down flooding.
C. Should be at least 0.04 m radian between 30 and 40 or between 30 and
angle of down flooding.
D. Should be at least 0.05 m radian between 30 and 40 or between 30 and
angle of down flooding.

Intact Stability Criteria for Container Ship greater than 100 meters

11. What is the intact stability criteria requirement on the initial GM or metacentric
height?
A. Not less than 0.10 meters C. Not more than 0.10 meters
B. Not less than 0.15 meters D. Not more than 0.15 meters
12. What is the intact stability criteria requirement on the righting lever GZ?
A. Shall at least 0.030 meters C. Shall at least 0.032 meters
B. Shall at least 0.031 meters D. Shall at least 0.033 meters
13. What is the intact stability criteria requirement on the righting lever GZ angle of
heel?
A. ≥ 20 degrees C. ≤ 20 degrees
B. ≥ 30 degrees D. ≤ 30 degrees
14. What is the intact stability criteria requirement on the maximum righting lever should
occur?
A. At least 0.042
B. At least 0.043
C. At least 0.044
D. At least 0.045
15. What is the intact stability criteria requirement on the Area under the GZ curve?
A. Not less than 0.009 m radian up to 10 degrees angle of heel
B. Not less than 0.009 m radian up to 20 degrees angle of heel
C. Not less than 0.009 m radian up to 30 degrees angle of heel
D. Not less than 0.009 m radian up to 40 degrees angle of heel

16. What is the intact stability criteria requirement on the Area of the GZ curve?
A. Not less than 0.016 m radian up to 10 degrees angle of heel
B. Not less than 0.016 m radian up to 20 degrees angle of heel
C. Not less than 0.016 m radian up to 30 degrees angle of heel
D. Not less than 0.016 m radian up to 40 degrees angle of heel
17. What is the intact stability criteria requirement on the Area of the GZ curve?
Issue No. 1 Page 234 of 348
ST. THERESE- MTC COLLEGES SEAMANSHIP 6
Iloilo, Philippines (Advance Trim, Stability and Stress)
STUDENT LEARNING MODULE
Revision No. 1 Effectivity date: Reviewed by: Approved by:

10 August 2020 QMR President


A. Not less than 0.006 m radian between 20 degrees and 30 degrees angle of
heel
B. Not less than 0.006 m radian between 30 degrees and 40 degrees angle of
heel
C. Not less than 0.006 m radian between 40 degrees and 50 degrees angle of
heel
D. Not less than 0.006 m radian between 25 degrees and 35 degrees angle of
heel
18. What is the intact stability criteria requirement on the total area of the GZ curve?
A. Up to the angle of flooding should not be less than 0.027 m radians
B. Up to the angle of flooding should not be less than 0.028 m radians
C. Up to the angle of flooding should not be less than 0.029 m radians
D. Up to the angle of flooding should not be less than 0.030 m radians

Intact Stability Criteria for Ships Carrying Timber Deck Cargoes

19. What is the intact stability criteria requirement on the GM or metacentric height
during the voyage and at all times?
A. Not less than 0.10 meters C. Not more than 0.10 meters
B. Not less than 0.15 meters D. Not more than 0.15 meters
20. What is the intact stability criteria requirement on the maximum righting lever should
occur?
A. Shall at least 0.15 meters C. Shall at least 0.35 meters
B. Shall at least 0.25 meters D. Shall at least 0.40 meters
21. What is the intact stability criteria requirement on the Area under the GZ curve?
A. Not less than 0.04 meters C. Not more than 0.08 meters
B. Not less than 0.06 meters D. Not more than 0.10 meters
22. What is the intact stability criteria requirement on the limiting angle of heel under
action of steady wind?
A. 8 degrees C. 25 degrees
B. 16 degrees D. 30 degrees

Intact Stability Criteria for Cargo Ships Carrying Grain in Bulk

23. What is the intact stability criteria requirement on the initial GM or metacentric
height?
A. Not less than 0.10 meters C. Not more than 0.30 meters
Issue No. 1 Page 235 of 348
ST. THERESE- MTC COLLEGES SEAMANSHIP 6
Iloilo, Philippines (Advance Trim, Stability and Stress)
STUDENT LEARNING MODULE
Revision No. 1 Effectivity date: Reviewed by: Approved by:

10 August 2020 QMR President


B. Not less than 0.20 meters D. Not more than 0.40 meters
24. What is the intact stability criteria requirement on the angle of heel due to shift of
grain?
A. Not be greater than 8 degrees
B. Not be greater than 10 degrees
C. Not be greater than 12 degrees
D. Not be greater than 14 degrees
25. What is the intact stability criteria requirement on the residual area between the
heeling arm curve and righting arm curve?
A. Shall not be less than 0.065 m radians up to 20º angle of heel or angle of
flooding
B. Shall not be less than 0.065 m radians up to 40º angle of heel or angle of
flooding
C. Shall not be less than 0.075 m radians up to 20º angle of heel or angle of
flooding
D. Shall not be less than 0.075 m radians up to 40º angle of heel or angle of
flooding

Oral Assessment (Midterm)

Explain the following and make a video clip wearing upper uniform.

1. Rolling period
2. Inclining test
3. Test correction
4. Stability criteria and requirement

Oral presentation on the chosen research topic directions (this is a video oral presentation):

1. In this activity, you will work a one person only.


2. On a paper, please write down what you think is your idea regarding your chosen
research topic. Please accomplish this task in 3 minutes.
3. When done, please post your output on Power Point Slides and prepare to present.
4. Each person will be given at least 2 minutes to present their outputs.
5. During the presentation, please explain briefly why you think you listed items that
constitute the terms in your chosen research topic.

RUBRICS FOR GRADING ORAL ASSESSMENT

CRITERIA SCORE
Content (40%)
Issue No. 1 Page 236 of 348
ST. THERESE- MTC COLLEGES SEAMANSHIP 6
Iloilo, Philippines (Advance Trim, Stability and Stress)
STUDENT LEARNING MODULE
Revision No. 1 Effectivity date: Reviewed by: Approved by:

10 August 2020 QMR President


 The contetnt clearly incorporates salient points discussed about the
topic
Creativity and overall presentation (30%)
 The presentation showcases learner’s creativity and originality
Other elements (15%)
 The group effectively use gestures, eye contact, movements, and
facial expressions to convey the message.
 The speaker speaks audibly and clearly.
 The speaker uses props, and costumes appropriately and effectively.
Mastery (15%)
 The pair has mastered the concepts and delivered it well.

Research/Output

Research your OUTPUT for the following topics and explain it:

1. Heeling moment
2. Rolling ship (basic principles)
3. Synchronization
4. Centripetal force

Power Point Presentation, make a 5 minutes presentation of the chosen research topic. The
format of the slides must be:

- Slide 1: Title of the chosen research topic and members


– Slide 2: Justification for choosing the topic
– Slide 3: Existing problems, issues, gaps relevant to the chosen topic
- Font Size and Style: Title: Arial 36-40” Body: 24-28”
- Animation and Design: Free Style
- Follow 7 x 7 rule (7 words and 7 lines in a slide)

RUBRIC FOR GRADING POWER POINT PRESENTATION OF A CHOSEN RESEARCH


TOPIC

CRITERIA SCORE
CONTENT (40%)
Issue No. 1 Page 237 of 348
ST. THERESE- MTC COLLEGES SEAMANSHIP 6
Iloilo, Philippines (Advance Trim, Stability and Stress)
STUDENT LEARNING MODULE
Revision No. 1 Effectivity date: Reviewed by: Approved by:

10 August 2020 QMR President


 The content clearly incorporates salient points discussed about
the topic.
SLIDE CREATION (15%)
 Presentation flows well and logically. Presentation reflects
extensive use of tools in a creative way.
SLIDE TRANSITIONS (15%)
 Transitions are smooth. Transitions enhance the presentation
PICTURES, CLIP ART BACKGROUND (15%)
 Images are appropriate. Layout is pleasing to the eye.
MECHANICS (15%
 No spelling errors. No grammar errors. Text is in authors’ own
words

RESEARCH WORK

Submit a well-written literature review of your research topic and explain it:

1. Righting moment and Heeling couple


2. Dry docking and grounding (basic requirements)
3. Up thrust
4. Righting moment

Submit a well-written literature review of your research topic. The table provided below is a
guide in making your literature review. You may add extra pages.

1. Draft article summary paragraph: 3-4 paragraphs of these will become your short
literature review.
2. Create an order for your paragraph summaries (try this with just 3-4 paragraphs that
relate). You have now created a draft of a short literature review.
3. Revise your short literature review to include paragraph introduction, body, and
conclusions/synthesis.

ARTICLE SUMMARY TABLE (Related Literature and Studies)

1 2
 Reference
 Aim/Objective/Scope
 Participants and sampling
Issue No. 1 Page 238 of 348
ST. THERESE- MTC COLLEGES SEAMANSHIP 6
Iloilo, Philippines (Advance Trim, Stability and Stress)
STUDENT LEARNING MODULE
Revision No. 1 Effectivity date: Reviewed by: Approved by:

10 August 2020 QMR President


 Context and framework
 Results or findings
 Implications Practice to Research
 Your Comments

Rubric for grading research work on reviewing literature

CRITERIA SCORE
INTRODUCTION (10%)
 Clear overview of paper, demonstrates importance of topic.
BODY (40%)
 Balanced viewpoint: Objective, balanced view from various
perspectives. Coherent theme: Each cited study related to
the topic and to other studies.
 Depth and breadth of research: Variety of studies and
attention to detail about the topic.
 Analysis: Collection of studies analyzed for differences and
commonalities about the topic.
CONCLUSION AND SYNTHESIS (10%)
 Information synthesized and brought to a logical
conclusion.
ORGANIZATION AND ALIGNMENT (15%)
 Organization and Alignment: Information logically
organized with good flow. Issues threaded throughout
paper.
MECHANICS, APA, REFERENCES (25%)
 Correct spelling, punctuation, sentence structure, word
usage.
 Correct use of APA in body of paper References correctly.
typed, appropriate number and quality
Issue No. 1 Page 239 of 348
ST. THERESE- MTC COLLEGES SEAMANSHIP 6
Iloilo, Philippines (Advance Trim, Stability and Stress)
STUDENT LEARNING MODULE
Revision No. 1 Effectivity date: Reviewed by: Approved by:

10 August 2020 QMR President

Lesson 11: Rolling Period

Learning Module 11.1: Rolling Period

Competence, Course Outcomes and Learning Outcomes

Competence:

Control trim, stability and stress

Course Outcome:

1. Explain the importance of maintaining stability during loading, unloading


and in-transit in various conditions.
2. Calculate the effect on trim and stability of a ship in the event of damage
to and consequent flooding of a compartment and countermeasures to be
taken.

Learning Outcomes:

At the end of the lesson, the student should be able to:

1. Discuss the factors and conditions of rolling period of a ship.


2. Calculate the following:
Issue No. 1 Page 240 of 348
ST. THERESE- MTC COLLEGES SEAMANSHIP 6
Iloilo, Philippines (Advance Trim, Stability and Stress)
STUDENT LEARNING MODULE
Revision No. 1 Effectivity date: Reviewed by: Approved by:

10 August 2020 QMR President


– new displacement in tones (W);
– original radius of gyration (K);
– new displacement and new GM (W2 and GM2);
– new mass moment of inertia (I2);
– new radius of gyration (K2);
– new period of roll (T2).

Discussion

11.1 The Period of a Ship

This is the time taken by a ship to roll from one side to the other and back again. When the
period is exactly the same for every roll, the rolling is termed “isochronous”. It is often
assumed that isochronous rolling occurs in every ship for any angle of roll, but this is not
correct. We may accept the following general rules:

a. Different ships have different periods of roll.


b. The same ship will have a different period for different conditions of loading.
c. The same ship will have a longer period when she is tender than when she is stiff.
d. “Winging out” weights will increase the period, all other things being equal.
e. Rolling is isochronous for small angles of roll, up to about ten degrees, but the
period increases slightly for larger angles.

11.3 Synchronism

This is said to occur when the ship’s period of roll is the same as the apparent period of the
waves. When it occurs, the waves gives the ship a “push” each time she rolls, in the
direction in which she is rolling, causing her to roll more and more heavily. In theory, it
would continue until she capsized, but this does not happen in practice because of certain
resistances.

11.3 ‘Still Water’ Rolling

Rolling is a simple harmonic motion. A vessel can be made to roll under wave action. Rolling
period is an important parameter in ship handling too. It is important to be able to measure
the rolling period of a ship.
Issue No. 1 Page 241 of 348
ST. THERESE- MTC COLLEGES SEAMANSHIP 6
Iloilo, Philippines (Advance Trim, Stability and Stress)
STUDENT LEARNING MODULE
Revision No. 1 Effectivity date: Reviewed by: Approved by:

10 August 2020 QMR President

Figure 11.1

The stop watch is started when the roll has reached the maximum heel on any side, port
side or starboard side. The watch is stopped after the vessel rolls to the opposite side and
then returns to the maximum heel on the side from which time measurement was started.

The rolling characteristics of a ship are governed by:


1. the GM and;
2. the distribution of the weight components of the ship’s structure and deadweight
items with respect to the rolling axis, which is assumed to be at the center of gravity
of the ship (G).

The roll period (T) in seconds is the time taken for the ship to complete one complete
oscillation i.e. the time it takes for the ship to roll from one side back through the upright to
the extent of it’s roll on the other side and back again.

One complete oscillation

Roll Period (T) in seconds

Figure 11.2: Ship rolling

The amplitude on the roll is defined as the extent of the roll in degrees.

11.4 Unrestricted Rolling

Unrestricted rolling is a hypothetical condition that assumes the ship is rolling in a


‘frictionless’ fluid such that the amplitude of the oscillations remains constant. In reality the
amplitude of one roll decreases with each half-roll cycle until eventually the ship settles in
the upright condition in still water (being due to water and air resistance).

The energy in this assumed un-damped roll is totally potential energy at the limit of the roll,
and totally kinetic energy the instant the ship passes through the upright position. At any
intermediate position within the roll cycle the energy will be partly kinetic and partly
Issue No. 1 Page 242 of 348
ST. THERESE- MTC COLLEGES SEAMANSHIP 6
Iloilo, Philippines (Advance Trim, Stability and Stress)
STUDENT LEARNING MODULE
Revision No. 1 Effectivity date: Reviewed by: Approved by:

10 August 2020 QMR President


potential; but the sum of these two quantities will remain constant. The ship will act in a
similar manner to a pendulum under the same conditions.

Figure 11.3

At the extent of the roll, (A) and (C) in Figure 11.3, the ship will possess all potential energy
(PE) that will be equivalent to: PE at extent of roll to θ ° = Area under righting moment
curve from 0° to θ ° .

As the ship passes through the upright position, (B) in Figure 11.3, the GZ and hence
righting moment will be zero and potential energy at this point in the roll will be zero.
However, the ship will possess all kinetic energy by virtue of its motion as it passes through
the upright position.

Under the circumstance described the roll period (T) is approximated by the formula:

T (secs) =

where: ‘I’ is the moment of inertia of the ship’s structure and all deadweight components
about the rolling axis (G);
‘W’ is the ship’s displacement in tones;
‘g’ is the acceleration due to gravity (9.81 m/s2); and
‘GM’ is the ship’s effective metacentric height.

The value of I is found by: I = Σwr2

where ‘w’ is the weight of each structural component of the ship and each item of
deadweight and ‘r’ is the distance that each component is from the rolling axis, assumed to
be at G.

The quantity Σwr2 can be written as Wk2 where:

W is the ship’s displacement and


k is the radius of gyration.

11.5 Radius of Gyration


Issue No. 1 Page 243 of 348
ST. THERESE- MTC COLLEGES SEAMANSHIP 6
Iloilo, Philippines (Advance Trim, Stability and Stress)
STUDENT LEARNING MODULE
Revision No. 1 Effectivity date: Reviewed by: Approved by:

10 August 2020 QMR President


The radius of gyration might be a difficult concept to understand but may be loosely defined
as being for a ship the distance from the center of gravity (or rolling axis) at which the total
weight (W) would have to be concentrated in order to give the ship the same moment of
inertia as it actually has.

For any particular ship the radius of gyration can be changed by altering the distribution of
deadweight about the rolling axis

Figure 11.4

Moving weight to the sides of the ship away from the rolling axis (G) increases the radius of
gyration (k) and increases the ship’s rolling period.

11.4 Rolling Period Formula

Because I = Σwr2, the formula for the still water rolling period becomes:

T (secs) =

which simplifies to:


2 πk
T (secs) =
√ g xGM
This formula is substantially correct for small angles of heel where the formula GZ = GM x
Sin θ is valid.

The formula shows that for a constant value of I (or k) the rolling period is inversely
proportional to the square root of the GM (as long as GM is positive).

As GM increases the rolling period reduces.

The formula also shows that the rolling period is directly proportional to the radius of
gyration (k), that is, ask increases (by ‘winging out’ weights say), the rolling periods
increases.

Process me
Example 1

Calculate the natural rolling period of a ship for which k is 4.5 m and GM is 1.15 m.
Issue No. 1 Page 244 of 348
ST. THERESE- MTC COLLEGES SEAMANSHIP 6
Iloilo, Philippines (Advance Trim, Stability and Stress)
STUDENT LEARNING MODULE
Revision No. 1 Effectivity date: Reviewed by: Approved by:

10 August 2020 QMR President


Solution:
2 πk
T (secs) = where: ‘g’ is the acceleration due to gravity 9.81 m/s 2
√ g xGM
2 π x 4.5
=
√ 9.81 x 1.15
28.274
=
√11.2815
28.274
T (secs) = = 8.4 seconds
3.36

Example 2

A ship displaces 15,500 tonnes, has GM 1.25 m and a roll period of 12 seconds. The ship
loads 150 tonnes in position 6.3 m above G.

2 πk
Formula: T (secs) =
√ g xGM
Solution:
1. Calculate the initial radius of gyration (k).

T x √ g x GM 12 x √ 9.81 x 1.25 12 x √12.2625


k= = ==
2π 2π 2π
12 x 3.502
k=

42.024 42.024
k= = = 6.688 m
2π 6.2832
2. Calculate the new value of I and new GM:
Original I = Wk2 = W x K2

I = Wk2 = 15,500 x 6.6882 = 693,305 m2


I value of added weight = wd2 = 150 x 6.32 = + 5,954 m2
Total = 699,259 m2

wxd 150 t x 6.3 m 945


GGV =
W +w
= =
15500t +150 t 15650
= 0.0604 m

Correction to I for shift of G = W x (GGV)2 = 15,650 m x 0.06042 = 57 m4


therefore:
Find the value of I about the new G = 699,259 m4 – 57m4 = 699,202 m4
Issue No. 1 Page 245 of 348
ST. THERESE- MTC COLLEGES SEAMANSHIP 6
Iloilo, Philippines (Advance Trim, Stability and Stress)
STUDENT LEARNING MODULE
Revision No. 1 Effectivity date: Reviewed by: Approved by:

10 August 2020 QMR President


New GM = Initial GM – GGV
New GM = 1.25 m – 0.0604 t-m = 1.190 m

3. Calculate the new value of k:


I 699,202m4
I = Wk2∴ k2 =
W
= = 44.677 m
15650 t
New k = √ 44.677
New k =6.684

4. Calculate the new roll period:


2 πk
New T (secs) =
√ g xGM
2 π x 6.684 41.997 41.997
= = =
√ 9.81 x 1.190 √ 11.674 3.147
New T =13.3 seconds = 13 seconds

Example 3

A ship of 10,500 tonnes displacement has GM 0.50 m. The period of roll in still water is 20
seconds. Calculate the new period of roll if a mass of 95 tonnes is discharged from a
position 16 m above the center of gravity.

Procedures:
1. Calculate the new displacement in tonnes
2. Estimate the original radius of gyration (K)
3. Evaluate the new displacement and new GM (W 2 and GM2)
4. Calculate the new mass moment of inertia (I2)
5. Calculate the new radius of gyration (K2)
6. Evaluate the new period of roll (T2)

Solution:
1. Calculate the new displacement in tones (W2)

W2 = WO – w
W2 = 10,500 tonnes – 95 tonnes
W2 = 10,405 tonnes

2. Estimate the initial/original radius of gyration (k).


Issue No. 1 Page 246 of 348
ST. THERESE- MTC COLLEGES SEAMANSHIP 6
Iloilo, Philippines (Advance Trim, Stability and Stress)
STUDENT LEARNING MODULE
Revision No. 1 Effectivity date: Reviewed by: Approved by:

10 August 2020 QMR President


T x √ g x GM 20 x √ 9.81 x 0.50 20 x √ 4.905
k= = =
2π 2π 2π
20 x 2.215
k=

44.3 44.3
k= = = 7.051 m
2 π 6.2832

3. Calculate the new value of I and new GM:

Original I = Wk2 = W2 x K2

IO = Wk2 = 10,405 x 7.0512 = 517,301 m2


New I value of discharged weight = wd = 95 x 16 = - 24,320 m2
2 2

Total = 429,981 m2
wxd 95 t x 16 m 1520t−m
GGV = = =
W +w 10500t−95t 10,405 t = 0.146 m

Correction to I for shift of G = W x (GGV)2 = 10,405 m x 0.1462 = 222 m4

Therefore:
Calculate final value of I about the new G = 429,981 m4 – 222 m4 = 429,759 m4

New GM (GM2) = Initial GM + GGV


New GM (GM2) = 0.50 m + 0.146 m =0.646 m

4. Calculate the new value of radius of gyration (k2):

4
I
I1= Wk2∴k2 = = 429,981 m = 41.32 m
W 10,405t
New k (k2) = √ 41.32
New k (k2) = 6.43 m

5. Evaluate the new period of roll (T2):


2 πk
New T (secs) =
√ g x GM
2 π x 6.43 40.4
= =
√ 9.81 x 0.646 √6.337
40.4
=
2.517
New T =16.05 seconds = 16 seconds
Issue No. 1 Page 247 of 348
ST. THERESE- MTC COLLEGES SEAMANSHIP 6
Iloilo, Philippines (Advance Trim, Stability and Stress)
STUDENT LEARNING MODULE
Revision No. 1 Effectivity date: Reviewed by: Approved by:

10 August 2020 QMR President

Do this

Activity 1 – Rolling Period Problems

Calculate the natural rolling period of a ship when the radius of gyration is 6.5 m and
metacentric height is 0.70 m.

Write your solution and answer in the space provided.


Issue No. 1 Page 248 of 348
ST. THERESE- MTC COLLEGES SEAMANSHIP 6
Iloilo, Philippines (Advance Trim, Stability and Stress)
STUDENT LEARNING MODULE
Revision No. 1 Effectivity date: Reviewed by: Approved by:

10 August 2020 QMR President

Do this

Activity 2 – Rolling Period Problems

A ship displaces 15,500 tonnes, has GM 1.35 m and a roll period of 16 seconds. The ship
loads 80 tonnes in position 6.5 m above G. Calculate the following:

1. Initial radius of gyration (k);


2. New value of I;
3. New GM:
4. New value of k:
5. New roll period:
Write your solution and answer in the space provided.

Write your solution and answer in the space provided.


Issue No. 1 Page 249 of 348
ST. THERESE- MTC COLLEGES SEAMANSHIP 6
Iloilo, Philippines (Advance Trim, Stability and Stress)
STUDENT LEARNING MODULE
Revision No. 1 Effectivity date: Reviewed by: Approved by:

10 August 2020 QMR President

Assessment – Calculation of Rolling Period


Issue No. 1 Page 250 of 348
ST. THERESE- MTC COLLEGES SEAMANSHIP 6
Iloilo, Philippines (Advance Trim, Stability and Stress)
STUDENT LEARNING MODULE
Revision No. 1 Effectivity date: Reviewed by: Approved by:

10 August 2020 QMR President


A ship of 20,000 tonnes displacement has GM 0.75 m. The period of roll in still water is 18
seconds. Calculate the new period of roll if a mass of 150 tonnes is discharged from a
position 18 m above the center of gravity. Calculate the following:
1. New displacement in tonnes (W2);
2. Initial radius of gyration (K);
3. New Moment of Inertia (I);
4. New GM;
5. New radius of gyration (K2);
6. New Period of Roll (T2).

Write your solution and answer in the space provided.

Write your solution and answer in the space provided.


Issue No. 1 Page 251 of 348
ST. THERESE- MTC COLLEGES SEAMANSHIP 6
Iloilo, Philippines (Advance Trim, Stability and Stress)
STUDENT LEARNING MODULE
Revision No. 1 Effectivity date: Reviewed by: Approved by:

10 August 2020 QMR President


Issue No. 1 Page 252 of 348
ST. THERESE- MTC COLLEGES SEAMANSHIP 6
Iloilo, Philippines (Advance Trim, Stability and Stress)
STUDENT LEARNING MODULE
Revision No. 1 Effectivity date: Reviewed by: Approved by:

10 August 2020 QMR President

Lesson 12: Bilging and Permeability

Learning Module 12.1: Bilging and Permeability

Competence, Course Outcomes and Learning Outcomes

Competence:

Control trim, stability and stress

Course Outcome:

1. Explain the importance of maintaining stability during loading, unloading


and in-transit in various conditions.
2. Calculate the effect on trim and stability of a ship in the event of damage to
and consequent flooding of a compartment and countermeasures to be
taken.

Learning Outcomes:

At the end of the lesson, the student should be able to:

1. Discuss the bilging and permeability in relation to trim and stability of the ship.
2. Solve problems involving bilging and permeability according to the following:
– Bodily sinkage
– Change of trim
– Final drafts
Issue No. 1 Page 253 of 348
ST. THERESE- MTC COLLEGES SEAMANSHIP 6
Iloilo, Philippines (Advance Trim, Stability and Stress)
STUDENT LEARNING MODULE
Revision No. 1 Effectivity date: Reviewed by: Approved by:

10 August 2020 QMR President

Discussion

12.1 Bilging

Bilging is said to occur when ingress of water


takes place into the vessel from a point below
the waterline, such that the water is free to flow
in and out of the vessel. If the water enters the
vessel from above the waterline then it will not
be able to flow out of the vessel, in which case
it is called flooding and not bilging.

How stability is affected? Figure 12.1

 Bilging is said to occur when ingress of water takes place into the vessel from a point
below the waterline, such that the water is free to flow in and out of the vessel. If the
water enters the vessel from above the waterline then it will not be able to flow out of
the vessel, in which case it is called flooding and not bilging.
 After bilging, water level in the bilged compartment will be same as water level
outside the vessel. 
 However, if the bilged compartment is a DB tank / deep tank, located at the bottom of
the vessel, and it is fitted with a tank-top / watertight flat which is located below the
outside water level, then water in the compartment will rise only up to the tank top /
watertight flat. 
 Because the water is free to flow in and out of the bilged compartment, it is not part
of the ship and hence there is no addition of weight i.e. the vessel’s displacement does
not increase. 
 Because the displacement does not change, the COG of the vessel will not shift
vertically, transversely or longitudinally from its original position i.e. the vessel’s KG
and LCG will not change after bilging.
 However, the part of the compartment, which is submerged under water, is lost to the
sea i.e. the vessel’s underwater volume is said to be reduced by an amount equal to
Issue No. 1 Page 254 of 348
ST. THERESE- MTC COLLEGES SEAMANSHIP 6
Iloilo, Philippines (Advance Trim, Stability and Stress)
STUDENT LEARNING MODULE
Revision No. 1 Effectivity date: Reviewed by: Approved by:

10 August 2020 QMR President


the volume of the bilged part of the compartment. Hence the vessel’s buoyancy is
reduced. 
 To compensate for the lost buoyancy/volume without any change in displacement, the
vessel will submerge till the same is regained so that once again the vessel’s buoyancy
and displacement will remain equal. 
 This means that the vessel’s draft will increase due to bilging, but the underwater
volume and the displacement of the vessel will not change.
 Due to increase of the draft, COB of the vessel will shift upwards i.e. KB will increase.
 Due to loosing and regaining of the vessel’s buoyancy, the shape of the vessel’s
underwater volume will change. Hence the COB will also shift longitudinally and/or
transversely, away from the bilged compartment.
 Due to the longitudinal shift of COB, the horizontal distance between the new COB and
the original COG will change, which will change the vessel’s trim.
 Due to the transverse shift of COB, the horizontal distance between the new COB and
the original COG will change, which will change the vessel’s list.
 If the water level in the bilged compartment is the same as the water level outside the
vessel, then the vessel’s water-plane is also said to be bilged i.e. the area of the
water-plane is reduced by an amount equal to the area of the bilged compartment.
 COF is the geometric center of the water-plane. Hence, if the area of the water-plane
is reduced due to bilging, then the COF will shift longitudinally and/or transversely,
away from the bilged compartment.
 Vessel’s water-plane will not reduce and its COF will not shift if the water level in the
compartment does not rise up to the water level outside the vessel e.g. if a DB tank or
a deep tank located at the bottom of the vessel is bilged.
 Trim and list caused, as stated above, will occur about the new position of COF.
 Loss of buoyancy/volume due to bilging will be less if the compartment has some solid
cargo in it, because water entering the compartment will only occupy the empty
spaces within the cargo i.e. the loss of buoyancy/volume will depend on the
permeability of the compartment.
 By the same theory, the loss of water-plane area due to bilging will also be less,
depending on the permeability of the compartment.
 Bilging of a part or full compartment does not affect the original FSM of the vessel,
because the bilged compartment is not considered to be part of the vessel any more.
 If a tank containing some liquid is bilged then it is assumed that initially the liquid in
the tank is discharged, which will cause the following changes:
 Draft, U/W volume, displacement, KB and FSM will reduce.
 KG will increase.
 LCG, trim and list will change depending on location of the bilged tank.
 LCB will also change for vessel but for a box-shaped vessel it will remain the same
as half the length of the vessel.
Issue No. 1 Page 255 of 348
ST. THERESE- MTC COLLEGES SEAMANSHIP 6
Iloilo, Philippines (Advance Trim, Stability and Stress)
STUDENT LEARNING MODULE
Revision No. 1 Effectivity date: Reviewed by: Approved by:

10 August 2020 QMR President


Subsequently, the empty tank is said to be bilged, which will further change the draft, KB,
LCB, LCF, trim and list, as explained above.

12.2 Changes in stability parameters due to bilging


Following parameters will NOT change:
a. Displacement
b. Underwater volume
c. Position of COG i.e. KG, LCG and distance of COG from shipside
d. Shape and size of water-plane, when the level of water in the bilged
tank/compartment is less than the level of water outside the ship
e. Position of COF i.e. LCF and distance of COF from shipside, when the shape and size
of water-plane does not change
f. FSM / FSC

Following parameters will change:

a. Hydrostatic draft will increase


b. Position of COB i.e. KB, LCB and distance of COB from shipside
c. Trim and List
d. Shape and size of water-plane will reduce, when the level of water in the bilged
compartment is same as the level of water outside the ship.
e. Position of COF i.e. LCF and distance of COF from shipside, when the shape and size
of water-plane changes

12.3 Bilging Effects of Stability


12.3.1 The Effects of Bilging an Empty Amidships Compartments

The changes in draught and stability when a compartment becomes flooded due to damage
can be investigated by either of the two methods:

1. The lost buoyancy (constant displacement) method;


2. The added weight method

When a vessel floats in still water it displaces its own weight of water. Figure 12.2 shows a
box-shaped vessel floating at the waterline (WL). The weight of the vessel (W) is considered
to act downwards through G, the center of gravity. The force of buoyancy is also equal to W
and acts upwards through B, the center of buoyancy, b = W.

L
b = buoyancy

Aft Forward

G
Issue No. 1 Page 256 of 348
ST. THERESE- MTC COLLEGES SEAMANSHIP 6
Iloilo, Philippines (Advance Trim, Stability and Stress)
STUDENT LEARNING MODULE
Revision No. 1 Effectivity date: Reviewed by: Approved by:

10 August 2020 QMR President

W L
B
Transverse
Empty amidship
bulkhead
compartment

w = Weight
Figure 12.2

Figure 12.3 represents a vessel in which a compartment amidships has been bilged. The
vessel is shown as box shaped, for the sake of simplicity, but the effect will be the same for
ship shapes.
A B
W1 L1
m n
X
W L
v
D C
Figure 12.3
Bilged amidship
compartment

Suppose the ship if loaded originally at the waterline WL and that an empty compartment
ABCD is bilged. The buoyancy of this compartment is now lost to the ship. Call this lost
buoyancy v. The ship must continue to displace her own weight of water and so must
displace the same volume as before. In order to do this, she will sink to the new waterline
W1L1 so that the total volume of the layers m and n, which have become immersed, must
be equal to that of the lost buoyancy.

This mean that: v = m + n

If the total areas of the original and the new water planes are the same, the volumes of m
and n are together equal to the area of intact water plane multiplied by the increase of draft
(WW1 or LL1). So, if A is the total area of water plane, a the area within the compartment
and x the increase of draft in meters:
m + n = (A –a) x

But v= m + n
So, v = (A –a) x
v
Sinkage/Increase in draft (x) =
A−a
Issue No. 1 Page 257 of 348
ST. THERESE- MTC COLLEGES SEAMANSHIP 6
Iloilo, Philippines (Advance Trim, Stability and Stress)
STUDENT LEARNING MODULE
Revision No. 1 Effectivity date: Reviewed by: Approved by:

10 August 2020 QMR President

Process me
Example 1

A box-shaped lighter is 60 m long, 10 meters wide and floats at a mean draft of 2.00
meters. Find the sinkage if an empty compartment amidships, 20 meters long is bilged.

Given: Ship length : 60 m long


Ship breadth : 10 meters
Draft : 2.00 meters
Bilged compartment: 20 meters

L = 60 m

W1 L1
G
Sinkage (X) = 1.00 m
W L
20 m D=2.00 m

v Bilged
Figure 12.2
compartment

Figure 12.4

Required: Sinkage/ increase in draft

Formula: Lost buoyancy (v) = Bilged compartment x B x Draft


Original water-plane area (A) = L x B

v
Sinkage (x) =
A−a
Solution:
v
Sinkage (x) =
A−a
v 20 m x 10 m x 2.00 m
= =
A−a (60 m x 10 m)−(20 m x 10 m)
3
400 m m
= 2 2
600 m −200 m
Issue No. 1 Page 258 of 348
ST. THERESE- MTC COLLEGES SEAMANSHIP 6
Iloilo, Philippines (Advance Trim, Stability and Stress)
STUDENT LEARNING MODULE
Revision No. 1 Effectivity date: Reviewed by: Approved by:

10 August 2020 QMR President


3
400 m
Sinkage (x) = 2 = 1.00 m
400 m

Example 2

A box-shaped vessel is 50 meters long and floating on an even keel at 4.20 meters draft. An
amidships compartment is 10 meters long and is empty. Find the increase in draft if this
compartment is bilged.

Given: Ship length : 50 m long


Draft : 4.20 meters
Bilged compartment: 10 meters
v
Formula: Increase in draft (x) =
A−a
Solution:
v
Increase in draft (x) =
A−a
v 10 m x B m x 4.20 m
= =
A−a (50 m x Bm)−(10 m x Bm )

42 B m3
=
50 B m2−10 Bm 2
3
42 B m
Increase in draft (x) = 2 = 1.05 m
40 Bm

W1 G1 L1
G Sinkage (X ) = 1.05 m
W L
B1
B
v Bilged compartment
Figure 12.2
K
Figure 12.5
Example 3

A box-shaped vessel is 140 meters long, 20 meters wide and 12 meters deep and is floating
on an even keel at 5.50 meters drafts. The initial GM is 0.90 meters. An empty compartment
amidships is 16 meters long. Calculate the new GM if this compartment is bilged.

Given: Ship dimension: Length 140 meters; Breadth 20 meters; Deep 12 meters
Draft : 5.50 meters
Bilged compartment : 16 meters
Issue No. 1 Page 259 of 348
ST. THERESE- MTC COLLEGES SEAMANSHIP 6
Iloilo, Philippines (Advance Trim, Stability and Stress)
STUDENT LEARNING MODULE
Revision No. 1 Effectivity date: Reviewed by: Approved by:

10 August 2020 QMR President


Initial GM : 0.90 meters

W1 G1 L1
G Sinkage (X) = 0.71 m
W L
D2 = 6.21m B1
D1 = 5.50 m B
v Bilged compartment
Figure 12.2
K
Figure 12.6
Solution:
1. Calculate the Old Height of Center of Buoyancy (KB):
1
Old KB = Old draft
2
1
= (5.50 m)
2
Old KB= 2.75 meters

2. Calculate Old Metacentric radius (BM):


2
B
Old BM =
12d
202
=
12 x 5.50

400
Old BM= = 6.06 meters
66

3. Calculate the Old Height of metacenter (KM)


Old KMBOX = Old KB + Old BM
= 2.75 m + 6.06 m
Old KMBOX = 8.81 meters

4. Calculate the Height of center of gravity (KG):


KG = Old KM – Old GM
= 8.81 m - 0.90 m
KG = 7.91 m
Issue No. 1 Page 260 of 348
ST. THERESE- MTC COLLEGES SEAMANSHIP 6
Iloilo, Philippines (Advance Trim, Stability and Stress)
STUDENT LEARNING MODULE
Revision No. 1 Effectivity date: Reviewed by: Approved by:

10 August 2020 QMR President


Note: This KG will not change after bilging has taken place.
5. Calculate the Increase in Draft:
Lost buoyancy (v) = Bilged compartment x B x Draft
Original water-plane area (A) = L x B
v
Increase in draft (x) =
A−a

v 16 m x 20 m x 5.50 m
=
A−a
= (140 m x 20 m)−(16 m x 20 m)
3
1760m
= 2 2
2800 m −320 m

1760 m3
Increase in draft (x) = = 0.71 m
2480 m2

6. Calculate the New Draft:


New draft (d2) = Old draft + Increase in draft
= 5.50 m + 0.71 m
New draft (d2) = 6.21 m

7. Calculate the New KB:


1 d2
New KB = New draft =
2 2
1
= (6.21 m)
2
New KB= 3.11 meters

8. Calculate New Metacentric radius (BM):


2
B
New BM =
12d 2
2
20
=
12 x 6.21

400
New BM= = 5.37 meters
74.52

9. Calculate the New Height of metacenter (KM)


New KMBOX = New KB + New BM
= 3.11 m + 5.37 m
Issue No. 1 Page 261 of 348
ST. THERESE- MTC COLLEGES SEAMANSHIP 6
Iloilo, Philippines (Advance Trim, Stability and Stress)
STUDENT LEARNING MODULE
Revision No. 1 Effectivity date: Reviewed by: Approved by:

10 August 2020 QMR President


New KMBOX = 8.48 meters
10. Calculate the New GM:
New GM = New KM – KG
= 8.48 – 7.91 m
New GM =0.57 m

12.3.2 Bilging of an Empty Compartment not Amidships

Bilging End Compartments

When the bilged compartments is situated in a position away from amidships, the vessel’s
mean draft will increase to make good the lost buoyancy, but the trim will also change.

This may be calculated as follows:

a) The bodily sinkage is found in the same way as for an amidships compartment.

b) The method of finding change of trim is illustrated by Figure 12.7b. Here, it can be
seen that, since there is a loss of buoyancy at one end of the ship. B will move away
from that end to a new position B 1. The forces of buoyancy and gravity then form a
couple, forcing the bilged end of the ship downwards.

As the ship changes her trim, the center of buoyancy will move back towards G until it again
comes vertically under G.

It can be seen from the diagram that the couple causing the ship to change her trim
consists of the forces of gravity and buoyancy acting on the lever BB 1.

So, Moment changing trim = W x BB1

Moment changing trim


Change of trim =
MCT 1C

W x BB 1
Change of trim =
MTC 1C

c) To find BB1:
v x ¿1
It has been shown that BB1 = and that B obeys the same laws as any other center
V
of gravity.
So if D is the distance of the center of gravity of the bilged compartment from B, then:
v xd
BB1=
V
Issue No. 1 Page 262 of 348
ST. THERESE- MTC COLLEGES SEAMANSHIP 6
Iloilo, Philippines (Advance Trim, Stability and Stress)
STUDENT LEARNING MODULE
Revision No. 1 Effectivity date: Reviewed by: Approved by:

10 August 2020 QMR President


Note: In the case of an end compartment in a box shaped vessel, BB 1 will be equal to half
the length of the bilged compartment. This will only apply to this particular case.

d) If the new drafts are required, we must calculate the change of draft due to trim.
Remember, in this case, that the vessel will trim about the new center of flotation,
which will be at the center of gravity of the new, intact water-plane.

W
L

W L

B
Figure 12.2

w
Figure 12.7a

d
W
W1 F L1
G
L
B1 B
V
Figure 12.2 Bilged compartment

w
Figure 12.7b

Process me
Example 1

A box-shaped vessel is 120 meters long and 25 meters beam, floats at a drafts of 5.00
meters fore and aft. Find the new drafts after an empty end compartment 15 meters long is
bilged forward.

Given: Ship length : 120 meters


Beam : 25 meters
Issue No. 1 Page 263 of 348
ST. THERESE- MTC COLLEGES SEAMANSHIP 6
Iloilo, Philippines (Advance Trim, Stability and Stress)
STUDENT LEARNING MODULE
Revision No. 1 Effectivity date: Reviewed by: Approved by:

10 August 2020 QMR President


Draft : 5.00 meters fore and aft
Bilged compartment: 15 meters

Required: New drafts after an empty end compartment is bilged forward.

W
L = 120 m

M
x = 15 m

G d = 45 m

W L

D=5.00 m B1 B
Intact water-plane area 7.5 m
Figure 12.2

w
Figure 12.8a

W
W1 F L1
G
d = 52.5 m L
d
B1 B V
Bilged
Figure 12.2 compartment

BB1 = 7.5 m w
Figure 12.8b
Solution:
Lost buoyancy (v) = Bilged compartment x B x Draft
Original water-plane area (A) = L x B

1. Calculate increase of draft/Bodily sinkage


v
Bodily sinkage (x) =
A−a
Bilged compartment ( m ) x Beam( m) x Draft (m)
=
( L x B)−(Bilged compartment x B)
15 m x 25 m x 5.00 m
=
(120 m x 25 m)−(15 m x 25 m)
Issue No. 1 Page 264 of 348
ST. THERESE- MTC COLLEGES SEAMANSHIP 6
Iloilo, Philippines (Advance Trim, Stability and Stress)
STUDENT LEARNING MODULE
Revision No. 1 Effectivity date: Reviewed by: Approved by:

10 August 2020 QMR President


3 3
1875m 1875 m
= 2 2 = 2
3000 m −375 m 2625 m

Bodily sinkage (x) = 0.71 m

2. Calculate the Moment to Change Trim by 1 Centimeter:


Note: We must find first the W and BML.

Displacement (W) = L x B x D x R.D.


Displacement (W) = 120 m x 25 m x 5.00 m x 1.025 t/m 3
Displacement (W) = 15,375 tonnes

hence: I = L – Bilged compartment = 120 m – 15 m = 105 m


D = initial draft + bodily sinkage = 5.00 m + 0.71 m = 5.71 m

I L
2
BML =
V
=
12 D
2 2
105 11,025m
BML= = = 161 m
12 x 5.71 68.52 m

W x BM L 15,375t x 161 m
MCT1C = =
100 L 100 x 120 m

2,475,375t−m
=
12,000 m
MCT1C = 206 t-m
1
BB1 = length of compartment
2
15
BB1 = = 7.5 m
2
Intact water-plane = L - Bilged compartment
= 120 m - 15 m
Intact water-plane= 105 m

105
F is at half length of intact water plane = = 52.5 m from aft
2

3. Calculate Change of Trim:


W x BB 1 15,375t x 7.5 m 115,312.5t−m
Change of Trim = = = = 560 cm
MCT 1C 206 t−m/cm 206 t−m/cm
Issue No. 1 Page 265 of 348
ST. THERESE- MTC COLLEGES SEAMANSHIP 6
Iloilo, Philippines (Advance Trim, Stability and Stress)
STUDENT LEARNING MODULE
Revision No. 1 Effectivity date: Reviewed by: Approved by:

10 August 2020 QMR President


4. Calculate change of draft forward and aft:
where: (l a= 52.5)
la
Change of draft aft = x COT
120
52.5 m 52.5 x 560 cm 29,400 cm
Change of draft aft = x 560 cm = =
120 m 120 120
Change of draft aft =245 cm (convert to meters)

Change of draft aft = 2.45 m

Change of draft forward = C.T. – Change of draft aft


Change of draft forward = 560 cm – 245 cm
Change of draft forward = 315 cm – convert to meters
Change of draft forward = 3.15m

5. Calculate new drafts:


Forward Aft
Initial drafts 5.00 m 5.00 m
Bodily sinkage +0.71 m + 0.71 m
5.71 m 5.71 m
Change of trim +3.15 m -2.45 m
New Drafts 8.86 m 3.26 m

Example 2

A box-shaped vessel is 80 m long by 12 m wide x 6.0 m deep is floating in salt water on an


even keel at a draft of 4.50 m. Calculate the new drafts if a forward compartments 6.0
meter long is bilged.

Given: Ship length : 80 meters


Issue No. 1 Page 266 of 348
ST. THERESE- MTC COLLEGES SEAMANSHIP 6
Iloilo, Philippines (Advance Trim, Stability and Stress)
STUDENT LEARNING MODULE
Revision No. 1 Effectivity date: Reviewed by: Approved by:

10 August 2020 QMR President


Breadth : 12 meters
Depth : 6.0 meters
Draft : 4.50 meters – even keel
Bilged compartment: 6.0 meters

Required: New drafts after an empty end compartment is bilged forward.


b
L = 80 m
L2 = 74 m

I = 37 m F M x=6m

W1 L1
Increase in draft = 0.36 m G
W L
Bilge
B1 B compartment d1 4.50
Intact water-plane area 3m m
Figure 12.2

w
Figure 12.9
1. Calculate the vessel bodily sinkage:
a. w= x x B x d1 x R.D.
w = 6.0 m x 12 m x 4.50 m x 1.025 tonnes/m3
w = 332.1 tonnes
WPA L2 x B
b. TPC = =
97.56 97.56
where: L2 = Length – Bilged Compartment= 80 m – 6.0 m = 74 m
2
74 m x 12m 888 m
TPC = = = 9.10 tonnes
97.56 97.56
w
c. Increase in draft =
TPC
332.1tonnes
=
9.10 tonnes/ cm
Increase in draft = 36.49 cm (convert to meters)
Increase in draft = 0.36 m

2. Calculate New mean draft:


New Mean Draft (d2) = Old draft (d1) ± Increase in draft
= 4.50 m + 0.36 m
New Mean Draft (d2) = 4.86 m
Issue No. 1 Page 267 of 348
ST. THERESE- MTC COLLEGES SEAMANSHIP 6
Iloilo, Philippines (Advance Trim, Stability and Stress)
STUDENT LEARNING MODULE
Revision No. 1 Effectivity date: Reviewed by: Approved by:

10 August 2020 QMR President


3. Calculate the Change of Trim:
Displacement (W) = L x B x d1 x R.D.
= 80 m x 12 m x 4.50 x 1.025 tonnes/m3
W = 4,428 tonnes
4. Calculate the Longitudinal Metacentric Radius (BML):
IL
BML =
V
B L32
= where: v = L x B x d1
12V
12 m x 74 3
=
12(L x B x d 1)
3
12 m x 405,224 m
=
12(80 m x 12 m x 4.50 m)
4
4,862,688 m 4,862,688 m4
BML= 3 = = 93.80 m
12(4,320 m ) 51,840 m3
5. Calculate the MCT1C:
W x BM L
MTC1C =
100 L
4,428 t x 93.80 m
MTC1C =
100 cm x 80 m
415,346.4 t−m
MTC1C = =51.92 tonnes/cm
8000 cm−m
6. Calculate Change of Trim:
Moment changing trim
Change of Trim =
MCT 1C
where:
LBP 80 m
d= = = 40 m = Lever from new LCF (Longitudinal Center of Flotation)
2 2
w xd 332.1t x 40 m 13,284 t−m
Change of Trim =
MCT 1C
=
51.92t /cm
= 51.92t /cm
Change of Trim =256 cm by the head

7. Calculate the change of drafts:


After bilging, LCF has moved to F, that is I= (L – x)/2 from the stern
Issue No. 1 Page 268 of 348
ST. THERESE- MTC COLLEGES SEAMANSHIP 6
Iloilo, Philippines (Advance Trim, Stability and Stress)
STUDENT LEARNING MODULE
Revision No. 1 Effectivity date: Reviewed by: Approved by:

10 August 2020 QMR President


L−x 80 m−6 m 74
.∴ I = 2 = 2 = 2 = 37 m

I
7.1 Change of draft aft = x Change of Trim
L

37 9472
= x 256 cm =
80 80

Change of draft aft= 118.4 cm (convert to meters)

Change of draft aft = 1.18 m

7.2 Change of draft forward = Change of Trim – Change of draft aft


Change of draft forward = 256 cm – 118 cm
Change of draft forward = 138 cm (convert to meters)
Change of draft forward = 1.38 m

8. Calculate New Drafts


Forward Aft
Initial drafts 4.86 m 4.86 m
Change of trim +1.38 m - 1.18 m
New Drafts 6.24m 3.68 m

12.2 Permeability ( μ)

Permeability of a space in a ship is the percentage of empty volume in that space or the
ratio between the space available for water and the total space in the compartment.

Permeability is used in ship survivability and damaged stability calculations in ship design. In
this case, the permeability of a space is a percentage from 0 to 100. Alternately, the
permeability may be a coefficient from 0 to 1. The permeability of a space is the percentage
of volume of the space which may be occupied by seawater if the space is flooded. The
remaining volume [not filled with seawater] being occupied by machinery, cargo,
accommodation spaces, etc.

For instance, suppose that a compartment has a volume of 5000 m 3. This would be the
volume available for water if the empty compartment was bilged. If this compartment was
Issue No. 1 Page 269 of 348
ST. THERESE- MTC COLLEGES SEAMANSHIP 6
Iloilo, Philippines (Advance Trim, Stability and Stress)
STUDENT LEARNING MODULE
Revision No. 1 Effectivity date: Reviewed by: Approved by:

10 August 2020 QMR President


filled with cargo, the solid parts of that cargo would take up space which would be
otherwise available for water, so that less material in the compartment occupied 3,000 m 3,
then only 2,000 m3 of space would be available for water.

Spce available for water


In this case, the permeability =
Total space
2,000 m3
= 3 = 0.4 = 40%
5000 m

For certain types of cargo, it is possible to calculate the permeability if the stowage factor
and relative density are known.

Relative density is the ratio between the weight of solid material and the weight of an equal
volume of fresh water.

Space occupied by 1 tonne fresh water = 1 m3


1
Space occupied by 1 tonne solid cargo = m3
R elative density

For each tonne of cargo, as stowed, the space available for water = Space occupied by 1
tonne as stowed (Stowage factor) – Space occupied by 1 tonne of solid cargo.

= Stowage factor – (1 ÷ Relative density)

Space available for water


Permeability = (per tonne)
Total space

Space factor−(1÷ Relative density)


Permeability =
Total space

Permeability is the amount of water that can enter a compartment or rank after it has been
bilged. When an empty compartment is bilged, the whole of the buoyancy provided by that
compartment is lost. Typical values for permeability, μ, are as follows:
 Empty compartment μ = 100%
 Engine room μ = 80 – 85%
 Grain-filled cargo hold μ = 60 – 65%
 Coal-filled compartment μ= 36% approximately
 Filled water ballast tank (when ship is in salt water) μ = 0%

Consequently, the higher the value of the permeability for a bilged compartment, the
greater will be a ship’s loss of buoyancy when the ship is bilged.
Issue No. 1 Page 270 of 348
ST. THERESE- MTC COLLEGES SEAMANSHIP 6
Iloilo, Philippines (Advance Trim, Stability and Stress)
STUDENT LEARNING MODULE
Revision No. 1 Effectivity date: Reviewed by: Approved by:

10 August 2020 QMR President

The permeability of a compartment can be found from the formula:

Broken stowage
μ = Permeability = x 100%
Stowage factor

When a bilged compartment contains cargo, the formula for finding the increase in draft
must be amended to allow for the permeability. If ‘ μ ' represents the permeability,
expressed as a fraction, then the volume of lost buoyancy will be ‘ μ v ' and the area of the
intact water plane will be ‘A - μv ' square meters. The formula then reads:
μv
x= A−μa

Process me
Example 1

Determine the permeability of a cargo which has a relative density of 1.75 and stowage
factor of 1.10.

Space factor−(1÷ Relative density)


Permeability (μ) =
Stowage factor
1.10−(1÷ 1.75)
Permeability (μ) =
1.10
1.10−(0.571) 0.529
Permeability (μ) = = = 0.4809 or 48.1%
1.10 1.10

Example 2

A box-shaped vessel is 60 meters long and is floating on an even keel at 4.00 meters draft.
A compartment amidships is 10 m long and contains cargo having a permeability of 25%.
Calculate the increase in the draft if this compartment is bilged.

Given: Ship length : 60 meters


μPermeability of cargo: 25%
Draft : 4.00 meters – even keel
Bilged Compartment : 10.0 meters

Required: Increase in draft if compartment is bilged


μv
Formula: x=
A−μa
Issue No. 1 Page 271 of 348
ST. THERESE- MTC COLLEGES SEAMANSHIP 6
Iloilo, Philippines (Advance Trim, Stability and Stress)
STUDENT LEARNING MODULE
Revision No. 1 Effectivity date: Reviewed by: Approved by:

10 August 2020 QMR President

Solution:
where: v = Length of compartment x Breadth x draft
α = Length of compartment x Breadth
A=LxB
0.25 x 10.0 m x B x 4.00 m
x=
( 60 m x B )−(0.25 x 10 x B)
10 B
x=
60 B−2.5 B

10 B
x=
57.5 B
Increase in draft x = 0.17 m

Example 3

A box-shaped vessel is 160 m x 30 m x 14 m is floating on an even keel at 6.00 meters


draft. A compartment amidships is 15 m long and contains timber cargo of relative density
0.8 and stowage factor 1.60 cubic meters per tonne. Calculate the new draft if this
compartment is now bilged.

Given: Ship dimension : 160m x 30m x 14 m


Draft : 6.00 meters – even keel
Bilged Compartment : 15.0 meters long
Cargo R.D. : 0.8 Stowage factor: 1.6 cubic meters per tonne.

Required: New draft if compartment is bilged.

Solution:
a. Calculate permeability
Broken stowage
Formula: μ = Permeability = x 100%
Stowage factor

hence: 1 tonne of fresh water occupies 1 m3 and relative density of timber is 0.8.
1
∴ space occupied by 1 tonne of solid timber =
0.8
= 1.25 m3
Broken stowage = Space occupied by 1 tonne of solid timber – Stowage Factor
Broken stowage = 1.25 m3 1.60m3
Broken stowage = 0.35 m3
Issue No. 1 Page 272 of 348
ST. THERESE- MTC COLLEGES SEAMANSHIP 6
Iloilo, Philippines (Advance Trim, Stability and Stress)
STUDENT LEARNING MODULE
Revision No. 1 Effectivity date: Reviewed by: Approved by:

10 August 2020 QMR President

Broken stowage
μ = Permeability = x 100%
Stowage factor
0.35
μ = Permeability = x 100%
1.60
μ = Permeability = 0.21875 x 100= 21.88%

b. Calculate increase in draft:


Lost buoyancy (v) = Bilged compartment x B x Draft
Original water-plane area (A) = L x B

μv
Increase in draft (x) =
A−μa
0.21875 x 15 m x 30 m x 6 m
Increase in draft (x) =
( 160 m x 30 m )−(0.21875 x 15 m x 30 m)
3
590.625m
Increase in draft (x) =
( 4800 m2 )−(98.4275 m2)
3
590.625m
Increase in draft (x) = = 0.126 m
4702 m2

c. Calculate New draft (d2):


New draft (d2) = Old draft (d1) + Increase in draft
= 6.00 m + 0.126 m
New draft (d2) = 6.13 m

When a bilged compartment does not extend above the waterline, the area of the intact
water plane remains constant, as shown in Figure 12.10. The figure shows: μv = Ax

Let d = Density of the water


Then μv x d = Ax x d
But
μv x d = Mass of water entering the bilged compartment
Ax x d = Mass of the area layer of water displaced
∴ when the compartment is bilged, the extra mass of water displaced is equal to the
buoyancy lost in the bilged compartment. It should be carefully noted, however, that
although the effect on draft is similar to that of loading a mass in the bilged compartment
equal to the lost buoyancy, no mass has in fact been loaded. The displacement after bilging
Issue No. 1 Page 273 of 348
ST. THERESE- MTC COLLEGES SEAMANSHIP 6
Iloilo, Philippines (Advance Trim, Stability and Stress)
STUDENT LEARNING MODULE
Revision No. 1 Effectivity date: Reviewed by: Approved by:

10 August 2020 QMR President


is the same as the displacement before bilging and there is no alteration in the position of
the vessel’s center of gravity. The increase in the draft is due to solely to lost buoyancy.

L = 100 m

W1 L1
Ax x
W L
Bilge compartment
μv
Figure 12.2

Aft Figure 12.10 Forward

Example 4

A vessel is floating in salt water on an even keel at 5.00 m draft. TPC is 18 tonnes. A
rectangular-shaped compartment amidships is 15 m long, 8 m wide and 4 meters deep. The
compartment contains cargo with permeability 25%. Calculate the new draft if this
compartment is bilged.

W1 L1
x
W L
15.00 m
Bilge compartment
μv 4.00 m 5.00 m
Figure 12.2

Aft Forward
Figure 12.10
Solution:
a. Calculate buoyancy lost:
Buoyancy lost = μv
25
Buoyancy lost = x (15m x 8 m x 4m) x 1.025 t/m3
100
25
Buoyancy lost = x 480 m3 x 1.025 t/m3
100
Buoyancy lost = 0.25 x 480 m3 x 1.025 t/m3

Buoyancy lost = 123 tonnes

b. Calculate increase in draft


Issue No. 1 Page 274 of 348
ST. THERESE- MTC COLLEGES SEAMANSHIP 6
Iloilo, Philippines (Advance Trim, Stability and Stress)
STUDENT LEARNING MODULE
Revision No. 1 Effectivity date: Reviewed by: Approved by:

10 August 2020 QMR President


Extra mass of water displaced: w = TPC x X tonnes
w 123tonnes
∴x= = = 6.83 cm
TPC 18 t/cm
Increased in draft (x) = 6.83 cm (convert to meters)
Increased in draft (x) = 0.0683 meters

c. Calculate the new draft:


New draft (d2) = Old draft (d1) + Increase in draft
New draft (d2) = 5.00 m + 0.0683 m
New draft (d2) = 5.0683 m = 5.07 m

Do this

Activity 1

A box-shaped vessel is 100 m long, 15 meters wide and floats at a mean draft of 5.00
meters. Determine the increase in draft if an empty compartment amidships, 10 meters long
is bilged.
Write you solutions and answer in the space provided.
Issue No. 1 Page 275 of 348
ST. THERESE- MTC COLLEGES SEAMANSHIP 6
Iloilo, Philippines (Advance Trim, Stability and Stress)
STUDENT LEARNING MODULE
Revision No. 1 Effectivity date: Reviewed by: Approved by:

10 August 2020 QMR President


Do this

Activity 2 – Bilging of an Empty Compartment Amidships Problem

A box-shaped vessel is 100 meters long, 20 meters wide and 10 meters deep and is floating
on an even keel at 5.00 meters drafts. The initial GM is 0.95 meters. An empty compartment
amidships is 8 meters long and this compartment is bilged. Calculate the following:

1. Old Height of Center of Buoyancy (KB):


2. Old Metacentric radius (BM):
3. Old Height of metacenter (KM)
4. Height of center of gravity (KG):
5. Increase in Draft:
6. New Draft:
7. New KB:
8. New Metacentric radius (BM):
9. New Height of metacenter (KM)
10. New GM:
Write you solutions and answer in the space provided.

Write you solutions and answer in the space provided.


Issue No. 1 Page 276 of 348
ST. THERESE- MTC COLLEGES SEAMANSHIP 6
Iloilo, Philippines (Advance Trim, Stability and Stress)
STUDENT LEARNING MODULE
Revision No. 1 Effectivity date: Reviewed by: Approved by:

10 August 2020 QMR President

Do this

Activity 3 – Bilging End Compartments Problem


Issue No. 1 Page 277 of 348
ST. THERESE- MTC COLLEGES SEAMANSHIP 6
Iloilo, Philippines (Advance Trim, Stability and Stress)
STUDENT LEARNING MODULE
Revision No. 1 Effectivity date: Reviewed by: Approved by:

10 August 2020 QMR President


A box-shaped vessel is 110 meters long and 20 meters beam, floats at a drafts of 6.00
meters fore and aft. An empty end compartment 12 meters long is bilged forward. Calculate
the following:

1. Increase of draft
2. Initial Displacement
3. BML
4. Calculate the MCT1C
5. Calculate Change of Trim:
6. Change of draft forward
7. Change of draft aft:
8. Calculate new drafts forward and aft

Write you solutions and answer in the space provided.

Write you solutions and answer in the space provided.


Issue No. 1 Page 278 of 348
ST. THERESE- MTC COLLEGES SEAMANSHIP 6
Iloilo, Philippines (Advance Trim, Stability and Stress)
STUDENT LEARNING MODULE
Revision No. 1 Effectivity date: Reviewed by: Approved by:

10 August 2020 QMR President

Do this

Activity 4 – Bilging and Permeability ( μ) Problem

A vessel is floating in salt water on an even keel at 7.00 m draft. TPC is 25 tonnes. A
rectangular-shaped compartment amidships is 10 m long, 8 m wide and 4 meters deep. The

Write you solutions and answer in the space provided.


Issue No. 1 Page 279 of 348
ST. THERESE- MTC COLLEGES SEAMANSHIP 6
Iloilo, Philippines (Advance Trim, Stability and Stress)
STUDENT LEARNING MODULE
Revision No. 1 Effectivity date: Reviewed by: Approved by:

10 August 2020 QMR President


compartment contains cargo with permeability 40%. Calculate the new draft if this
compartment is bilged.

Assessment

A box-shaped vessel 100 meters long, 20 meters beam floats at drafts at an even drafts of
4.00 meters. Determine the new drafts after an empty end compartment 10 meters long is
bilged forward. Calculate the following:
1. Increase of draft
2. Initial Displacement
Issue No. 1 Page 280 of 348
ST. THERESE- MTC COLLEGES SEAMANSHIP 6
Iloilo, Philippines (Advance Trim, Stability and Stress)
STUDENT LEARNING MODULE
Revision No. 1 Effectivity date: Reviewed by: Approved by:

10 August 2020 QMR President


3. BML
4. Calculate the MCT1C
5. Calculate Change of Trim:
6. Change of draft forward
7. Change of draft aft:
8. Calculate new drafts forward and aft
Write your solutions and answer in the space provided.

Write your solutions and answer in the space provided.


Issue No. 1 Page 281 of 348
ST. THERESE- MTC COLLEGES SEAMANSHIP 6
Iloilo, Philippines (Advance Trim, Stability and Stress)
STUDENT LEARNING MODULE
Revision No. 1 Effectivity date: Reviewed by: Approved by:

10 August 2020 QMR President

Lesson 13: List due to Bilging Side Compartments

Learning Module 13.1: List due to Bilging Side Compartments

Competence, Course Outcomes and Learning Outcomes

Competence:

Control trim, stability and stress

Course Outcome:

1. Explain the importance of maintaining stability during loading, unloading


and in-transit in various conditions.
2. Calculate the effect on trim and stability of a ship in the event of damage to
and consequent flooding of a compartment and countermeasures to be
taken.

Learning Outcomes:

At the end of the lesson, the student should be able to:

1. Draw the estimation of second moment of an area and in the bilged condition with the
use of the parallel axis theorem.
2. Calculate the second moment of the area about the traverse axis through the center
of flotation using the parallel axis theorem.
3. Calculate the following:
– the mean bodily increase in draft;
– the shift of center of buoyancy and;
– the angle of list
– evaluate the new KB, BM, KM and GM result in relation to the stability
Issue No. 1 Page 282 of 348
ST. THERESE- MTC COLLEGES SEAMANSHIP 6
Iloilo, Philippines (Advance Trim, Stability and Stress)
STUDENT LEARNING MODULE
Revision No. 1 Effectivity date: Reviewed by: Approved by:

10 August 2020 QMR President

Discussion

13.1 Angle of List Due to Bilging of Side Compartments

When a compartment in a ship is bilged the buoyancy provided by that compartment is lost.
This causes the center of buoyancy of the ship to move directly away from the center of lost
buoyancy and, unless the center of gravity of the compartment is on the ship’s centerline, a
listing moment will be created, b = w.

Let the ship in Figure 13.1a float upright at the waterline, WL. G represents the position of
the ship’s center of gravity and B the center of buoyancy.

Now let a compartment that is divided at the centerline be bilged on the starboard side, as
shown in the figure 13.1b To make good the lost buoyancy the ship will sink to the waterline
W1L1. That is, the lost buoyancy is made good by the layer between WL and W 1 L1.

The center of buoyancy will move from B to B 1, directly away from the center of gravity of
the lost buoyancy, and the distance BB 1 is equal to (w x d)/W., where w represents the lost
buoyancy and d represents the distance between the ship’s center of buoyancy and the
center of the lost buoyancy.

(a) (b)
Figure 13.1

The shift in the center of buoyancy produces a listing moment. Let Ø be the resultant list.
GX BB 1
Then: tan Ø=
XM
= XM
Issue No. 1 Page 283 of 348
ST. THERESE- MTC COLLEGES SEAMANSHIP 6
Iloilo, Philippines (Advance Trim, Stability and Stress)
STUDENT LEARNING MODULE
Revision No. 1 Effectivity date: Reviewed by: Approved by:

10 August 2020 QMR President


Where XM represents the initial metacentric height for the bilged condition.

Process me
Example 1

A box-shaped vessel, of length 100 m and breadth 18 m, floats in salt water on an even keel
at 7.50 m draft. KG = 4.00 m. the ship has a continuous centerline bulkhead that is
watertight (see Figure 13.2). Find the list if a compartment amidships, which is 15 m long
and is empty, is bilged on one side.
L = 100 m

B1
O Z
C L B=18 m
B
a d1 = 9 m

I = 15 m
Figure 13.2

a) Find the new mean draft:

Volume of lost buoyancy


Bodily increase in draft =
Area of intact W . P .
I x d 1 x Draft
Bodily increase in draft =
( L x B ) −( I x d 1)
15 x 9 x 7.5 1012.5 1012.5
Bodily increase in draft =
( 100 x 18 )−(15 x 9)
==
1800−135
= 1665 = 0.61 m

Old draft = 7.50 m


Bodily rise = + 0.61 m
New draft = 8.11 m = Draft d2

b) Find the shift of the center of buoyancy:


Issue No. 1 Page 284 of 348
ST. THERESE- MTC COLLEGES SEAMANSHIP 6
Iloilo, Philippines (Advance Trim, Stability and Stress)
STUDENT LEARNING MODULE
Revision No. 1 Effectivity date: Reviewed by: Approved by:

10 August 2020 QMR President


a x B/4
BB1 =
LB−a
15 x 9 x 18 /4
BB1 =
( 100 x 18 )−(15 x 9)

2430/ 4
BB1 =
( 1800 )−( 135)

607.5
BB1 = =0.365 m
1665

c) Find IOZ:
3 3
B xL B (L−l)
ICL = +
3 3
3
93 x 100 9 (100−15)
ICL = +
3 3
729 x 100
ICL =
3
+729 x 85 ¿ ¿3
72,900
ICL =
3
+ 61,965
3
ICL = 24,300 + 20,655
ICL= 44,955 m4

IOZ = ICL – A x BB12 A = Intact area of water-plane

= 44,955 m4 – {(100 m x 18 m) – (15m x 9 m} x 0.3652m


= 44,955 m4– {1800 m2 – 135 m2} x 0.3652m
= 44,955 m4– 1665 m2 x 0.133255 m2
= 44,955 m4– 222 m4

IOZ = 44,733 m4

d) To find GM:

Ioz
BM=
V
4
44,733
BM =
100 m x 18 m x 7.5 m
Issue No. 1 Page 285 of 348
ST. THERESE- MTC COLLEGES SEAMANSHIP 6
Iloilo, Philippines (Advance Trim, Stability and Stress)
STUDENT LEARNING MODULE
Revision No. 1 Effectivity date: Reviewed by: Approved by:

10 August 2020 QMR President


4
44,733 m
BM = 3 = 3.31 m
13,500 m
d2
KB therefore:
2
8.11
KB = 4.06 m
2

KM = BM + KB
= 3.31m + 4.06m
KM = 7.37 m

After bilging GM = KM – KG (before bilging)


= 7.37 m – 4.00 m
After bilging GM = 3.37 m

e) To find List:
GX BB 1
tan Ø = =
XM XM

BB1
tan Ø =
GM

0.365
Ø= = 0.108308605 tan-1
3.37

List Ø = 6° 11’ = 6.2º

Example 2

A box-shaped vessel, 50 m long x 10 m wide, floats in salt water on an even keel at a draft
of 4.00 m. A centerline longitudinal watertight bulkhead extends from end to end and for
the full depth of the vessel. A compartment amidships on the starboard side is 15 m long,
and contains cargo with permeability μof 30% (see Figure 4.3). Calculate the list if this
compartment is bilged. KG = 3 m.

L = 50 m

B1
O Z
L B=10m
C B
a d1 = 5 m

I = 15 m
Issue No. 1 Page 286 of 348
ST. THERESE- MTC COLLEGES SEAMANSHIP 6
Iloilo, Philippines (Advance Trim, Stability and Stress)
STUDENT LEARNING MODULE
Revision No. 1 Effectivity date: Reviewed by: Approved by:

10 August 2020 QMR President

Figure 13.3

a) Find the new mean bodily increase in draft:


Cargo with 30% permeability ( μ ) = 30/100 = 0.30%
Volume of bilged tank = (l x d1 x draft x μ)
= (15 m x 5 x 4 m x 0.30 %) = 90 m3
Area of intact water-plane = 477.5 m2

Volume of lost buoyancy


Bodily increase in draft =
Area of intact W . P .
μ xI x d 1 x Draft
Bodily increase in draft =
( L x B ) −( μ x I x d 1)

0.3 x 15 x 5 x 4
Bodily increase in draft =
( 50 x 10 )−(0.3 x 15 x 5)

90 90
Bodily increase in draft =
500−22.5
= 477.5
= 0.19 m

Old draft = 4.00 m


Bodily rise = + 0.19 m
New draft = 4.19 m = Draft d2

b) Find the shift of the center of buoyancy:


μ a x B /4
BB1 =
LB−μ a
0.3 x 15 x 5 x 10 /4
BB1 =
( 50 x 10 )−(0.3 x 15 x 5)

225 /4 56.25
BB1 = =
500−22.5 477.5

BB1 = 0.1178 m = 0.12 m

c) Find Second Moment of Area IOZ:

L B3 μl b3
ICL = -
12 3

ICL =
(50 x 10
12
3

)(
- 0.3 x 15 x 5
3
3

)
50 x 1000 0.3 x 15 x 125
ICL =
12
- 3
Issue No. 1 Page 287 of 348
ST. THERESE- MTC COLLEGES SEAMANSHIP 6
Iloilo, Philippines (Advance Trim, Stability and Stress)
STUDENT LEARNING MODULE
Revision No. 1 Effectivity date: Reviewed by: Approved by:

10 August 2020 QMR President


50000 562.5
ICL =
12
- 3

ICL = 4,166.67 - 187.5

ICL= 3,979.17 m4

IOZ = ICL – A x BB12

= 3,979.17 m4 – (477.5 x 0.122)

= 3,979.17 m4 – (477.5 x 0.0144)

= 3,979.17 m4 – 6.88

IOZ = 3,972.3 m4

d) To find GM:

Ioz
BM=
V
4
3,972.3 m
BM =
50 m x 10 m x 4 m

3972m4
BM = = 1.986 m
2000 m3

d2
KB = therefore
2
4.19
KB = = 2.095 m
2

KM = BM + KB

= 1.99 m + 2.10m

KM = 4.09 m

After bilging GM = KM – KG (before bilging)


= 4.09 m – 3.00 m
After bilging GM= 1.09 m

e) To find List:
BB 1
tan Ø =
GM
Issue No. 1 Page 288 of 348
ST. THERESE- MTC COLLEGES SEAMANSHIP 6
Iloilo, Philippines (Advance Trim, Stability and Stress)
STUDENT LEARNING MODULE
Revision No. 1 Effectivity date: Reviewed by: Approved by:

10 August 2020 QMR President


0.12
tan Ø = = 0.1100917431
1.09

List Ø= 0.1100917431 tan-1 = 6° 17’ = 6.3º

13.2 Calculating the List when an amidships Side Compartment is Bilged –


Permeability 100%
Moment of Inertia
3 3
LB LB
For box-shaped vessel: BMBOX = where:
12V 12

Is the moment of inertia of the water plane area and V is the volume of displacement of the
vessel.

The most influential factor affecting the BM is the size of the water plane area but in
particular, the breadth. Many students have difficulty understanding the value of the
‘moment of inertia’ (I) because it is difficult to picture. A way of thinking about the role that
the moment of inertia plays is to consider that the water plane area offers the ship
resistance to rolling. This is true because the greater the water plane area and particularly
the breadth, the greater will be BM and consequently KM and GM, giving the ship greater
initial stability.

The moment of inertia about an axis (of rotation) is equal to the product of an area and the
square of its distance from that axis.

This will be looked at in more detail. The way to picture this is to consider that the ship
stays upright and the water plane area rotates about a longitudinal axis of rotation that
passes through the center of flotation (instead of the ship rolling and the water line
remaining horizontal).

Figure 13.4

Moments of Inertia of Rectangular Water Plane Area

The smallest values of moment of inertia for any rectangular water plane area occur when
the axis of rotation (longitudinal or transverse) passes through the centroid of the water
Issue No. 1 Page 289 of 348
ST. THERESE- MTC COLLEGES SEAMANSHIP 6
Iloilo, Philippines (Advance Trim, Stability and Stress)
STUDENT LEARNING MODULE
Revision No. 1 Effectivity date: Reviewed by: Approved by:

10 August 2020 QMR President


plane area concerned i.e. the axis of rotation passes through the center of flotation (which is
the geometric center of the water plane area).

The smallest transverse value of I is given by:


3
LB
III = F
12 I I

The smallest longitudinal value of I is given by: IL

L B3 F
ILL =
12
IL
Figure 13.5
The moment of inertia about an axis passing along one edge of the water plane area can be found.
These values may be calculated as follows:
1. Transverse value of I about one edge.

3 F
LB
IXX =
3 x x

2. Longitudinal value of I about one edge. Y

3
BL F
IYY =
3
Figure 13.6
Y

13.3 The Parallel Axis Theorem

Parallel Axis Theorem

It states that ‘ the moment of inertia about any axis passing through the centroid of a water
plane are equal to the moment of inertia of the same water plane area about any parallel
axis, minus the area of the water plane multiplied by the distance between the axis
squared.”

Figure 13.7 shows the moment of inertia about the axis II (which passes through the center
of flotation) is equal to the moment of inertia about the axis XX minus the product of the
water plane area and the distance between the axis squared that is ….
III = IXX – (A x d2)

F
Issue No. 1 Page 290 of 348
ST. THERESE- MTC COLLEGES SEAMANSHIP 6
Iloilo, Philippines (Advance Trim, Stability and Stress)
STUDENT LEARNING MODULE
Revision No. 1 Effectivity date: Reviewed by: Approved by:

10 August 2020 QMR President

I I
d
X X
Figure 13.7

For this formula to be true the two axis must be parallel and one of them must pass through
the centroid of the area concerned.

Process me
Example 1

A box-shaped vessel has length 30 meters and breadth of 8 meters. Calculate the following:

a. The moment of inertia for all the axis of rotation


b. The total moment of inertia about the two axis passing through the center of
flotation using parallel axis theorem.
IL Y
Illustration:

B=8 m
F
I I
d

X X
IL Y
Figure 13.8
Solution (a):

L B3 30 m8 m3 15360
III = = = =1,280 m4
12 12 12
3 3
BL 8 m30 m 216 , ,000
ILL = = = = 18,000m4
12 12 12
3 3
LB 30 m8 m 15,360
IXX = = = = 5,120 m4
3 3 3
3 3
BL 8 m30 m 216,000
IYY = = = = 72,000 m4
3 3 3 Y
L = 30
Solution (b): IL
B=8m
Using Parallel axis theorem:
F
III = IXX – (A x d2) I I
Issue No. 1 Page 291 of 348
ST. THERESE- MTC COLLEGES SEAMANSHIP 6
Iloilo, Philippines (Advance Trim, Stability and Stress)
STUDENT LEARNING MODULE
Revision No. 1 Effectivity date: Reviewed by: Approved by:

10 August 2020 QMR President


where d = 4 m
X d=4m
X
d = 15 m IL
III = IXX – (A x d2) Y
Figure 13.9
III = 5120 m4 – (30m x 8mx 4m2)
III = 5120 m4 – 3840 m4
III = 1,280 m4

Using Parallel axis theorem

ILL = IYY – (A x d2) where: d = 15 m

ILL = 72,000 m4 – (30m x 8m x 15m2)


ILL = 72,000 m4 – 54,000m4
ILL= 18,000 m4

13.4 Calculating the Moment of Inertia of a Water Plane Area of a Boxed-


shaped Vessel with Bilged Side Compartment
L

F
I I
B l CL

b
X X
A - amidships
Figure 13.10

When an amidships side compartment extending upwards the full depth of the vessel
becomes bilged the center of flotation (F) will move off the center line, its new position with
reference to side XX (axis XX) may be calculated by taking moments of area about one side.
This will give the distance, d, between the two axis to be considered.

In order to calculate the BM in the bilged condition it will be necessary to calculate the new
value of the transverse moment of inertia of the remaining intact water plane area about the
new longitudinal axis of rotation passing through the new position of the center of flotation.
This is achieved by using the parallel axis theorem whereby: III = IXX – (A x d2) where:
3 3
LB lb
(a) III = -
3 3
Issue No. 1 Page 292 of 348
ST. THERESE- MTC COLLEGES SEAMANSHIP 6
Iloilo, Philippines (Advance Trim, Stability and Stress)
STUDENT LEARNING MODULE
Revision No. 1 Effectivity date: Reviewed by: Approved by:

10 August 2020 QMR President


And the intact water-plane area is given by:
(b) A = (LB) – (lb)
Combining the formula will allow the transverse moment of inertia of the damaged water
plane area to be calculated:

III = ( L B3 lb 3
3

3 )
– ( ( LB−lb ) x d 2)

Once the value of I has been calculated, this is used to determine the BM in the bilged
condition.

Example 1

A box-shaped tanker barge has a length 90 m and breadth 15 m and floats at an even keel
draft of 4.50 m in salt water. An amidships side compartment of length 20 m extending in
from the side 5 meters in bilged. Calculate the following:

a. The initial BM value;


b. The BM in the damaged condition.

Solution for a:
L = 90m

F
I I
15 m
CL
l=20m

b=5m
X X
A - amidships
Figure 13.11

LB
3
90 m x 153 m
BMBOX = =
12V 12 x (90 m x 15 m x 4.50 m)

303,750 m4 303,750
= 3 =
12 x (6,075 m ) 72,900

BMBOX = 4.17 m

Solution for b:
1. Calculate first the distance of the new LCF from one side of the WPA (XX). Take the
moment of the area about the XX.
Issue No. 1 Page 293 of 348
ST. THERESE- MTC COLLEGES SEAMANSHIP 6
Iloilo, Philippines (Advance Trim, Stability and Stress)
STUDENT LEARNING MODULE
Revision No. 1 Effectivity date: Reviewed by: Approved by:

10 August 2020 QMR President


Volume (m2) Dist. From XX Moments (t-m)
Total area (90m x 15m) 1350 m2 7.5 m 10,125
Area bilged compartment (20m x 5 m) - 100 m2 2.5 m 250
Final 1250 m2 7.90 m 9,875 t-m

2. Calculate the transverse moment of inertia about the axis passing through the new
position of the LCF. (LCF is now 7.90 m from the side of XX – d 2).

III = ( L B3 lb 3
3

3 )
– ( ( LB−lb ) x d )
2

( 90 X3 15 − 20 3X 5 )
3 3
– ( ( 90 x 15¿−(20 x 5)) x 7.90 )
2
III =

III = ( 303,750
3

3 )
2500
– ( ( 1350−100 ) x 7.90 )
2

III =( 101,250−833.33 ) – ( 1250 x 7.902 )


III =100,416.67 – 78,012.5
III =22,404.17 m4

3. Calculate the BM for the bilged condition:


I
Formula: BM =
V
22,404,17 m4
BM =
90 m x 15 m x 4.50 m

22,404,17 m4
BM = = 3.69 m
6,075 m3

Do this

Activity 1

A box-shaped vessel, of length 120 m and breadth 20 m, floats in salt water on an even keel
at 6.50 m draft. KG = 4.95 m. the ship has a continuous centerline bulkhead that is
watertight. A compartment amidships, which is 16 m long and is empty, is bilged on one
side. Calculate the following:
b) New mean draft;
c) Shift of the center of buoyancy (BB1);
d) Second Moment of area in the bilged condition (IOZ);
e) New BM;
Issue No. 1 Page 294 of 348
ST. THERESE- MTC COLLEGES SEAMANSHIP 6
Iloilo, Philippines (Advance Trim, Stability and Stress)
STUDENT LEARNING MODULE
Revision No. 1 Effectivity date: Reviewed by: Approved by:

10 August 2020 QMR President


f) KM;
g) GM;
h) Angle of List
Write you solutions and answer in the space provided.

Write you solutions and answer in the space provided.

Do this
Issue No. 1 Page 295 of 348
ST. THERESE- MTC COLLEGES SEAMANSHIP 6
Iloilo, Philippines (Advance Trim, Stability and Stress)
STUDENT LEARNING MODULE
Revision No. 1 Effectivity date: Reviewed by: Approved by:

10 August 2020 QMR President


Activity 2

A box-shaped vessel 90 m long x 15 m wide, floats in salt water on an even keel at a draft
of 4.50 m. A centerline longitudinal watertight bulkhead extends from end to end and for
the full depth of the vessel. A compartment amidships on the port side is 15 m long, and
contains cargo with permeability μof 45%. This compartment is bilged. KG = 5.15 m.
Calculate the following:
a) New mean draft;
b) Shift of the center of buoyancy (BB1);
c) Second Moment of area in the bilged condition (IOZ);
d) New BM;
e) KM;
f) GM;
g) Angle of List

Write you solutions and answer in the space provided.

Write you solutions and answer in the space provided.


Issue No. 1 Page 296 of 348
ST. THERESE- MTC COLLEGES SEAMANSHIP 6
Iloilo, Philippines (Advance Trim, Stability and Stress)
STUDENT LEARNING MODULE
Revision No. 1 Effectivity date: Reviewed by: Approved by:

10 August 2020 QMR President

Assessment 1

A box-shaped tanker barge of length 100 m and 15 m wide and floats in salt water on an
even keel at 5.00 m draft. KG = 3.00 m. The barge has a continuous centerline bulkhead
Issue No. 1 Page 297 of 348
ST. THERESE- MTC COLLEGES SEAMANSHIP 6
Iloilo, Philippines (Advance Trim, Stability and Stress)
STUDENT LEARNING MODULE
Revision No. 1 Effectivity date: Reviewed by: Approved by:

10 August 2020 QMR President


that is watertight. An empty compartment amidships on the starboard side is 10 m long is
bilged. Calculate the following;
a) New mean draft;
b) Shift of the center of buoyancy (BB1);
c) Second Moment of area in the bilged condition (IOZ);
d) New BM;
e) KM;
f) GM;
g) Angle of List

Write you solutions and answer in the space provided.

Write your solution and answer in the space provided.


Issue No. 1 Page 298 of 348
ST. THERESE- MTC COLLEGES SEAMANSHIP 6
Iloilo, Philippines (Advance Trim, Stability and Stress)
STUDENT LEARNING MODULE
Revision No. 1 Effectivity date: Reviewed by: Approved by:

10 August 2020 QMR President

Assessment 2

A box-shaped vessel 50 m long x 10 m wide, floats on an even keel in salt water at a draft
of 4.00 m and a KG of 3.00 m. A compartment amidships is 12 m long, and contains cargo
with permeability μof 30%. This compartment is bilged.
Calculate the following:
a) New mean draft;
Issue No. 1 Page 299 of 348
ST. THERESE- MTC COLLEGES SEAMANSHIP 6
Iloilo, Philippines (Advance Trim, Stability and Stress)
STUDENT LEARNING MODULE
Revision No. 1 Effectivity date: Reviewed by: Approved by:

10 August 2020 QMR President


b) Shift of the center of buoyancy (BB1);
c) Second Moment of area in the bilged condition (IOZ);
d) New BM;
e) KM;
f) GM;
g) Angle of List

Write you solutions and answer in the space provided.

Write you solutions and answer in the space provided.


Issue No. 1 Page 300 of 348
ST. THERESE- MTC COLLEGES SEAMANSHIP 6
Iloilo, Philippines (Advance Trim, Stability and Stress)
STUDENT LEARNING MODULE
Revision No. 1 Effectivity date: Reviewed by: Approved by:

10 August 2020 QMR President

Lesson 14: Dry-docking and Grounding

Learning Module 14.1: Dry-docking and Grounding

Competence, Course Outcomes and Learning Outcomes

Competence:

Control trim, stability and stress

Course Outcome:

1. Explain the importance of maintaining stability during loading, unloading


and in-transit in various conditions.
2. Calculate the effect on trim and stability of a ship in the event of damage
to and consequent flooding of a compartment and countermeasures to be
taken.

Learning Outcomes:

At the end of the lesson, the student should be able to:

1. Explain the effects of stability in the event of dry-docking.


2. Calculate the following with given GM to ensure that the ship remains stable at the
point of taking the blocks overall:
– Minimum GM
– Maximum trim
– Virtual loss of GM and the draughts of the ship

Overview
Issue No. 1 Page 301 of 348
ST. THERESE- MTC COLLEGES SEAMANSHIP 6
Iloilo, Philippines (Advance Trim, Stability and Stress)
STUDENT LEARNING MODULE
Revision No. 1 Effectivity date: Reviewed by: Approved by:

10 August 2020 QMR President


It is requirement that all ships be dry-docked for inspection and maintenance below the
waterline. When a ship is being dry-docked additional forces acting at the keel take effect,
being the reaction or up thrust afforded by the blocks onto which the ship is being landed.
These factors can create undue loads on the stern structure and causes loss of stability of
the ship. This module investigates these effects.

Discussion

14.1 Stability during Dry-docking

When a ship is dry-docked, her support has to be transferred from the water to the keel
blocks and shores. She may be considered safe while she is waterborne, or once the shores
have been set up, but there is a danger that the she may become unstable during the
intervening period, which is often termed the “critical period”.

While the dock is being pumped out, the ship at first sinks bodily as the water level falls, but
as soon as she touches the keel blocks she stops sinking and the waterfalls around her. She
thus loses displacement so that weight, equal to the amount of the lost displacement, is
transferred to the blocks. As far as the ship’s stability is concerned, this weight is equivalent
to a force acting vertically upwards at the keel and it will decrease the metacentric height.
The latter must, sooner or later, become negative and if this were to happen before the
shores were properly set up, the ship might capsize in the dock. It is thus of the utmost
importance to keep full control of the ship during the critical period and to get the shores set
up as soon as possible. To assist in this, it is usual to have the ship trimmed a little by the
stern when she enters the dock, so that the heel of the stern post is the first part to touch
the blocks.
P

W F L
I
keel blocks

P Figure 14.1
Figure 14.1 illustrates what happens in the above case. As soon as the ship’s stern touches
the blocks the upward force, P, is the up thrust at the stern comes into existence and ‘I’ is
the distance of the center of flotation from aft. This force is small at first, but gradually
Issue No. 1 Page 302 of 348
ST. THERESE- MTC COLLEGES SEAMANSHIP 6
Iloilo, Philippines (Advance Trim, Stability and Stress)
STUDENT LEARNING MODULE
Revision No. 1 Effectivity date: Reviewed by: Approved by:

10 August 2020 QMR President


increases as the water-level falls and the ship’s bow comes down. The advantage of this is
that the decrease in metacentric height, caused by the force P, is more gradual than it
would be if the ship suddenly sat flat on the blocks fore and aft, so that we have more
control over the ship’s stability. Also, although the shores cannot be set up before ship
comes down flat on the blocks, we can start to put in the after shores loosely as soon as the
stern touches. By the time that the ship is right down on the blocks a large number of
shores are already in place, so that the remainder can be put in and all set up with the
minimum of delay. This decrease the risk of the ship falling over in the dock.

It is important to have the ship upright when she enters a dry dock. If she were not, this
could be due to one of two causes: a negative metacentric height (GM), or the weights on
board not being symmetrical about the center-line. In the first case, the ship would be
certain to fall over as soon as her keel touched the blocks. In the second, she might fall over
at some time during the critical period on account of the excess of weight on one side.

14.1.1 Procedure of dry-docking

1. When the ship enters the dry dock, it must have a positive metacentric height and is
usually trimmed by the stern and is floated into position.

W L
B

Figure 14.2a

2. The dock gates are then closed and water is pumped out of the block until the ship
touches the blocks aft. The rate of pumping out water is reduced as the stern is
almost about to touch the keel blocks. The reason is, it is from this stage of the
docking procedure when the stability of the ship starts getting critical. The interval of
time from when the stern takes the blocks to the moment when the entire ship’s
weight is borne by the blocks is called the Critical Period. It is now that the ship
will start to experience a loss of stability, hence the trim.

W L
B

Figure 14.2b
Issue No. 1 Page 303 of 348
ST. THERESE- MTC COLLEGES SEAMANSHIP 6
Iloilo, Philippines (Advance Trim, Stability and Stress)
STUDENT LEARNING MODULE
Revision No. 1 Effectivity date: Reviewed by: Approved by:

10 August 2020 QMR President

3. As more water is pumped out of the dock the true mean draught will start to reduce
as the ship experiences more and more support at the stern. The up-thrust afforded
by the blocks at the stern is termed the ‘P force’, this continues to increase as
buoyancy force reduces. Throughout the docking process the ship will displace a
progressively lessening volume of water as the true mean draught reduces and the P
force increases to provide more support for the ship (in effect, the P force takes over
supporting the ship and the role of the buoyancy force in supporting the ship
reduces). At this stage the aft draught will be reducing at a greater rate than what
the forward draught is increasing, the ship will be trimming by the as the overall true
mean draught reduces. The loss of stability will also be increasing as the P force
increases.
P

W L
B

Figure 14.2c

4. Eventually the ship will come to rest on the blocks along its entire length, this critical
instant denotes the end of the critical period, since for a flat bottomed ship the
problem of stability loss is no longer of concern.
P

W L
B

Figure 14.2d

5. After settling on the blocks forward and aft water continues to be pumped from the
dock and the draught reduces at the same rate forward and aft. The up-thrust P
becomes uniformly distributed along the ship’s length and continues to increase as
the effective buoyancy force reduces.
P

B
Issue No. 1 Page 304 of 348
ST. THERESE- MTC COLLEGES SEAMANSHIP 6
Iloilo, Philippines (Advance Trim, Stability and Stress)
STUDENT LEARNING MODULE
Revision No. 1 Effectivity date: Reviewed by: Approved by:

10 August 2020 QMR President

Figure 14.2e

6. When the dock becomes nearly empty and the ship is fully dry the up-thrust P will be
equal to the ship’s displacement having now replaced all the up-thrust afforded by
the buoyancy force.

Figure 14.2f

The following formula will give the ship’s metacentric height at any time during the process
of dry-docking:

P = the force acting upwards through the keel


KM = the KM corresponding to the actual displacement – P
W = the ship’s displacement entering the dock (the real/initial displacement)
P x KM
Method 1 –Virtual loss of GM (MM1) =
W
Righting moment = W x GM1x sin θ
P x KG
Method 2 –Virtual loss of GM (GG1) =
W −P

Righting moment = (W – P) x G1Mx sin θ


The force P is the difference between the displacement of the ship on entering the dock and
her displacement at the time for which we wish to calculate her GM. After the ship has come
flat on the blocks, this calculation is quite simple, since the two displacements will be those
for the respective mean drafts; that is

P= displacement at original draft – displacement at new draft


It is more difficult to find P during the critical period, after the ship’s stern has touched the
blocks, but before she comes flat on them. The most dangerous part of this period, and
hence the one with which we are most concerned, occurs at the instant before the ship
takes the blocks fore and aft. For this instant, P can be found approximately by the
following formula:

Where t = the trim, in centimeters, on entering the dock


Issue No. 1 Page 305 of 348
ST. THERESE- MTC COLLEGES SEAMANSHIP 6
Iloilo, Philippines (Advance Trim, Stability and Stress)
STUDENT LEARNING MODULE
Revision No. 1 Effectivity date: Reviewed by: Approved by:

10 August 2020 QMR President


l = the distance between the after block and the center of flotation

14.1.2 Calculating the P Force

.1 Calculation of P Force at any stage during dry-docking

Throughout the dry-docking procedure the true mean draught reduces as it would if the ship
were rising out of the water due to weights being discharged.

w(t)
Formula: Rise (cms) =
TPC

The P force may be considered to have the same effect on true mean draught as if a weight
P force(t )
had actually been discharged, therefore: Reduction in TMD (cms) = or
TPC

P force (t) = Reduction in TMD (cms) x TPC

This formula may be used to calculate the up thrust at the blocks at any stage in the
docking process since true mean draught is always reducing as water is taken out of the
dock.

.2 Calculation of P Force during the Critical Period when dry-docking

In the period between the ship touching the blocks aft (start of the critical period) and
touching the blocks forward and aft (the critical instant) the ship undergoes a change of
trim.

The change of trim at any stage during the critical period may be considered to be the same
as the change of trim that would have occurred had weight ‘w’ been discharged from a
position at the aft perpendicular equivalent to the up thrust P in tonnes. Consider the
w xd
formula: COT (cms) = MCT 1C

If the force P is considered to have the same effect as a weight discharged at the aft
perpendicular, then:

Px d cot ( cms ) x MTC 1 C M . C . T .1 C x t


COT (cms) = or P = or P =
MCT 1C Dist LCF foap l

14.1.3 Loss of Stability when dry-docking

Loss of stability commences as soon as the ship touches the blocks aft and continues to
worsen as the value of the P force increases. The maximum loss of GM of concern occurs
Issue No. 1 Page 306 of 348
ST. THERESE- MTC COLLEGES SEAMANSHIP 6
Iloilo, Philippines (Advance Trim, Stability and Stress)
STUDENT LEARNING MODULE
Revision No. 1 Effectivity date: Reviewed by: Approved by:

10 August 2020 QMR President


the instant immediately prior to the ship settling on the blocks forward and aft – this time
being termed the critical instant. Once the ship is flat on the blocks it will be in a safe
condition as the risk of heeling over as a result of becoming unstable will have passed (most
ship’s having a substantial area of flat bottom). For ships that have a relatively small
percentage of flat bottom area additional measures must also be taken such as using side
shores to support the ship in the upright condition when in the dry dock.

Process me
Example 1

A ship of 5,500 tonnes displacement enters a dry dock trimmed 0.40 m by the stern. KM is
7.60 m, KG of 6.00 m and MCT1C is 90 tonnes-meter. The center of flotation is 45 m from
aft. Calculate the effective metacentric height at the critical instant before the ship takes the
blocks overall and the moment of statical stability at 0.5º heel. Assume the trim at the
critical instant is zero.

Solution:

Calculate the Force acting upward (P):


M . C . T .1 C x t 90 t−m x 40 cm 3600t−m
P= = = = 80 tonnes
l 45 m 45
Method 1:

P x KM 80 tx7.6 m 608 t−m


a. Virtual loss of GM (MM1) = = = = 0.11 m
W 5,500 t 5,500 t

b. Original GM = KM – KG
= 7.60 m – 6.00 m
Original GM =1.60 m

c. GM1 = Original GM – Virtual loss of GM


GM1 = 1.60 m – 0.11 m
GM1 = 1.49 m
Righting moment = W x GM1x sin θ
= 5500 t x 1.49 m x sin 0.5º
Issue No. 1 Page 307 of 348
ST. THERESE- MTC COLLEGES SEAMANSHIP 6
Iloilo, Philippines (Advance Trim, Stability and Stress)
STUDENT LEARNING MODULE
Revision No. 1 Effectivity date: Reviewed by: Approved by:

10 August 2020 QMR President


Righting moment = 71.51 t-m

Method 2:

P x KG 80tx 6.0 m 480 t−m


a. Virtual loss of GM (GG1) = = = = 0.09 m
W −P 5,500−80 t 5,420
b. Original GM = KM – KG
= 7.60 m – 6.00 m
Original GM =1.60 m

c. GM1 = Original GM – Virtual loss of GM


GM1 = 1.60 m – 0.09 m
GM1 = 1.51 m
Righting moment = W x GM1x sin θ
= 5500 t x 1.51 m x sin 0.5º
Righting moment = 72.47 t-m

Thus, each of the two methods used gives a correct indication of the ship’s stability during
the critical period.

Example 2

A ship of 3200 tonnes displacement is 120 m long, KM is 7.00m, and KG 6.40 m. the center
of flotation is 5 m aft of amidships and MTC1C is 45 t-m. Calculate the maximum trim for
the ship to enter a dry dock if the metacentric height at the critical instant before the ship
takes the blocks forward and aft is to be not less than 0.40 m.

Solution:
Original GM = KM – KG
= 7.00 m – 6.40 m
Original GM = 0.60 m

Virtual loss = Original GM – Virtual GM


= 0.60 m – 0.40 m
Virtual loss = 0.20 m

Method 1:
P x KM
Virtual loss of GM =
W
Issue No. 1 Page 308 of 348
ST. THERESE- MTC COLLEGES SEAMANSHIP 6
Iloilo, Philippines (Advance Trim, Stability and Stress)
STUDENT LEARNING MODULE
Revision No. 1 Effectivity date: Reviewed by: Approved by:

10 August 2020 QMR President

Virtual loss x W
P=
KM
0.20 m x 3200 t 640 t−m
Maximum P = = = 91 tonnes
7.00 m 7.00 m
But
M . C . T .1 C x t
P= l
or
Px l
Maximum t =
MCT 1C

L 120 m
Hence: l = 2 = 2 = 60 – 5 m (center of flotation is 5 m aft of amidships) = 55 m
91t x 55 m
Maximum trim =
45 t−m
5005t−m
=
45 t−m
Maximum trim = 111 cm by the stern

Method 2:
P x KG
Virtual loss of GM =
W −P
P x 6.40 m
0.20 m =
3200 t−P
Transposition:
P x 6.40 m
0.20 m =
3200 t−P

(3200 t x 0.2) – (0.2 P) =6.40P

640 t-m – 0.2 P =6.40P

640 t-m = 6.40P + 0.2P

640 t-m = 6.60P

640 t−m
Maximum P = = 96.97 tonnes
6.60 m
But
Issue No. 1 Page 309 of 348
ST. THERESE- MTC COLLEGES SEAMANSHIP 6
Iloilo, Philippines (Advance Trim, Stability and Stress)
STUDENT LEARNING MODULE
Revision No. 1 Effectivity date: Reviewed by: Approved by:

10 August 2020 QMR President


M . C . T .1 C x t
P=
l
or
Px l
Maximum t =
MCT 1C
L 120 m
hence: l = 2 = 2 = 60 – 5 m (center of flotation is 5 m aft of amidships) = 55 m
96.97 t x 55 m
Maximum trim =
45 t −m
5333.35t−m
=
45 t−m
Maximum trim = 118.5 cm by the stern

There are therefore two possible answers to this question, depending on the method of
solution used. The reason for this is that although the effective metacentric height at the
critical instant in each case will be the same, the righting moments at equal angles of heel
will not be the same.

Example 3

A ship of 5000 tonnes displacement enters a dry dock trimmed 0.35 m by the stern. KM is
7.50 m, KG 6.0 m and MCT1C 110 t-m. The center of flotation is 55 m from aft. Find the
effective metacentric height at the critical instant before the ship takes the blocks overall,
assuming that the transverse metacenter rises 0.085 m.
M . C . T .1 C x t
Solve for P: P= 0.35 m convert to cms.
l
110t−m x 35 cm
P=
55 m
3850t
P= = 70 tonnes
55
Method 1:
P x KM 70tx 7.50 m 525t−m
a. Virtual loss of GM(MM1)= = = = 0.105 m
W 5,000t 5,000
b. New KM = Original KM + Rise of M
= 7.50 m – 0.085 m
New KM = 7.585 m

c. GM = New KM - KG
= 7.585 m – 6.000 m
GM = 1.585 m
Issue No. 1 Page 310 of 348
ST. THERESE- MTC COLLEGES SEAMANSHIP 6
Iloilo, Philippines (Advance Trim, Stability and Stress)
STUDENT LEARNING MODULE
Revision No. 1 Effectivity date: Reviewed by: Approved by:

10 August 2020 QMR President


d. New GM = GM – Virtual loss of GM
= 1.585 m – 0.105 m
New GM = 1.48 m

Method 2:
P x KG 70 tx 6.0 m 420 t−m
a. Virtual loss of GM (GG1) = = = = 0.085 m
W −P 5,000t−70 t 4930 t
b. New KG = Old KG + Virtual loss of GM
New KG = 6.000 m + 0.085 m = 6.085 m

c. New GM = New KM – New KG


New GM = 7.585 m – 6.085 m =1.500 m
The Virtual Loss of GM after Taking the Blocks Overall

When a ship takes the blocks overall, the water level will then fall uniformly about the ship
and for each centimeter fallen by the water level P will be increased by a number of tonnes
equal to the TPC. Also the force P at any time during the operation will be equal to the
difference between the weight of the ship and the weight of water she is displacing at that
time.

Example 4

A ship of 8000 tonnes displacement enters a dry dock on an even keel. KM is 8.00 m, KG
7.500 m and TPC of 50 tonnes. Calculate the virtual loss of metacentric height after the ship
has taken the blocks and the water has fallen another 0.30 m.

Solution: P = TPC x Reduction in draft in cm


P = 50 x 30 cms
P = 1500 tonnes
Method 1:
P x KM
Virtual loss (MM1) =
W
1500t x 8.00 m 12,000 m
= =
8000t 8000
Virtual loss (MM1) = 1.50 m

Method 2:
P x KG
Virtual loss (GG1) =
W −P
Issue No. 1 Page 311 of 348
ST. THERESE- MTC COLLEGES SEAMANSHIP 6
Iloilo, Philippines (Advance Trim, Stability and Stress)
STUDENT LEARNING MODULE
Revision No. 1 Effectivity date: Reviewed by: Approved by:

10 August 2020 QMR President


1500 t x 7.50 m 11,250 m
= =
8000 t−1500T 6500
Virtual loss (GG1) = 1.73 m
Example 5

A ship entered a dry dock with drafts of 3.00 meters forward and 3.50 meters aft. At this
draft, her displacement was 4,650 tonnes with a KG of 7.40 meters, KM 9.20 meters, MCT1C
148 tonnes-meters while the center of flotation was 72 meters from aft. Calculate the
following:
1. The GM at the instant before the ship came flat on the blocks.
2. The GM when the water had fallen so that the draft read 2.65 meters, her
displacement was 3,650 tonnes and KM of 10.0 meters.

Solution:
1. When entering the dock
a. Calculate first the trim when entering the dock:
Trim = Forward draft Aft draft
Trim = 3.00m 3.50 m = 0.50 m = 50 cm
b. Calculate Force acting upward (P):
M . C . T .1 C x t 148t−m x 50 cm 7400
P= = = = 103 tonnes
l 72m 72
c. Find for the Virtual rise of G:
P x KM
Virtual rise of G (or virtual loss of KM) =
W
103t x 9.2 m 947.6 t−m
= =
4650t 4650t
Virtual rise of G (or virtual loss of KM) = 0.20 m
d. Calculate Virtual KG:
Virtual KG = KG + Virtual Rise of G
= 7.40 m+ 0.20 m
Virtual KG = 7.60 m
e. Calculate Virtual GM:
Virtual GM = Virtual KG KM
= 7.60 m 9.20 m
Virtual GM = 1.60 m
2. GM when water had fallen:
a. Calculate the displacement (P):
P= displacement at original draft – displacement at new draft
Issue No. 1 Page 312 of 348
ST. THERESE- MTC COLLEGES SEAMANSHIP 6
Iloilo, Philippines (Advance Trim, Stability and Stress)
STUDENT LEARNING MODULE
Revision No. 1 Effectivity date: Reviewed by: Approved by:

10 August 2020 QMR President


= 4,650 tonnes – 3,650 tonnes
P = 1,000 tonnes
b. Find for the Virtual rise of G:
P x KM
Virtual rise of G =
W
1,000t x 10 m 10,000t−m
= =
4650t 4650 t
Virtual rise of G = 2.15 m
c. Calculate Virtual KG:
Virtual KG = KG + Virtual Rise of G
= 7.40 m+ 2.15 m
Virtual KG = 9.55 m
d. Calculate Virtual GM:
Virtual GM = KM - Virtual KG
= 10.00 m – 9.55 m
Virtual GM = 0.45 m
Stability Requirements for Dry Docking
When a ship enters a dry dock she must have:
positive initial GM
Figure 14.3
Be upright
And trimmed slightly, usually by the stern

14.2 Grounding
When a ship runs ashore, her metacentric
height will decrease or become negative as
in dry-docking, but the exact effect of this
on her stability is almost unpredictable. It
will vary according to the nature of the
ground, how the ship is placed on the
bottom, what damage she has sustained
and the nature and state of the tides. In
practice, we can only attempt to get the
ship afloat again as soon as possible, if it
appears safe to do so.

When the ship runs aground, its stability is Figure 14.4

affected in more or less the same way as with dry docking, except that, because there are
no bottom and sides to restrict the heel, it might even capsize. The pressure exerted on the
bottom results in a smaller displacement than the original displacement causing KM and KN
Issue No. 1 Page 313 of 348
ST. THERESE- MTC COLLEGES SEAMANSHIP 6
Iloilo, Philippines (Advance Trim, Stability and Stress)
STUDENT LEARNING MODULE
Revision No. 1 Effectivity date: Reviewed by: Approved by:

10 August 2020 QMR President


sinφ to change. If these decrease so much that the ship has a negative initial stability, it
could capsize during recovery and certainly if a large towing force is exerted in the
transverse direction. It is even worse if the ship runs aground asymmetrically and lists.
However, there is no immediate risk of capsizing if the ship runs aground over its full width.

Process me
Example 1

A ship of 8500 tonnes displacement takes the ground on a sand bank on a falling tide at an
even-keel draft of 6.20 m, KG 4.50 m. the predicted depth of water over the sandbank at
the following low water is 4.20 m. Calculate the GM at this time assuming that the KM will
then be 5.00 m and that the mean TPC is 15 tonnes.

Solution: P = TPC x fall in water level in cm

where: draft = 6.20m = 620 cm and depth of water over sandbank = 4.20m = 420 cm
Fall in water level = 620 cm – 420 cm = 200 cm

then P = 15 x 200 cms


P = 3,000 tonnes

Figure 14.5 Figure 14.6


Method 1
P x KM
a. Virtual loss GM (MM1) =
W
Issue No. 1 Page 314 of 348
ST. THERESE- MTC COLLEGES SEAMANSHIP 6
Iloilo, Philippines (Advance Trim, Stability and Stress)
STUDENT LEARNING MODULE
Revision No. 1 Effectivity date: Reviewed by: Approved by:

10 August 2020 QMR President


3000t x 5.00 m 15,000 m
= =
8500 t 8500
Virtual loss (MM1) = 1.76 m

b. Virtual KM = Actual KM – Virtual loss of GM


= 5.00 m – 1.76 m
Virtual KM = 3.24 m
c. New GM = Virtual KM - KG
= 3.24 m – 4.50
New GM = – 1.26

Method 2
P x KG
a. Virtual loss (GG1) =
W −P
3000t x 4.50 m 13,500 m Note: this vessel has developed a
=
8500 t−3000 t = 5500
negative GM. Consequently she is
Virtual loss (GG1) = 2.45 m unstable. She would capsize if
transverse external forces such as
b. Virtual KG: wind and waves were to remove
her from zero angle of heel.
Virtual KG = KG + Virtual Rise of G
Suggest a change of loading to
= 4.50 m+ 2.45 m
reduce KG and make GM a
Virtual KG = 6.95 m
positive value greater than the
d. New GM = KM – Virtual KG minimum of 0.15 m.
= 5.00 m – 6.95 m
New GM = – 1.95 m

Do this

Activity 1 – Dry Docking Problem

A ship of 4,300 tonnes displacement enters a dry dock trimmed 60 centimeters by the stern.
KM is 8.40 m, KG 6.90 m and MCT1C 100 t-m. The center of flotation is 70 m from aft. Find
the effective metacentric height at the critical instant before the ship takes the blocks
overall, assuming that the transverse metacenter rises 0.105 m. (Use Method 1 in solving
this problem).
Required:
a. P force
b. Virtual loss of GM
c. New KM
Issue No. 1 Page 315 of 348
ST. THERESE- MTC COLLEGES SEAMANSHIP 6
Iloilo, Philippines (Advance Trim, Stability and Stress)
STUDENT LEARNING MODULE
Revision No. 1 Effectivity date: Reviewed by: Approved by:

10 August 2020 QMR President


d. Original GM
e. New GM (GM1)

Write you solutions and answer in the space provided.


Issue No. 1 Page 316 of 348
ST. THERESE- MTC COLLEGES SEAMANSHIP 6
Iloilo, Philippines (Advance Trim, Stability and Stress)
STUDENT LEARNING MODULE
Revision No. 1 Effectivity date: Reviewed by: Approved by:

10 August 2020 QMR President

Write you solutions and answer in the space provided.


Issue No. 1 Page 317 of 348
ST. THERESE- MTC COLLEGES SEAMANSHIP 6
Iloilo, Philippines (Advance Trim, Stability and Stress)
STUDENT LEARNING MODULE
Revision No. 1 Effectivity date: Reviewed by: Approved by:

10 August 2020 QMR President

Do this

Activity 2 – Dry Docking Problem

A ship of 4,200 tonnes displacement enters a dry dock trimmed 0.75 m by the stern. KM is
8.00 m, KG of 7.25 m and MCT1C is 120 tonnes-meter. The center of flotation is 60 m from
aft. Calculate the effective metacentric height at the critical instant before the ship takes the
blocks overall and the moment of statical stability at 1.5º heel. Assume the trim at the
critical instant is zero. (Use Method 1 in solving this problem).
Required:
a. P force
b. Virtual loss of GM
c. Original GM
d. New GM (GM1)
e. Righting moment
Write you solutions and answer in the space provided.
Issue No. 1 Page 318 of 348
ST. THERESE- MTC COLLEGES SEAMANSHIP 6
Iloilo, Philippines (Advance Trim, Stability and Stress)
STUDENT LEARNING MODULE
Revision No. 1 Effectivity date: Reviewed by: Approved by:

10 August 2020 QMR President

Write you solutions and answer in the space provided.


Issue No. 1 Page 319 of 348
ST. THERESE- MTC COLLEGES SEAMANSHIP 6
Iloilo, Philippines (Advance Trim, Stability and Stress)
STUDENT LEARNING MODULE
Revision No. 1 Effectivity date: Reviewed by: Approved by:

10 August 2020 QMR President

Do this

Activity 3 – Grounding Problem

A ship of 7,500 tonnes displacement takes the ground on a reef on a falling tide at an even-
keel draft of 7.30 m, KG 5.50 m. the predicted depth of water over the reef at the following
low water is 6.20 m. At this time assuming that the KM will then be 6.15 m and that the
mean TPC is 15 tonnes. (Use Method 2 to solve this problem).
Calculate the following:
a. Fall in water level
b. P force
c. Virtual loss (GG1)
d. Virtual KG
e. New GM
Write you solutions and answer in the space provided.

Write you solutions and answer in the space provided.


Issue No. 1 Page 320 of 348
ST. THERESE- MTC COLLEGES SEAMANSHIP 6
Iloilo, Philippines (Advance Trim, Stability and Stress)
STUDENT LEARNING MODULE
Revision No. 1 Effectivity date: Reviewed by: Approved by:

10 August 2020 QMR President

Assessment 1
Issue No. 1 Page 321 of 348
ST. THERESE- MTC COLLEGES SEAMANSHIP 6
Iloilo, Philippines (Advance Trim, Stability and Stress)
STUDENT LEARNING MODULE
Revision No. 1 Effectivity date: Reviewed by: Approved by:

10 August 2020 QMR President


A ship entered a dry dock with drafts of 4.00 meters forward and 4.65 meters aft. At this
draft, her displacement was 5,650 tonnes with a KG of 7.65 meters, KM 9.15 meters, MCT1C
130 tonnes-meters while the center of flotation was 70 meters from aft. Calculate the
following:
1. The GM at the instant before the ship came flat on the blocks.
1.1. Trim
1.2. Force acting upward (P)
1.3. Virtual rise of G.
1.4. Virtual KG
1.5. Virtual GM
2. The GM when the water had fallen so that the draft read 3.55 meters, her
displacement was 4,750 tonnes and KM of 10.10 meters.
2.1 Displacement (P)
2.2 Virtual rise of G
2.3 Calculate Virtual KG
2.4 Calculate Virtual GM
Issue No. 1 Page 322 of 348
ST. THERESE- MTC COLLEGES SEAMANSHIP 6
Iloilo, Philippines (Advance Trim, Stability and Stress)
STUDENT LEARNING MODULE
Revision No. 1 Effectivity date: Reviewed by: Approved by:

10 August 2020 QMR President

Lesson 15: Trim and Stability due to Flooding

Learning Module 15.1: Trim and Stability due to Flooding

Competence, Course Outcomes and Learning Outcomes

Competence:

Control trim, stability and stress

Course Outcome:

1. Explain the importance of maintaining stability during loading, unloading


and in-transit in various conditions.
2. Calculate the effect on trim and stability of a ship in the event of damage
to and consequent flooding of a compartment and countermeasures to be
taken.

Learning Outcomes:

At the end of the lesson, the student should be able to:

1. Calculate the effect on trim and stability of a ship in the event of damage to and
consequent flooding of a compartment and counter-measures to be taken.
Issue No. 1 Page 323 of 348
ST. THERESE- MTC COLLEGES SEAMANSHIP 6
Iloilo, Philippines (Advance Trim, Stability and Stress)
STUDENT LEARNING MODULE
Revision No. 1 Effectivity date: Reviewed by: Approved by:

10 August 2020 QMR President


Discussion

15.1 Added Weight Method

This assumes that the flood water entering the ship increases the displacement and effects
the ship’s KG by reason of the effect of added mass and (in some cases) the effect of the
introduced free surface. Essentially the problem is approached in the same way that would
apply when a tank is either partially or fully filled during routine ships operations.

Process me
Example 1

A box-shaped vessel has length 140 m, breadth 36 m and is on an even keel draught of
6.00 m in salt water. In the present condition the KG is 13.30 m. An empty amidships
compartment extending the full breadth and depth of the vessel 65 m in length is bilged.
Calculate:

a. The draught in the bilged condition;


b. The initial GM;
c. The GM in the bilged condition;
d. The moment of statical stability if the vessel is heeled to 5º.

Solution (a):
140 m 36 m

x Flood water
d =6 m

l= 65 m
Figure 15.1
1. Calculate the mass of flood water admitted into the compartment.
Let X equal the new draft after bilging.

Added mass of water = Compartment l x B x New draft after bilging x R.D.


= 65 m x 36 m x X m x 1.025 t/m3
Added mass of water = 2,399 X-tonnes
Issue No. 1 Page 324 of 348
ST. THERESE- MTC COLLEGES SEAMANSHIP 6
Iloilo, Philippines (Advance Trim, Stability and Stress)
STUDENT LEARNING MODULE
Revision No. 1 Effectivity date: Reviewed by: Approved by:

10 August 2020 QMR President

2. Calculate the initial displacement


Initial displacement = L x B x d x R.D.
= 140 m x 36 m x 6 m x 1.025 t/m3
Initial displacement = 30,996 tonnes

3. Calculate the new displacement


New displacement = L x B x X x 1.025t/m3
= 140 m x 36 m x X x 1.025t/m3
New displacement = 5,166 X-tonnes
4. Calculate the final draught
New displacement = Initial displacement + mass of flood water
5,166X-t = 30,996 t+ 2,399X-t
5,166X-t – 2,399X-t = 30,996 t
therefore: 2767X-t = 30,996 t
30,996 t
therefore: X= = 11.20 m (draught in the bilged condition)
2767 t

Solution (b):

1. Calculate the initial GM:


KMBOX= KB + BM
Draught 6.00 m
KB = = = 3.00 m
2 2

LB
3
140 m x 36 3 m 140 m x 46,656 m3 6,531,840 m4
BM = = = 3 = 3
12V 12 x (140 m x 36 m x 6 m) 12 x 30240 m 362,880 m
BM = 18.00 m

therefore: KMBOX= KB + BM
KM = 3.00 m + 18.00 m = 21.00 m - now you can calculate the GM.
GM = KM – KG
= 21.00 m – 13.30 m
GM = 7.70 m
Issue No. 1 Page 325 of 348
ST. THERESE- MTC COLLEGES SEAMANSHIP 6
Iloilo, Philippines (Advance Trim, Stability and Stress)
STUDENT LEARNING MODULE
Revision No. 1 Effectivity date: Reviewed by: Approved by:

10 August 2020 QMR President


Solution (c): Calculate for GM on the bilged condition

a. Calculate the final KG:

Initial Displacement= 30,996 tonnes

Mass of flood water= Compartment l x B x X x R.D.


= 65 m x 36 m x 11.20 m x 1.025 t/m3
Mass of flood water = 26,863.2 tonnes = 26,863.2 tonnes
X 11.20 m
Kg of the flood water = = = 5.60 m
2 2
l B3 65 x 363
Introduced free surface moments = x R.D. = x 1.025 t/m3
12 12
3
65 m x 46,656 m
= x 1.025 t/m3
12
3,108,456t−m
Free surface moments = = 259,038 t-m
12
Take moments about the keel:
Weight (t) KG (m) Moments (t-m)
Initial 30,996 13.30 412,246.80
Flood water 26,863.2 5.60 150,433.92
FSMs 259,038.0
Final 57,859.2 t KG=14.20 m 821,718.72
b. Calculate the final KM:
KMBOX = KB + BM
Draught 11.20 m
KB = = = 5.60 m
2 2
4
L B3 140 m X 363 6,531,840 m
BM = = = 3 = 9.64 m
12V 12 x (140 m x 36 m x 11.20 m) 677,376 m
therefore:
KM = 5.60 m + 9.64 m = 15.24 m - now you can calculate the GM.

Calculate for GM in the bilged condition:


GM = KM –KG = 15.24 m – 14.20 = 1.04 m - in the bilged condition
Solution (d):

a. Calculate the Righting moment (t-m):


Righting moment (t-m) = GZ x Displacement
Issue No. 1 Page 326 of 348
ST. THERESE- MTC COLLEGES SEAMANSHIP 6
Iloilo, Philippines (Advance Trim, Stability and Stress)
STUDENT LEARNING MODULE
Revision No. 1 Effectivity date: Reviewed by: Approved by:

10 August 2020 QMR President


Final displacement = 57,859.2 tonnes

At 5º, a small angle of heel: GZ = GM x Sin θ


∴Righting moment = (GM x Sinθ ) x Final Displacement
∴Righting moment = (1.04 m x Sin 5º) x 57,859.2 t
∴Righting moment = 5,244.47 tonnes
Example 2

A box-shaped vessel has length 75 m, breadth 12 m and is floating on an even keel draught
in salt water of 2.50 m. in this condition the KG is 3.00 m. An empty forward end
compartment (assuming amidships) of length 6.00 m extending the full breadth and depth
of the vessel is bilged.
Calculate the draughts in the flooded condition.
75 m 12 m

12 m 2.5 m

l=¿ 6 m
Figure 15.2
Volume of buoyancy lost
Volume of buoyancy gained

Solution: By the lost buoyancy (constant displacement) method:


KG remains constant;
Displacement (and volume of displacement) remains constant.

1. Calculate the new mean draft


Volume of Buoyancy Lost = Volume Buoyancy Gained
Let x = sinkage
Compartment l x B x Draft = ((L – l ) x 12) x
6 m x 12 m x 2.5 m = (75 m – 6 m x 12) x
180 m3 = (69 m2x 12) x
180 m3 = 828 x m2
x = 0.22 m
therefore:
Mean draught in the bilged condition = 2.50 m + 0.22 m = 2.72 m
Issue No. 1 Page 327 of 348
ST. THERESE- MTC COLLEGES SEAMANSHIP 6
Iloilo, Philippines (Advance Trim, Stability and Stress)
STUDENT LEARNING MODULE
Revision No. 1 Effectivity date: Reviewed by: Approved by:

10 August 2020 QMR President

2. Move the compartment to its actual position and calculate the trimming moment.
AP 12 m

75 m FP

F GL
X

l=¿ 34.5m foap B1


3m
B
12 m

l=¿ 6 m
Figure 15.3

Consider Figure 15.3


Initially the LCG and LCB are in the same longitudinal position at amidships (since the vessel
is on an even keel). The change of trim is caused by the loss of volume of buoyancy forward
which causes the LCB to move aft (B to B1). This creates a trimming moment given by:

Trimming moment (t-m) = W x BB1


Because of the symmetry of the intact volume it follows that the trimming lever (BB 1) = 3
m, being half the length of the bilged compartment.
l 6m
BB1= =
2 2 = 3.00 m
∴Trimming moment = (L x B X Draft x RD) x BB1
= (75 m x 12 m x 2.50 m 1.025 t/m3) x 3.00 m
= 2306.25 t x 3 m
∴Trimming moment = 6,918.75 t-m (Displacement remains constant)

3. Calculate the MCT1C:


W x GM L
MTC1C =
100 LBP
where: GML = KML – KG
First calculate the KML for bilged condition.
KML = KB + BML
Issue No. 1 Page 328 of 348
ST. THERESE- MTC COLLEGES SEAMANSHIP 6
Iloilo, Philippines (Advance Trim, Stability and Stress)
STUDENT LEARNING MODULE
Revision No. 1 Effectivity date: Reviewed by: Approved by:

10 August 2020 QMR President

Draught 2.72m
In the bilged condition: KB = = = 1.36 m
2 2
B L3
BML=
12V

The water plane are afforded by the bilged compartment has been lost, therefore:
3 3 4
BL 12 m x 69 m 3,942,108 m
BML = = 12 m x ¿ ¿ = 3 = = 146.00 m
12V 12 x 2250m 27000 m3

Therefore:
KML = KB + BML
KML = 1.36 m + 146.0 m = 147.36 m

GML = KML – KG
GML = 147.36 m – 3.00 m
GML = 144.36 m (KG remains constant)
Now calculate MCT1C:
W x GM L
MTC1C =
100 LBP
( 75 m x 12 m x 2.5 m x 1.025 ) x 144.36 m
MTC1C =
100 x 75 m
2306.25t x 144.36 m
MTC1C =
7500 m
332,930.25t−m
MTC1C = = 44.39 m
7500 m

Note: The LBP is 75 m, do not use the effective length of the remaining water plane by
mistake.

4. Calculate the change of trim (COT):


Trimming moment
Change of Trim =
MCT 1C
6,918.75t −m
Change of Trim = = 155.86 cms by the head
44.39 m

5. Calculate change of draft forward and aft:


Issue No. 1 Page 329 of 348
ST. THERESE- MTC COLLEGES SEAMANSHIP 6
Iloilo, Philippines (Advance Trim, Stability and Stress)
STUDENT LEARNING MODULE
Revision No. 1 Effectivity date: Reviewed by: Approved by:

10 August 2020 QMR President


Consider Figure 15.3 because the water plane area afforded by the bilged
compartment forward is lost the LCF will move aft. Because of the symmetry of the
box shaped vessel the movement of the LCF aft will be the same as the movement
of the LCB (BB1), which is 3.00 m.

L 75 m
where: l= 2= 2 = 37.5 m – 3m = 34.5 m new position of the LCF.

l
Change of draft aft = x Change of trim (CT)
L
34.5 m
Change of draft aft = x 155.86 cms
75 m
34.5 m x 155.86 cms
Change of draft aft =
75 m
5377.17 cms
Change of draft aft = = 71.70 cms = 0.72 m
75
Change of draft forward = COT – Change draft aft
= 155.86 cms – 71.70 cms
Change of draft forward = 84.16 cms = 0.84 m

Forward Aft
Final mean 2.72 m 2.72 m
Trim + 0.84 m - 0.72 m
Final 3.56 m 2.00 m

Example 3: With Watertight Flat – 100% Permeability

A box-shaped vessel has length 100 m, breadth 18 m and is floating on an even keel
draught in salt eater of 4.00 m. in this condition the KG is 6.80 m. an empty forward end
compartment (assuming amidships) of length 10 m below a watertight flat 3.00 m above the
keel and extending the full breadth of the vessel is bilged. Calculate the draught in the
flooded condition.

Calculate the draughts in the flooded condition.


100 m 18 m

X
Issue No. 1 Page 330 of 348
ST. THERESE- MTC COLLEGES SEAMANSHIP 6
Iloilo, Philippines (Advance Trim, Stability and Stress)
STUDENT LEARNING MODULE
Revision No. 1 Effectivity date: Reviewed by: Approved by:

10 August 2020 QMR President

18 m D=4 m
d=3

l=¿ 10
Figure 15.4
Volume of buoyancy lost
Volume of buoyancy gained

Solution: By the lost buoyancy (constant displacement) method:


KG remains constant;
Displacement (and volume of displacement) remains constant.

1. Calculate the new mean draft


Volume of Buoyancy Lost = Volume Buoyancy Gained
Let x = sinkage
Compartment l x B x dist. above keel = (L x B x x)
10 m x 18 m x 3.0 m = 100 m x 18 m x x
540 m3 = 1800 x m2
x = 0.30 m
therefore:
Mean draught in the bilged condition = 4.00 m + 0.30 m = 4.30 m

2. Move the compartment to its actual position and calculate the trimming moment.
AP 100 m FP 18 m

95 m
5m
W F GL L
X
4.3 m lcb=¿ 46.625 m B1 B
18 m
3m
3.375 m

l=¿ 10
Figure 15.5

Consider Figure 15.5


Initially the LCG and LCB are in the same longitudinal position at amidships (since the vessel
is on an even keel). The change of trim is caused by the loss of volume of buoyancy forward
which causes the LCB to move aft (B to B1). This creates a trimming moment given by:

Trimming moment (t-m) = W x BB1

It is necessary to take moments of volume about the AP to find the new LCB.
Volume (m3) lcb foap (m) Moments (m4)
Total volume (100m x 18m x 4.30 m) 50m 387,000 m4
below final WL 7,740
Volume of DB lost (10m x 18m x 3m) 95m - 51,300 m4
- 540
Issue No. 1 Page 331 of 348
ST. THERESE- MTC COLLEGES SEAMANSHIP 6
Iloilo, Philippines (Advance Trim, Stability and Stress)
STUDENT LEARNING MODULE
Revision No. 1 Effectivity date: Reviewed by: Approved by:

10 August 2020 QMR President


Final 7,200 m3 46.63m 335,700 m4

LCB moves aft from a positive 50 m foap to 46.63 m foap.


Trimming lever = 50m – 46.63 m = 3.37 m

∴Trimming moment = (L x B X Draft x RD) x BB1


= (100 m x 18 m x 4.00 m x 1.025 t/m3) x 3.37 m
= 7,380 t x 3.37 m
∴ Trimming moment = 24,870.6 t-m (Displacement remains constant)

3. Calculate the MCT1C:


W x GM L
MTC1C =
100 LBP
where: GML = KML – KG
First calculate the KML for bilged condition.
KML = KB + BML

Draught
At the final waterline KB ≠
2
Consider the shape of the new intact underwater volume of the vessel in Figure 15.5. To
calculate the KB for the bilged condition take ‘moments of volume’ about the keel.

Volume (m3) KB (m) Moments (m4)


Total volume before (100m x 18m x 4.30m) 4.3 16,641
final WL 7,740 2
Volume of DB lost (10m x 18m x 3m) 3 - 810
- 540 2
Final 7200 m3 2.20 m 15,831 m4

In the bilged condition


The water plane area afforded by the bilged compartment has remained intact due to the
watertight flat, therefore:
3 4
BL
3
18 m x 1003 18 m x 1,000,000 m 18,000,000m
BML = = = 3 =
12V 12(100 x 18 x 4 m) 12 x 7200 m 86,400 m3
BML =208.33 m

Therefore:
KML = KB + BML
KML = 2.20 m + 208.33 m = 210.53 m
Issue No. 1 Page 332 of 348
ST. THERESE- MTC COLLEGES SEAMANSHIP 6
Iloilo, Philippines (Advance Trim, Stability and Stress)
STUDENT LEARNING MODULE
Revision No. 1 Effectivity date: Reviewed by: Approved by:

10 August 2020 QMR President


GML = KML – KG
GML = 210.53 m – 6.80 m
GML = 203.73 m -------KG remains constant)

Now calculate MCT1C:


W x GM L
MTC1C =
100 LBP
( 100 m x 18 m x 4 m x 1.025 ) x 203.73 m
MTC1C =
100 x 100 m
7,380t x 203.73 m
MTC1C =
10,000 m
1,503,527.4 t−m
MTC1C = = 150.35 m
10,000 m

4. Calculate change of trim (COT):

Trimming moment
Change of Trim =
MCT 1C
24,870.6 t−m
Change of Trim = = 165.42 cms by the head
150.35 m

5. Calculate change of draft forward and aft:


Because the water plane area has remained intact the position of the LCF remains
unchanged at amidships. Therefore,
165.42cms
Change of draft aft (Ta) =Change of draft forward (Tf) =
2
= 82.71 cms
= 0.83 m
Apply Ta and Tf to the final mean draught of 4.30 m.

Forward Aft
Final mean 4.30 m 4.30 m
Trim + 0.83 m - 0.83 m
Final 5.13 m 3.47 m
Issue No. 1 Page 333 of 348
ST. THERESE- MTC COLLEGES SEAMANSHIP 6
Iloilo, Philippines (Advance Trim, Stability and Stress)
STUDENT LEARNING MODULE
Revision No. 1 Effectivity date: Reviewed by: Approved by:

10 August 2020 QMR President

Do this

Activity 1

A box-shaped vessel has length 160 m, breadth 36 m and is on an even keel draught of
6.50 m in salt water. In the present condition the KG is 12.75 m. An empty amidships
compartment extending the full breadth and depth of the vessel 55 m in length is bilged.

Calculate the following:


1. The draught in the bilged condition:
1.1 Mass of flood water admitted into the compartment (Added mass of water)
1.2 Initial displacement
1.3 New displacement
1.4 Final draught
2. The initial GM:
2.1 KB
2.2 BM
2.3 KM
2.4 Initial GM
3. GM in the bilged condition:
3.1 Mass of flood water
3.2 Kg of the flood water
3.3 Free Surface Moments
3.4 KB
3.5 BM
3.6 KM
3.7 GM
4. The moment of statical stability if the vessel is heeled to 7º.

Write your answer in the space provided.


Issue No. 1 Page 334 of 348
ST. THERESE- MTC COLLEGES SEAMANSHIP 6
Iloilo, Philippines (Advance Trim, Stability and Stress)
STUDENT LEARNING MODULE
Revision No. 1 Effectivity date: Reviewed by: Approved by:

10 August 2020 QMR President

Write you solutions and answer in the space provided.

Write you solutions and answer in the space provided.


Issue No. 1 Page 335 of 348
ST. THERESE- MTC COLLEGES SEAMANSHIP 6
Iloilo, Philippines (Advance Trim, Stability and Stress)
STUDENT LEARNING MODULE
Revision No. 1 Effectivity date: Reviewed by: Approved by:

10 August 2020 QMR President

Do this
Issue No. 1 Page 336 of 348
ST. THERESE- MTC COLLEGES SEAMANSHIP 6
Iloilo, Philippines (Advance Trim, Stability and Stress)
STUDENT LEARNING MODULE
Revision No. 1 Effectivity date: Reviewed by: Approved by:

10 August 2020 QMR President


Activity 2

A box-shaped vessel has length 120 m, breadth 20 m and is floating on an even keel
draught in salt eater of 6.00 m. in this condition the KG is 7.80 m. An empty forward end
compartment (assuming amidships) of length 12 m below a watertight flat 3.50 m above the
keel and extending the full breadth of the vessel is bilged. Calculate the following using the
lost buoyancy (constant displacement) method:

1. New Mean draft in bilged condition


2. LCB foap
3. Trimming lever
4. Trimming moment
5. KB
6. BML
7. KML
8. GML
9. MCT1C
10. Change of Trim (COT)
11. Change of draft aft (Ta) in meters
12. Change of draft forward (Tf) in meters
13. Final draught in the flooded condition
13.1 Forward
13.2 Aft
Write your answer in the space provided.

Write your answer in the space provided.


Issue No. 1 Page 337 of 348
ST. THERESE- MTC COLLEGES SEAMANSHIP 6
Iloilo, Philippines (Advance Trim, Stability and Stress)
STUDENT LEARNING MODULE
Revision No. 1 Effectivity date: Reviewed by: Approved by:

10 August 2020 QMR President

Write your answer in the space provided.


Issue No. 1 Page 338 of 348
ST. THERESE- MTC COLLEGES SEAMANSHIP 6
Iloilo, Philippines (Advance Trim, Stability and Stress)
STUDENT LEARNING MODULE
Revision No. 1 Effectivity date: Reviewed by: Approved by:

10 August 2020 QMR President

Assessment

A box-shaped vessel has length 90 m, breadth 12 m and is floating on an even keel draught
in salt water of 3.50 m. in this condition the KG is 4.00 m. An empty forward end
compartment (assuming amidships) of length 8.00 m extending the full breadth and depth
of the vessel is bilged.
Issue No. 1 Page 339 of 348
ST. THERESE- MTC COLLEGES SEAMANSHIP 6
Iloilo, Philippines (Advance Trim, Stability and Stress)
STUDENT LEARNING MODULE
Revision No. 1 Effectivity date: Reviewed by: Approved by:

10 August 2020 QMR President


By the lost buoyancy (constant displacement) method, calculate the following:
1. Calculate the new mean draft in the bilged condition
2. Trimming moment
3. KB in the bilged condition
4. BML
5. MCT1C
6. Change of trim (COT)
7. Change of draft aft
8. Change of draft forward
9. Final draft aft
10. Final draft forward
11.
Write your solutions and answer in the space provided.

Write your solutions and answer in the space provided.

Write your solutions and answer in the space provided.


Issue No. 1 Page 340 of 348
ST. THERESE- MTC COLLEGES SEAMANSHIP 6
Iloilo, Philippines (Advance Trim, Stability and Stress)
STUDENT LEARNING MODULE
Revision No. 1 Effectivity date: Reviewed by: Approved by:

10 August 2020 QMR President

Oral Assessment (Final)

Explain the following and make a video clip wearing upper uniform.

1. Shearing stress
2. Bending moment
3. Loadline

Oral presentation on the chosen research topic directions (this is a video oral presentation):

1. In this activity, you will work a one person only.


2. On a paper, please write down what you think is your idea regarding your chosen
research topic. Please accomplish this task in 3 minutes.
Issue No. 1 Page 341 of 348
ST. THERESE- MTC COLLEGES SEAMANSHIP 6
Iloilo, Philippines (Advance Trim, Stability and Stress)
STUDENT LEARNING MODULE
Revision No. 1 Effectivity date: Reviewed by: Approved by:

10 August 2020 QMR President


3. When done, please post your output on Power Point Slides and prepare to present.
4. Each person will be given at least 2 minutes to present their outputs.
5. During the presentation, please explain briefly why you think you listed items that
constitute the terms in your chosen research topic.

RUBRICS FOR GRADING ORAL ASSESSMENT

CRITERIA SCORE
Content (40%)
 The contetnt clearly incorporates salient points discussed about the
topic
Creativity and overall presentation (30%)
 The presentation showcases learner’s creativity and originality
Other elements (15%)
 The group effectively use gestures, eye contact, movements, and
facial expressions to convey the message.
 The speaker speaks audibly and clearly.
 The speaker uses props, and costumes appropriately and effectively.
Mastery (15%)
 The pair has mastered the concepts and delivered it well.

Research/Output

Research your OUTPUT for the following topics and explain it:

1. Torsional stress
2. Calculation of vessel condition after flooding (passenger vessel and cargo ship)

Power Point Presentation, make a 5 minutes presentation of the chosen research topic. The
format of the slides must be:

- Slide 1: Title of the chosen research topic and members


– Slide 2: Justification for choosing the topic
– Slide 3: Existing problems, issues, gaps relevant to the chosen topic
- Font Size and Style: Title: Arial 36-40” Body: 24-28”
Issue No. 1 Page 342 of 348
ST. THERESE- MTC COLLEGES SEAMANSHIP 6
Iloilo, Philippines (Advance Trim, Stability and Stress)
STUDENT LEARNING MODULE
Revision No. 1 Effectivity date: Reviewed by: Approved by:

10 August 2020 QMR President


- Animation and Design: Free Style
- Follow 7 x 7 rule (7 words and 7 lines in a slide)

RUBRIC FOR GRADING POWER POINT PRESENTATION OF A CHOSEN RESEARCH


TOPIC

CRITERIA SCORE
CONTENT (40%)
 The content clearly incorporates salient points discussed about
the topic.
SLIDE CREATION (15%)
 Presentation flows well and logically. Presentation reflects
extensive use of tools in a creative way.
SLIDE TRANSITIONS (15%)
 Transitions are smooth. Transitions enhance the presentation
PICTURES, CLIP ART BACKGROUND (15%)
 Images are appropriate. Layout is pleasing to the eye.
MECHANICS (15%
 No spelling errors. No grammar errors. Text is in authors’ own
words

RESEARCH WORK

Submit a well-written literature review of your research topic and explain it:

1. Effect of flooding on trim


2. Theories affecting trim and stability

Submit a well-written literature review of your research topic. The table provided below is a
guide in making your literature review. You may add extra pages.

3. Draft article summary paragraph: 3-4 paragraphs of these will become your short
literature review.
4. Create an order for your paragraph summaries (try this with just 3-4 paragraphs that
relate). You have now created a draft of a short literature review.
5. Revise your short literature review to include paragraph introduction, body, and
conclusions/synthesis.

ARTICLE SUMMARY TABLE (Related Literature and Studies)


Issue No. 1 Page 343 of 348
ST. THERESE- MTC COLLEGES SEAMANSHIP 6
Iloilo, Philippines (Advance Trim, Stability and Stress)
STUDENT LEARNING MODULE
Revision No. 1 Effectivity date: Reviewed by: Approved by:

10 August 2020 QMR President

1 2
 Reference
 Aim/Objective/Scope
 Participants and sampling
 Context and framework
 Results or findings
 Implications Practice to Research
 Your Comments

Rubric for grading research work on reviewing literature

CRITERIA SCORE
INTRODUCTION (10%)
 Clear overview of paper, demonstrates importance of topic.
BODY (40%)
 Balanced viewpoint: Objective, balanced view from various
perspectives. Coherent theme: Each cited study related to
the topic and to other studies.
 Depth and breadth of research: Variety of studies and
attention to detail about the topic.
 Analysis: Collection of studies analyzed for differences and
commonalities about the topic.
CONCLUSION AND SYNTHESIS (10%)
 Information synthesized and brought to a logical
conclusion.
ORGANIZATION AND ALIGNMENT (15%)
 Organization and Alignment: Information logically
organized with good flow. Issues threaded throughout
Issue No. 1 Page 344 of 348
ST. THERESE- MTC COLLEGES SEAMANSHIP 6
Iloilo, Philippines (Advance Trim, Stability and Stress)
STUDENT LEARNING MODULE
Revision No. 1 Effectivity date: Reviewed by: Approved by:

10 August 2020 QMR President


paper.
MECHANICS, APA, REFERENCES (25%)
 Correct spelling, punctuation, sentence structure, word
usage.
 Correct use of APA in body of paper References correctly.
typed, appropriate number and quality

References

T1 – Dokkum, Klass Van… [et.al]. (2018). Ship stability. 6 th ed. The Netherlands:
DOKMAR Maritime Publishers BV.
T2 – Dokkum, Klass Van… [et.al]. (2018).Ship stability hydrostatic particulars
Delfship and Elandsgracht. 6th ed. The Netherlands: DOKMAR Maritime
Publishers BV.
T3 – Abangan, Emeterio B. (2018). Trim, stability and stress – Seamanship II. Cebu
City: Jjack Wisdom Publishing.
T4 – Dokkum, Klass Van… [et.al]. (2016). Ship knowledge, ship design, construction
and operation. 6th ed. The Netherlands: DOKMAR Maritime Publishers BV.
T5 – Ship and operations management (2014). 3G E Learning FZLLC: Replika Press
PVT., Ltd.
T6 – Barrass, C.B. and Derrett, D. R. (2012). Ship stability for masters and mates.
7th ed. USA: Butterworth-Heinemann
T7 – Rhodes, Martin A. (2012). Ship stability for mates and masters. Scotland, UK:
Issue No. 1 Page 345 of 348
ST. THERESE- MTC COLLEGES SEAMANSHIP 6
Iloilo, Philippines (Advance Trim, Stability and Stress)
STUDENT LEARNING MODULE
Revision No. 1 Effectivity date: Reviewed by: Approved by:

10 August 2020 QMR President


Witherby Publishing Group Ltd.
T8 – Dokkum, H.J. (2011). Ship knowledge; design, construction and operation. 7 th
ed. The Netherlands: DOKMAR Maritime Publishers BV.
T9 – House, D.J. (2010). Elements of modern ship construction. Glasgow: Brown,
Son & Ferguson, Ltd. Nautical Publisher.

R1 – IMO. (2017). STCW: Including 2010 Manila amendments: STCW convention


and STCW code. London: International Maritime Organization.
R2 – IMO. (2014). Model Course 7.01: Master and Chief Mate 2014 ed. London:
International Maritime Organization.
R3 – IMO. (2014). SOLAS consolidated edition 2014.London: International Maritime
Organization
R4 – IMO. (2011). STCW Including 2010 Manila amendments. London: International
Maritime Organization
R5 – IMO. (2008). International Code on Intact Stability, 2008 (2008 IS Code)
London: International Maritime Organization.
R6 – Derret, D.R. (2000). Ship stability for masters and mates. Reed Educational and
Professional Publishing Ltd.: Butterworth-Heinemann

Web and online links

 Advanced Ship Stability – Simpson’s First Rule (Area Calculation). Retrieved July 24,
2020 from https://www.youtube.com/watch?v=rZ6A4lj8Kac
 Ship Stability – Combining Simpson’s First and Second Rule to Calculate Area.
Retrieved July 29, 2020 from https://www.youtube.com/watch?
v=05yqp6WjKNw&t=250s
 Ship Stability – Use of Simpson’s Third Rule to Calculate the Area. Retrieved July 29,
2020 from https://www.youtube.com/watch?v=-uO_wQXCpDg
 Ship Stability – Effect of density on draft and displacement. Retrieved July 24, 2020
from https://www.youtube.com/watch?v=MB_dyf-AIJ4
 Ship Stability – Definition of Dock Water Allowance (DWA). Retrieved July 24, 2020
from https://www.youtube.com/watch?v=GUfnDbI_iaQ
 Ship Stability – Effect of density on draft and displacement Ex. 5 Problems 1-5
Retrieved July 24, 2020 from https://www.youtube.com/watch?v=0opFqkKO9qQ
 Stability Part 9 Free Surface Effect. Retrieved July 29,2020
https://www.youtube.com/watch?v=IIKR13b3eLQ&t=113s
Issue No. 1 Page 346 of 348
ST. THERESE- MTC COLLEGES SEAMANSHIP 6
Iloilo, Philippines (Advance Trim, Stability and Stress)
STUDENT LEARNING MODULE
Revision No. 1 Effectivity date: Reviewed by: Approved by:

10 August 2020 QMR President


 Stability Part 9 Free Surface Effect. Retrieved July 27, 2020 from
https://www.youtube.com/watch?v=IIKR13b3eLQ&t=113s
 Stability Part 8 Heel and List. Retrieved July 27, 2020 from
https://www.youtube.com/watch?v=zbL50a4xEYw
 Increase Due to Ship’s List. Retrieved July 24, 2020 from
https://www.youtube.com/watch?v=UCWaexmoc6Q
 Ship Stability Introduction to List Formula Example of List due to Loading. Retrieved
August 08, 2020 from https://www.youtube.com/watch?v=AvmkuN6g4Y8
 Lesson 7 Statical & Dynamical Stability Retrieved August 08, 2020 from
https://www.youtube.com/watch?v=N6OV0egyX-M
 Trim Problems (Part 1) – Advance Ship Stability for Mariners. Retrieved July 24, 2020
from https://www.youtube.com/watch?v=UXh-k1WCX8c
 Trim Problems (Part 2) – Advance Ship Stability for Mariners. Retrieved July 24, 2020
from https://www.youtube.com/watch?v=Fj_F2m9tDnY
 Trim Calculations – Part 3 (Advance Ship Stability for Mariners). Retrieved July 24,
2020 from (https://www.youtube.com/watch?v=SlrlnZvE-yo
 Curve of Statical Stability – Theory (Advance Ship Stability. Retrieved July 24, 2020
from https://www.youtube.com/watch?v=g5DdSSMYa-Q&t=222s
 Cross Curves of Statical Stability – Advance Ship Stability For Mariners. Retrieved July
24, 2020 fromhttps://www.youtube.com/watch?v=SOkwlijrTOA
 Creating Statical Stability Curves. Retrieved July 24, 2020 from
https://www.youtube.com/watch?v=mq7cE9YSy_w
 Curve of Statical Stability or GZ Curve, F3. Retrieved August 05, 2020 from
https://www.youtube.com/watch?v=d7S-ax-WL9I&t=13s
 Ship Stability – Trim Introduction Part 1. Retrieved August 05, 2020 from
https://www.youtube.com/watch?v=Jnzuj2jqjCE
 Ship Stability – Trim Introduction Part 2 Formula for MTC1C. Retrieved August 05,
2020 from https://www.youtube.com/watch?v=hEyGJjwPN38
 Ship Stability – Trim Introduction Part 3 Trim due to loading. Retrieved August 05,
2020 from https://www.youtube.com/watch?v=QzS2AgSfeC0&pbjreload=101
 Ship Stability – Trim Introduction Part 4 Trim due to discharging. Retrieved August 05,
2020 from https://www.youtube.com/watch?
v=mDm5R1LsqAs&list=TLPQMDUwODIwMjAYPpnNIkuk9Q&index=15
 Ship Stability Introduction to list formula Example of List due to discharge of weight.
Retrieved August 05, 2020 from https://www.youtube.com/watch?v=mFm_neXrBS0
 Ship Stability Use of Moment Table to find list. Retrieved August 05, 2020 from
https://www.youtube.com/watch?v=pPDzHoioYlY
 Ship Stability List Distribution of cargo to make the ship upright 2 nd mate examination.
Retrieved August 05, 2020 from https://www.youtube.com/watch?v=VssVxbMGoeY
 Using Simpson’s Rules to Calculate Dynamical Stability. Retrieved August 05, 2020
from https://www.youtube.com/watch?v=FIl5-LfU5Yo&t=220s
Issue No. 1 Page 347 of 348
ST. THERESE- MTC COLLEGES SEAMANSHIP 6
Iloilo, Philippines (Advance Trim, Stability and Stress)
STUDENT LEARNING MODULE
Revision No. 1 Effectivity date: Reviewed by: Approved by:

10 August 2020 QMR President


 Intact Stability Criteria. Retrieved August 05, 2020 from
https://www.youtube.com/watch?v=RzK4lTa-F8g
 Intact Stability Code 2008. All ships stability criteria discussed. Retrieved August 05,
2020 from https://www.youtube.com/watch?v=0PFN-ZD-Gio
 Rolling Period: USCG Questions. Retrieved August 05, 2020 from
https://www.youtube.com/watch?v=5lszBpFG8qM
 Rolling Period. Retrieved July 24, 2020 from https://www.youtube.com/watch?
v=mPdgFAHRVKg
 Synchronous Resonant Rolling. Retrieved August 05, 2020 from
https://www.youtube.com/watch?v=2LNxkgCo9ak
 Bilging. Retrieved July 24,2020 from https://www.youtube.com/watch?
v=4WeyXFLRh1w
 Bilging Amidships – Ship Stability (Theory and Numeral). Retrieved July 24, 2020 from
https://www.youtube.com/watch?v=UxxCDmoMVTI
 Bilging Amidships – Ship Stability (Example 2) https://www.youtube.com/watch?
v=pljRuAHb_Hg
 Bilging Midships. Retrieved July 24, 2020 from https://www.youtube.com/watch?
v=tbxqHgAohro
 Bilging Side. Retrieved July 24, 2020 from https://www.youtube.com/watch?
v=6BUe_9tpR5U
 Bilging End. Retrieved July 24, 2020 from https://www.youtube.com/watch?
v=K2SKTz85HEw
 Bilging of end or intermediate compartments – Ship Stability. Retrieved July 24, 2020
from https://www.youtube.com/watch?v=NbkGQb0Xp88
 Permeability. Retrieved July 24, 2020 from https://www.youtube.com/watch?
v=T8tcma-X6DY
 “Dry Docking of Ships – Understanding Stability and Docking Plan” by Soumya
Chakraborty. Retrieved July 31, 2020 from https://www.marineinsight .com.
 “Understanding Intact & Damage Stability of Ships” by Karan C. Retrieved July 31,
2020 from https://www.marineinsight .com.
 Permeability of bilged compartment – Ship Stability (Theory and Numerical). Retrieved
July 24, 2020 from https://www.youtube.com/watch?v=FjuMpM6osYs&t=26s
 How to calculate change on trim due to change in density – MV Hindship. Retrieved
July 24, 2020 from https://www.youtube.com/watch?v=AJmGbd3D3Jc
 Ship Stability – Calculation and correction of Ships List. Retrieved July 29, 2020 from
https://www.youtube.com/watch?v=tV28-XaGcmg
 Ship Stability List Formula. Retrieved July 29, 2020 from
https://www.youtube.com/watch?v=82bFDRdaETs
 Parallel Axis Theorem. https://www.youtube.com/watch?v=LwMIJJHUuJY
 Parallel Axis Theorem & Inertia. Retrieved July 24, 2020 from
https://www.youtube.com/watch?v=JrkimXqnCLw
Issue No. 1 Page 348 of 348
ST. THERESE- MTC COLLEGES SEAMANSHIP 6
Iloilo, Philippines (Advance Trim, Stability and Stress)
STUDENT LEARNING MODULE
Revision No. 1 Effectivity date: Reviewed by: Approved by:

10 August 2020 QMR President


 Moment of Inertia –Rectangular Body. Retrieved July 24, 2020 from
https://www.youtube.com/watch?v=GOE_dqnRZ94
 Second moment Area of a Rectangle (Parallel Axis Theorem). Retrieved July 24, 2020
from https://www.youtube.com/watch?v=ZTbV8B2J0Oc

You might also like